0% found this document useful (0 votes)
5 views211 pages

Hematology & Oncologyy

The document discusses various hematological conditions including acute lymphoblastic leukemia (ALL), aplastic anemia, and hemophilia A and B, detailing their epidemiology, clinical features, diagnosis, and treatment options. It also covers tumor lysis syndrome and antiphospholipid antibody syndrome, emphasizing the importance of laboratory evaluations and specific therapeutic approaches. Additionally, it includes information on hemoglobin electrophoresis patterns relevant to sickle cell disease and beta-thalassemia, highlighting the diagnostic significance of these tests.

Uploaded by

eldoqshbola
Copyright
© © All Rights Reserved
We take content rights seriously. If you suspect this is your content, claim it here.
Available Formats
Download as PDF, TXT or read online on Scribd
0% found this document useful (0 votes)
5 views211 pages

Hematology & Oncologyy

The document discusses various hematological conditions including acute lymphoblastic leukemia (ALL), aplastic anemia, and hemophilia A and B, detailing their epidemiology, clinical features, diagnosis, and treatment options. It also covers tumor lysis syndrome and antiphospholipid antibody syndrome, emphasizing the importance of laboratory evaluations and specific therapeutic approaches. Additionally, it includes information on hemoglobin electrophoresis patterns relevant to sickle cell disease and beta-thalassemia, highlighting the diagnostic significance of these tests.

Uploaded by

eldoqshbola
Copyright
© © All Rights Reserved
We take content rights seriously. If you suspect this is your content, claim it here.
Available Formats
Download as PDF, TXT or read online on Scribd
You are on page 1/ 211

*Acute lymphoblastic leukemia

Acute lymphoblastic leukemia


• Most common childhood cancer
Epidemiology • Peak age: 2-5 years

• Nonspecific systemic symptoms (eg,


fever, weight loss)
• Leukemic cells overcrowd bone
marrow
o Pallor/fatigue (anemia)
o Easy bruising/petechiae
(thrombocytopenia)
o Bone pain
• Extramedullary spread
Clinical
o Lymphadenopathy
features
o Hepatosplenomegaly
o Testicular enlargement
• Mediastinal mass (T cell lineage):
airway compression &/or superior vena
cava syndrome
• Leptomeningeal spread: neurologic
symptoms (eg, cranial nerve deficits,
meningismus)

• Complete blood count*


• Bone marrow biopsy (>20% blasts is
Evaluation diagnostic) with flow cytometry
& diagnosis • Lumbar puncture to evaluate for CNS
involvement

*≥2 cytopenias (leukocytes may be ↓ or ↑), ± blasts on


peripheral smear.

This patient's bone pain and pancytopenia are concerning for acute lymphoblastic leukemia
(ALL), the most common malignancy in children.

Pathogenesis of ALL involves rampant growth of leukemic cells in the bone marrow, which can
result in bone pain classically affecting the long bones (eg, femur, tibia). The dull pain is often
more noticeable at night, and children may limp or refuse to walk during the day. Although
families may relate the onset of pain to an episode of trauma (eg, fall), the presence of systemic
findings (eg, fever, hepatomegaly), bilateral tibial pain, and hematologic abnormalities (as seen
in this patient) suggests a more systemic process.
As leukemic cell counts in the bone marrow increase, production of all normal cell lines
declines (ie, pancytopenia), resulting in pallor due to anemia and bruising/petechiae due to
thrombocytopenia. The leukocyte count is also initially low due to a concurrent decrease in
normal white blood cell (WBC) production. However, as the disease progresses and
lymphoblasts overcrowd the marrow, laboratory evaluation often shows leukocytosis due to
lymphoblasts spilling into the periphery. In addition, hepatosplenomegaly can occur with
extramedullary leukemic spread.

Diagnosis of ALL is confirmed on bone marrow biopsy. With treatment (ie, multidrug
chemotherapy), prognosis is favorable in children, with a 5-year survival rate of >85%.
*partial anomalous pulmonary venous return

This adolescent with exertional dyspnea has evidence of right-sided cardiac enlargement and a
step-up in oxygen saturation (SaO2) from the inferior vena cava (IVC) to the right atrium
(RA). These findings are consistent with partial anomalous pulmonary venous return
(PAPVR).

Normally, all 4 pulmonary veins deliver oxygenated blood to the left atrium (ie, into the left-
sided circulation). In contrast, in PAPVR, some (ie, ≥1) but not all pulmonary veins
abnormally drain oxygenated blood into the right-sided circulation, usually via the superior
vena cava (SVC) or directly into the RA. As a result, mixing of deoxygenated blood (eg, from
the SVC) and oxygenated blood (eg, from any anomalous pulmonary veins) occurs within or
proximal to the RA. Therefore, SaO2 is relatively increased in the RA, right ventricle (RV), and
pulmonary artery (eg, step-up in SaO2 from ~69% in the IVC to ~84% in the RA). An atrial
septal defect (ASD) is often present, further increasing the left-to-right shunt.

In
contrast to total anomalous pulmonary venous return, in which all pulmonary veins drain
abnormally, causing neonatal cyanosis, in PAPVR, patients are often asymptomatic for much of
childhood. However, chronic right-sided volume overload and pulmonary overcirculation in
PAPVR can lead to right-sided cardiac enlargement, as seen on this patient's x-ray. Over
time, pulmonary vascular remodeling can lead to secondary pulmonary hypertension (eg,
fatigue, exertional dyspnea). Echocardiography with confirmatory cardiac CT scan or MRI
establishes the diagnosis, and management is surgical correction.
*Aplastic anemia
Aplastic anemia
• Multipotent hematopoietic stem cells are destroyed by cytotoxic T cells
Pathogenesis or direct cytotoxic injury → bone marrow aplasia/hypoplasia → lack of
circulating peripheral blood cells

• Autoimmune
• Drugs: cytotoxic chemotherapy, immunosuppressants, idiosyncratic
Common reactions
triggers • Ionizing radiation & toxins
• Viral infections (eg, viral hepatitis, HIV)

• Anemia (eg, fatigue, weakness, pallor)


• Thrombocytopenia (eg, bleeding, bruising)
Manifestations
• Leukopenia (eg, recurrent infections)

• Bone marrow biopsy: hypocellular marrow with abundance of stromal


Diagnosis & fat cells

Spontaneous epistaxis, petechiae, and scattered ecchymosis often indicate an underlying


coagulation disorder or platelet deficiency. Because this patient's coagulation studies (PT/PTT)
are normal, a coagulation disorder is unlikely; however, the presence of pancytopenia and
diminished, morphologically normal cells on peripheral blood smear strongly suggests aplastic
anemia.

Aplastic anemia is a form of acquired bone marrow failure typically caused by direct toxic
injury or cytotoxic T-cell destruction of multipotent hematologic stem cells. This reduces
production of the precursor cells (eg, megakaryocytes) that generate all blood cell
lines. Although most pediatric cases have no discernable cause (idiopathic), a minority are
triggered by viruses (eg, hepatitis, parvovirus), medications, or radiation exposure.

Patients generally present with symptoms of anemia (eg, fatigue, dyspnea), thrombocytopenia
(eg, bruising, bleeding, petechiae), or leukopenia (eg, infection). Examination will show no
hepatosplenomegaly because significant extramedullary hematopoiesis does not occur (stem
cells are damaged and cannot effectively produce cells in any organ). Laboratory results will
reveal pancytopenia with impaired reticulocyte response and diminished, morphologically
normal blood cells on peripheral smear. Definitive diagnosis is made with bone marrow
biopsy, which usually shows profound hypocellularity and fatty infiltration.
*some important definitions
This patient presents with a PSA recurrence of advanced prostate cancer. He initially had
localized disease treated with radical prostatectomy, with good results and undetectable
postoperative PSA. However, he now has a PSA recurrence without any symptoms. Because
he failed the initial therapy for the prostate cancer, he undergoes local external beam radiation
therapy to kill residual tumor cells. The radiation therapy is considered salvage therapy,
defined as a form of treatment for a disease when a standard treatment fails. For men with PSA
recurrence after radical prostatectomy, salvage radiation therapy can provide long-term disease
control for localized recurrent disease.

(Choice A) Adjuvant therapy is defined as treatment given in addition to standard


therapy. This would be the case in this patient if the radiation therapy was given at the same
time as the radical prostatectomy.

(Choice B) Consolidation therapy is typically given after induction therapy with multidrug
regimens to further reduce tumor burden. An example is multidrug therapy after induction
therapy for acute leukemia.

(Choice C) Induction therapy is an initial dose of treatment to rapidly kill tumor cells and send
the patient into remission (<5% tumor burden). A typical example is induction chemotherapy
for acute leukemia.

(Choice D) Maintenance therapy is usually given after induction and consolidation therapies
(or initial standard therapy) to kill any residual tumor cells and keep the patient in
remission. An example is daily antiandrogen therapy for prostate cancer.

(Choice E) Neoadjuvant therapy is defined as treatment given before the standard therapy for a
particular disease. This would be the case in this patient if the radiation therapy was given
before the radical prostatectomy was done.

Educational objective:
Salvage therapy is defined as treatment for a disease when standard therapy fails, such as
radiation therapy for prostate-specific antigen recurrence after radical prostatectomy for prostate
cancer.
*Tumor lysis syndrome

Tumor lysis syndrome


• Tumors with high cell burden or rapid turnover
Risk • Combination chemotherapy/immunotherapy

• Severe electrolyte abnormalities


o ↑ Phosphorus, potassium, uric acid
o ↓ Calcium
Manifestations • Acute kidney injury
• Cardiac arrhythmias
• Seizures

• Intravenous fluids
Prophylaxis • Xanthine oxidase inhibitor* or rasburicase

Treatment • Intravenous fluids + rasburicase


• Continuous telemetry
• Aggressive electrolyte monitoring & treatment

*Allopurinol or febuxostat.

This patient has developed significant laboratory abnormalities (eg, hyperkalemia, hyperphosphatemia,
hypocalcemia, hyperuricemia) a day following chemotherapy, indicating tumor lysis syndrome (TLS). TLS
typically develops in patients who have tumors with high cell burden or rapid cellular turnover (eg, acute
leukemia) and is characterized by rapid release of intracellular potassium, phosphate, and nucleic acids into the
circulation.

Phosphate binds up calcium to cause hypocalcemia, and purine nucleic acids are metabolized into uric acid (ie,
urate) by xanthine oxidase, leading to hyperuricemia and risk of acute kidney injury (AKI) due to renal tubular
crystal deposition. This risk can be reduced by pretreatment with intravenous fluids and urate-lowering
therapy consisting of medications that perform the following:

• Reduce new urate formation: xanthine oxidase inhibitors (eg, allopurinol, febuxostat)
• Directly break down urate: recombinant urate oxidase analogues (eg, rasburicase)

Xanthine oxidase inhibitors are commonly given in conjunction with intravenous fluids prior to chemotherapy
to reduce urate formation. However, in this patient with hyperuricemia and evidence of TLS following
chemotherapy, xanthine oxidase inhibition (eg, with allopurinol) would not be effective because it does not
eliminate urate that has already formed (Choice A). Urate oxidase breaks down urate to reduce hyperuricemia
that has already developed, making rasburicase the preferred urate-lowering therapy for the treatment of TLS.

(Choice B) Hemodialysis is usually needed only in patients with TLS who have severe AKI and electrolyte
derangements that are refractory to medical treatment. This patient with normal renal function should receive
medical treatment with intravenous fluids and rasburicase.

(Choice C) In addition to treatment of hyperuricemia, patients with TLS should undergo monitoring of other
electrolyte derangements (eg, hyperkalemia, hyperphosphatemia, hypocalcemia) and treatment if necessary.

(Choice E) Urinary alkalinization prior to chemotherapy theoretically reduces renal tubular precipitation of
urate by increasing its solubility. However, this is not routinely performed due to a lack of evidence and the
potential risk of promoting calcium phosphate deposition in the renal tubules (ie, nephrocalcinosis).

Educational objective:
Tumor lysis syndrome can occur after chemotherapy and results in significant elevations in serum potassium,
phosphate (with decreased serum calcium), and uric acid (ie, urate). In patients with preexisting hyperuricemia
or in those who develop hyperuricemia following chemotherapy, the administration of rasburicase directly
breaks down urate and rapidly lowers its serum concentration.
*Diag. criteria for antiphospholipid antibody $y

Diagnostic criteria for antiphospholipid-antibody syndrome

(1 clinical & 1 laboratory criterion must be met)


Vascular thrombosis

• Arterial or venous

Pregnancy morbidity

Clinical • ≥3 consecutive, unexplained fetal losses before


10th week
• ≥1 unexplained fetal losses after 10th week
• ≥1 premature births of normal neonates before 34th
week due to preeclampsia, eclampsia, or placental
insufficiency

• Lupus anticoagulant
• Anticardiolipin antibody
Laboratory
• Anti–beta-2 glycoprotein antibody I

This patient has pulmonary emboli due to a right lower extremity deep venous thrombosis (DVT). Although
risk of DVT is increased in several inherited and acquired conditions, this patient's history of fetal loss and signs
of systemic lupus erythematosus (eg, myalgias, arthralgias, photosensitivity) suggest antiphospholipid-
antibody syndrome (APS).

APS is characterized by the following:

• Venous, arterial, or small vessel thromboembolic disease and/or pregnancy complications such as fetal
loss
• Laboratory evidence of an antiphospholipid antibody in the serum: lupus anticoagulant (LA),
anticardiolipin antibody, and/or beta2-glycoprotein-I antibody

Patients with antiphospholipid antibodies (particularly LA) generally have prolonged PTT because
antiphospholipid antibodies bind the phospholipids used in most testing assays and prevent them from inducing
coagulation. This laboratory artifact will not be fixed by a 1:1 mixing with normal plasma (unlike coagulation
factor deficiency). However, adding phospholipids to the testing assay will eventually overcome the presence
of the inhibitor and normalize the PTT. This is an indirect way of identifying LA antibodies; it is usually
combined with other tests, such as the diluted Russell viper venom test or the kaolin clotting time test for
diagnosis.
*Hemophilia A & B

Hemophilia A & B
• X-linked recessive
Inheritance
• Delayed/prolonged bleeding after mild trauma
o Hemarthrosis, intramuscular hematomas
Clinical o Gastrointestinal or genitourinary tract bleeding
features o Intracranial hemorrhage
• Complications: hemophilic arthropathy

• ↑ Activated PTT
• Normal platelet count & PT
Laboratory
findings • Absent or ↓ factor VIII (hemophilia A) or factor IX
(hemophilia B) activity

• Factor replacement
Treatment • Desmopressin for mild hemophilia A

This child's history of easy bruising and acute onset of joint effusion following minor trauma
(suggestive of hemarthrosis) raise concern for hemophilia, a coagulation disorder. Hemophilia
A and B, or deficiency of factor VIII and IX, respectively, are X-linked recessive disorders that
typically affect boys.

The severity of symptoms depends on the degree of factor deficiency, ranging from
spontaneous bleeding (eg, intracranial hemorrhage) to delayed postprocedure bleeding (eg,
oozing from dental extraction site). Hemarthrosis, or joint hemorrhage, is common and
typically presents following mild injury with pain and swelling in weight-bearing joints (eg,
knees, ankles). Recurrent hemarthroses in the same joint can lead to chronic hemophilic
arthropathy.

Initial screening tests for hemophilia include coagulation studies and a complete blood
count. Hemophilia causes prolonged PTT due to absent factor VIII or IX but normal PT;
platelet count is normal.
*Hg electrophoresis patterns in SCA & B-thalassemia

Hemoglobin electrophoresis patterns in sickle cell & beta-thalassemia


Condition Hemoglobin A Hemoglobin A2 Hemoglobin F Hemoglobin S
Normal 95%-98% ~2.5% <1% Absent
Beta-thalassemia minor ↓ ↑ Near normal Absent
Beta-thalassemia major Absent ↑↑ ↑↑ Absent
Sickle cell trait ↓↓ Near normal Near normal ↑
Sickle cell disease Absent Near normal ↑↑ ↑↑

This patient's hemoglobin electrophoresis results are consistent with beta-thalassemia. Mutations in both beta
globin genes result in absent beta globin expression, known as beta-thalassemia major. Instead of the two
alpha and two beta chains that typically compose hemoglobin A, patients have increased hemoglobin F (two
alpha and two gamma chains) and hemoglobin A2 (two alpha and two delta chains).

Since fetal hemoglobin (HbF) predominates in the first few months of life until adult hemoglobin (HbA) is
synthesized, beta thalassemia major is generally asymptomatic in the newborn period. Patients around age 6-12
months develop fatigue and pallor due to microcytic anemia. Splenic hemolysis of red blood cells (RBCs) can
cause jaundice, dark urine, and splenomegaly. If left untreated, patients can develop skeletal abnormalities due
to extramedullary hematopoiesis.

Beta-thalassemia major is transfusion-dependent. The additional iron from transfused RBCs increases the risk
for iron overload. Chelation therapy is required to avoid damage to the liver, kidneys, and endocrine glands
and improve survival.

(Choice B) Hydroxyurea therapy increases HbF in patients with sickle cell disease (by inducing a change in
gene expression at the beta globin locus), thereby reducing the relative concentration of hemoglobin S and
minimizing complications caused by sickling.

(Choice C) Iron supplementation should be avoided in patients with beta-thalassemia as they are at risk for iron
overload given their need for transfusions.

(Choice D) Penicillin prophylaxis is required for anatomic or functional asplenia due to increased risk of
infection with encapsulated organisms. Patients with beta-thalassemia have a functioning spleen.

(Choice E) Vitamin B12 deficiency causes macrocytic anemia and is more common in patients with a vegan
diet.

Educational objective:
Beta-thalassemia major is characterized by increased hemoglobin A2 and hemoglobin F on electrophoresis due
to absent beta globin expression. Patients are transfusion-dependent and at risk for iron overload. Therefore,
chelation therapy is required to improve survival.
*dilutional effect of large-volume resuscitation
This patient with serious pelvic trauma leading to blood loss anemia requiring large-volume
transfusion now has thrombocytopenia, prolonged PT/PTT, and normal fibrinogen and D-dimer
levels. Although traumatic injury is a common trigger for consumptive coagulopathy
(disseminated intravascular coagulation), this condition is marked by the widespread deposition
of fibrin- and platelet-rich thrombi, which rapidly consumes fibrinogen and is associated with
D-dimer elevation (Choice D). Given this patient's normal fibrinogen and D-dimer levels, the
most likely explanation for her coagulation abnormalities is the dilutional effect of large-
volume resuscitation.

Packed red blood cells restore blood volume and improve oxygen-carrying capacity but
contain minimal platelets and clotting factors. Therefore, the administration of large volumes of
packed red blood cells (generally ≥4 units in 30 minutes or ≥10 units during hospitalization) can
dilute circulating clotting factors, leading to prolonged PT/PTT, and platelets, leading to mild
thrombocytopenia. This is generally exacerbated by the concomitant administration of
intravenous fluids.

To avoid bleeding complications, platelet level and coagulation times are generally monitored;
replacement (eg, fresh frozen plasma, platelets, whole blood) is indicated when the INR is >1.5
or platelets are <50,000/mm3.
*Carbon monoxide poisoning

Carbon monoxide poisoning


• Smoke inhalation
• Defective heating systems
Epidemiology
• Gas motors operating in poorly ventilated areas

Mild-moderate

• Headache, confusion
• Malaise, dizziness, nausea
Manifestations
Severe

• Seizure, syncope, coma


• Myocardial ischemia, arrhythmias

• ABG: carboxyhemoglobin level


Diagnosis • ECG ± cardiac enzymes

• High-flow 100% oxygen


Treatment • Intubation/hyperbaric oxygen (severe)

ABG = arterial blood gas.

This patient, who is exposed to automobile exhaust in an enclosed space, has intermittent headaches,
dizziness, nausea, and polycythemia (elevated hematocrit), most likely due to chronic carbon monoxide
(CO) poisoning.

CO is a byproduct of combusting organic matter (eg, oil, gas, wood). Exposure to toxic levels is more likely in
enclosed or poorly ventilated areas. CO tightly binds hemoglobin - forming carboxyhemoglobin - with an
affinity much greater than that of oxygen. Nonsmokers have low levels (<3%) of carboxyhemoglobin (due to
normal enzymatic reactions). Cigarette smokers may have carboxyhemoglobin levels as high as 10%. Even
though patients at this level are generally asymptomatic, small additional amounts of CO exposure may cause
toxicity manifesting as headache, malaise, and nausea.

Carboxyhemoglobin shifts the oxygen dissociation curve to the left, impairing the ability of heme to unload
oxygen at the tissue level. This results in tissue hypoxia. The kidney responds to tissue hypoxia by producing
more erythropoietin (EPO). EPO stimulates the bone marrow to differentiate more red blood cells. Chronic
CO toxicity is a cause of secondary polycythemia.

Pulse oximetry does not differentiate between carboxyhemoglobin and oxyhemoglobin; it cannot be used in the
diagnosis of CO poisoning. Diagnosis is made by arterial blood gas with cooximetry
*Wiskott-Aldrich syndrome

Wiskott-Aldrich syndrome
• X-linked recessive defect in WAS protein gene
Etiology • Impaired cytoskeleton changes in leukocytes, platelets

• Eczema
Clinical • Microthrombocytopenia (small platelets, low platelet count)
features • Recurrent infections

• Stem cell transplant


Treatment

This patient's eczema (itchy, red, scaly skin), microthrombocytopenia (small and low number of platelets),
and recurrent infections are consistent with Wiskott-Aldrich syndrome (WAS). WAS is caused by an X-
linked recessive defect in the WAS gene. This gene is primarily expressed in hematopoietic cells and regulates
cytoskeleton remodeling in response to cell signaling. In WAS, the actin cytoskeleton in white blood cells is
abnormal, resulting in immune dysfunction due to impaired cellular migration and immune synapse
formation. Patients are at increased risk for recurrent bacterial, viral, and fungal infections.

Similarly, the cytoskeleton of platelets is also dysfunctional. Virtually all patients have significantly decreased
platelet counts and size at the time of diagnosis. The resulting clinical findings can range from petechiae or
purpura to severe bleeding such as intracranial hemorrhage, hematemesis, or hematochezia. In addition,
autoimmune disorders (eg, eczema) occur in most patients with WAS. The treatment of WAS is hematopoietic
stem cell transplantation.

(Choice A) T cell lymphocytes that lack CD40 ligand cannot bind to the CD40 receptor on B cells. Without
CD40 activation, B cells cannot switch the antibody isotype they produce (eg, from IgM to IgG or IgA),
resulting in hyper-IgM syndrome.

(Choice C) Ataxia telangiectasia is a T cell deficiency associated with a defect in DNA repair. In addition to
immune dysfunction, patients with ataxia telangiectasia experience progressive cerebellar degeneration and are
at high risk for cancer.

(Choice D) Chronic granulomatous disease results from an inability of phagocytes to produce hydrogen
peroxide in their lysosomes. Abscesses due to fungi or catalase-positive bacteria (eg, Staphylococcus aureus)
are the characteristic feature.

(Choice E) Severe combined immunodeficiency (SCID) results from a severe T cell deficiency. A variety of
gene defects can cause SCID and all prevent interleukin-7-driven maturation of T cells in the thymus. Patients
with SCID have virtually no functional T cells; lack of T cells causes severe B cell dysfunction as well.

Educational objective:
Wiskott-Aldrich syndrome is an X-linked disorder characterized by the triad of thrombocytopenia, eczema, and
recurrent infections. The thrombocytopenia is the most consistent feature and is characterized by a significant
reduction in platelet volume and size.
*Vitamin C deficiency (scurvy)

Vitamin C deficiency (scurvy)


• Insufficient dietary intake (lack of citrus fruits/vegetables)
o Chronic alcohol/substance use
Pathogenesis o Severe malnutrition (eg, anorexia, restricted diet)
• Impaired collagen production

• Cutaneous:
o Coiled hair, perifollicular hemorrhages
o Petechiae, ecchymoses
o Impaired wound healing
Manifestations
• Mucosal: gingivitis with bleeding
• Musculoskeletal (eg, periosteal hemorrhage): arthralgia, limp
• Constitutional: malaise, depression, vasomotor instability

• Anemia (normocytic or microcytic)


Laboratory
• Normal platelet count & coagulation studies
findings
• Oral/injectable vitamin C (improvement within days to weeks)
• Toxicity (eg, abdominal pain, diarrhea) with excessive
Treatment
supplementation

This patient with a severely limited diet has leg pain, petechiae, and gingivitis, findings
concerning for vitamin C deficiency (scurvy). This condition is rare in resource-rich countries
but can occur in patients who are malnourished; children with autism, as in this case, may have
self-restrictive diets that lack vitamin C–rich foods (eg, vegetables, citrus fruits), increasing the
risk of deficiency.
Vitamin C is a cofactor for the hydroxylation of proline and lysine, which is essential for collagen cross-
linking. Impaired collagen synthesis compromises tensile strength, particularly in the skin and vascular walls,
resulting in perifollicular hemorrhages, hemorrhagic gingivitis, petechiae, and ecchymoses. Intramuscular or
subperiosteal hemorrhages can occur, especially in the lower extremities due to gravity-dependent hydrostatic
pressure within fragile capillaries. This results in pain and difficulty with ambulation, which may be the
presenting symptoms in children, as seen here. Generalized malaise is also common.

The diagnosis can be confirmed by a low ascorbic acid level. Laboratory evaluation may also reveal anemia
due to blood loss within tissues (normocytic) or associated iron deficiency (microcytic, as in this case), the latter
of which occurs in part because vitamin C normally enhances iron absorption in the small intestine.

Educational objective:

Vitamin C deficiency (scurvy) may occur in patients who are malnourished (eg, severely restricted
diet). Impaired collagen synthesis results in fragile vascular walls, causing petechiae/ecchymoses, gingival
bleeding, and musculoskeletal hemorrhage (eg, leg pain, limp). A low ascorbic acid level confirms the
diagnosis.
*phenytoin & folic acid supplements
This patient with a history of seizure disorder is being treated with phenytoin, an antiepileptic
with several common adverse effects, including:

• Folic acid deficiency: Phenytoin impairs folic acid absorption in the jejunum, which
often results in folic acid deficiency over weeks or months. Because folate
(tetrahydrofolate) is required for purine and thymidine synthesis, deficiency impairs
DNA synthesis, particularly in cells that undergo rapid turnover (eg, hematologic
cells). This often leads to megaloblastic anemia (eg, macrocytosis, hypersegmented
neutrophils).
• Gingival overgrowth/hyperplasia: Phenytoin has several effects on the gingival
mucosa but is thought to reduce folic acid use in the extracellular matrix of the gums,
leading to impaired collagenase activity and a subsequent buildup of gingival matrix.

These adverse effects can usually be prevented with oral folic acid supplementation. Because
phenytoin also disrupts bone and mineral metabolism (leading to reduced bone density),
calcium and vitamin D supplementation are also generally administered.

Other medications that commonly cause megaloblastic anemia include methotrexate,


trimethoprim, and pyrimethamine due to disruptions in dihydrofolate reductase (the enzyme that
mediates tetrahydrofolate production).
*non-Hodgkin lymphoma in AIDS patients
This patient with HIV has diffuse lymphadenopathy (LAD), intermittent fevers, and night
sweats. Although this constellation of findings can be seen in untreated HIV, the presence of an
undetectable viral load indicates compliance with antiretroviral therapy. Therefore, the
differential diagnosis primarily includes hematologic malignancy, rheumatic disease, and other
systemic infections. Given the presence of supraclavicular, epitrochlear, and mediastinal LAD,
a lymphoid neoplasm such as non-Hodgkin lymphoma (NHL) is most likely.

Patients with HIV are at a dramatically elevated risk of malignancy due to immune
dysregulation and impaired cytokine signaling. This limits the ability of patrolling cytotoxic T-
cells to induce apoptosis in cells that have accumulated genomic mutations. It also promotes
reactivation of latent oncogenic viruses, which can cause AIDS-defining malignancies such as
invasive cervical carcinoma (human papillomavirus), Kaposi sarcoma (human herpesvirus 8),
CNS system lymphoma (Epstein-Barr virus [EBV]), and NHL.

Approximately 10% of patients with AIDS develop NHL; nearly all cases in this population are
linked to the reactivation of EBV and the subsequent production of oncogenic viral
proteins. Major manifestations include diffuse LAD, which may involve the epitrochlear,
supraclavicular, and mediastinal (enlarged cardiac silhouette) lymph nodes. B symptoms (eg,
night sweats, fevers, weight loss) are generally present. Diagnosis is usually made by
excisional lymph node biopsy.

EBV is also associated with development of Hodgkin lymphoma, which can develop in patients
with HIV, although NHL is more common in this population (possibly reflecting the immune
interaction between Reed-Sternberg cells and nonmalignant inflammatory cells needed for the
development of Hodgkin lymphoma).
*traumatic macrovascular hemolysis
This patient with a mechanical aortic valve developed symptomatic anemia and reticulocytosis,
raising strong suspicion for traumatic macrovascular hemolysis. Most cases arise in the
setting of a dysfunctional mechanical or severely calcified aortic valve; high pressure
gradients across the irregular valve cause shearing of passing erythrocytes, leading to
intravascular hemolysis. Damaged erythrocytes that are not lysed in the intravascular space
are generally destroyed by splenic macrophages, leading to concurrent extravascular hemolysis.

Mechanical aortic valves often cause mild, asymptomatic hemolysis. However, valve
deterioration can trigger progressive symptomatic anemia (eg, fatigue, dyspnea with exertion)
with signs of erythrocyte lysis (eg, jaundice, dark urine) and
reticulocytosis. Thrombocytopenia can also occur because platelets may be damaged as they
pass through the rigid valve opening. Hepatosplenomegaly can develop as the fragmented
erythrocytes are cleared in the macrophages and monocytes within the liver and
spleen. Peripheral blood smear generally shows schistocytes (helmet cells). A transthoracic
echocardiogram should be performed to visualize valve function and determine valvular
pressure gradients
*neonatal unconjugated hyperbilirubinemia

This neonate with unconjugated hyperbilirubinemia is receiving phototherapy, which


reduces bilirubin by converting it into a photoisomer that can be excreted in bile and
urine. Although benign neonatal hyperbilirubinemia due to liver immaturity and decreased
bilirubin clearance is most common, this condition is usually associated with a decline in
bilirubin levels after initiation of phototherapy; further treatment is typically unnecessary.

In contrast, repeat laboratory evaluation in this patient on phototherapy shows a rapid rise in
bilirubin. This circumstance warrants exchange transfusion, an invasive but far more rapid
therapy to lower levels. Other indications for exchange transfusion include:

• Severe hyperbilirubinemia (>25 mg/dL), also seen here


• Symptomatic bilirubin-induced neurologic dysfunction (BIND) (eg, lethargy, change
in tone)

Exchange transfusion replaces an infant's blood volume and is extremely effective at lowering
bilirubin levels, thereby preventing unbound bilirubin from crossing the blood-brain barrier and
causing brain injury (ie, BIND). A low serum albumin, as in this case, can be physiologic in
neonates but further increases the risk of BIND because albumin-bilirubin binding normally
helps to limit the amount of unbound, neurologically toxic bilirubin.
In this case, the patient also has a significant decline in hemoglobin, suggestive of hemolysis
causing the excessive production of indirect bilirubin. Therefore, this patient should also
undergo evaluation for the underlying cause of hemolysis (eg, hereditary spherocytosis,
glucose-6-phosphatase dehydrogenase deficiency).

(Choice A) Intravenous immunoglobulin is not typically used in management of neonatal


hyperbilirubinemia but can be considered for severe hemolytic disease of the newborn due to
RhD incompatibility, which can only occur in RhD-positive infants born to RhD-negative
mothers.

(Choice B) Phototherapy is typically the first-line treatment for neonatal unconjugated


hyperbilirubinemia. Although increasing phototherapy intensity while awaiting exchange
transfusion can improve hyperbilirubinemia, this patient with rapidly rising bilirubin levels >25
mg/dL is at high risk for BIND and warrants urgent exchange transfusion to quickly reduce
levels.

(Choices C and E) Intravenous fluid administration or formula supplementation in breastfed


infants can be adjuncts to phototherapy in patients with dehydration to improve urinary
excretion of bilirubin. However, this newborn has had adequate elimination (ie, ≥3 wet diapers
on day 3) and no signs of dehydration (eg, sunken fontanelle, tachycardia). Moreover, the
degree of hyperbilirubinemia in this case warrants more invasive intervention.

Educational objective:
Indications for exchange transfusion in neonates with unconjugated hyperbilirubinemia include
a rapid rise in bilirubin despite phototherapy or levels >25 mg/dL. Exchange transfusion
decreases the risk of bilirubin-induced neurologic dysfunction.
*Immune thrombocytopenia

Immune thrombocytopenia
• Platelet autoantibodies
Etiology • Preceding viral infection

• Petechiae, ecchymosis
Clinical findings • Mucosal bleeding (eg, epistaxis, hematuria)

Laboratory • Isolated thrombocytopenia <100,000/mm3


• Few platelets (size normal to large) on peripheral smear
findings
• Children
o Observe if cutaneous symptoms only
o Glucocorticoids, IVIG, or anti-D if bleeding
• Adults
o Observation if cutaneous symptoms AND platelets
Treatment ≥30,000/mm3
o Glucocorticoids, IVIG, or anti-D if bleeding or platelets
<30,000/mm3

IVIG = intravenous immunoglobulin.

Immune thrombocytopenia (ITP), most common in children age 2-5, is characterized by an


abnormally low platelet count (<100,000/mm3) that results in an increased propensity for
bruising and bleeding. In ITP, autoantibodies are directed against platelet membrane antigens,
leading to destruction of antibody-platelet complexes in the spleen.

ITP is usually preceded by a viral infection, as in this case, and presents with petechiae and
purpura. In severe cases, patients may experience mucosal bleeding (eg, epistaxis, hematuria,
gastrointestinal bleeding). Laboratory studies show isolated thrombocytopenia, and a few
variably sized (normal to large) platelets may be present on peripheral smear due to increased
platelet turnover.

Observation alone is recommended for children with only cutaneous symptoms (eg,
petechiae), regardless of platelet count. The course is usually self-limited with spontaneous
recovery within 3 months. Glucocorticoids, anti-D immune globulin (if Rh-positive and
Coombs-negative), or intravenous immunoglobulin is first-line treatment in children with
mucosal bleeding
*Waldenström macroglobulinemia vs multiple myeloma

Waldenström macroglobulinemia vs multiple myeloma


Waldenström macroglobulinemia Multiple myeloma
• Hyperviscosity syndrome
• Osteolytic lesions/fractures
• Neuropathy
• Anemia
• Bleeding
Major manifestations • Hypercalcemia
• Hepatosplenomegaly
• Renal insufficiency
• Lymphadenopathy

• IgM • IgG, IgA, light chains


Monoclonal antibody
• Rouleaux • Rouleaux
Peripheral smear
• >10% clonal B cells • >10% clonal plasma cells
Bone marrow biopsy

This patient with a week of headaches, dizziness, and blurry vision has a large gamma gap
(difference between total protein and albumin) and "sausage-link" retinal changes, suggesting
hyperviscosity syndrome due to Waldenström macroglobulinemia (WM).

WM is a B-cell malignancy characterized by the excessive production of monoclonal IgM


antibody. IgM is a large immunoglobulin, and high levels can clog the microvasculature,
resulting in hyperviscosity syndrome. Symptoms typically include vision changes, headaches,
vertigo, dizziness, and/or ataxia; rarely, stroke or coma may occur. Other IgM-mediated
conditions commonly seen in patients with WM include peripheral neuropathy,
cryoglobulinemia, and renal insufficiency. Physical examination typically shows "sausage-
link" (dilated, segmented, tortuous) retinal veins, and laboratory studies usually reveal anemia,
a gamma gap, and elevated erythrocyte sedimentation rate (ESR).

The screening test for WM is serum protein electrophoresis (SPEP), which can identify a
monoclonal IgM spike. If positive, bone marrow biopsy is performed (lymphocytic infiltration
with certain immunophenotype markers). This patient would also require serum viscosity
testing as urgent plasmapheresis may be required.
*androgen abuse

Clinical features of androgen abuse


• Exogenous (eg, testosterone replacement therapy)
Types of • Synthetic (eg, stanozolol, nandrolone)
androgens • Androgen precursors (eg, DHEA)

• Reproductive
o Men: decreased testicular function & sperm production,
gynecomastia
o Women: acne, hirsutism, voice deepening, menstrual
Side effects/
clinical irregularities
presentation • Cardiovascular: left ventricular hypertrophy, possible ↓ HDL & ↑
LDL
• Psychiatric: aggressive behavior (men), mood disturbances
• Hematologic: polycythemia, possible hypercoagulability

DHEA = dehydroepiandrostenedione.

Androgen abuse is common amongst high-performance athletes. Androgens are anabolic;


they increase muscle mass and maximum voluntary strength. They are taken directly (natural or
synthetic androgens) or induced to rise indirectly (estrogen blockers, androgen precursors,
gonadotropins). Adverse effects are common. In men, androgen abuse can cause testicular
atrophy, reduced spermatogenesis, gynecomastia, mood disturbances, and aggressive
behavior. In women, androgens can cause acne, hirsutism, deepening of the voice, and
menstrual irregularities. Laboratory findings include erythrocytosis, hepatotoxicity, and
dyslipidemia. The mechanism for erythrocytosis in androgen abuse remains unclear, but
increases in hematocrit and hemoglobin are dose dependent. This patient is likely abusing
androgens in preparation for his upcoming football season.
*Anaphylactic vs Urticarial Transfusion reactions

Transfusion reactions
Anaphylactic Urticarial
• Anti-IgA antibodies (IgG or IgE) • Preformed recipient IgE
in IgA-deficient patient against antibodies against soluble
Mechanism
donor blood IgA allergen in donated plasma

• Seconds to minutes • Hours


Onset
• Respiratory distress/wheeze
• Angioedema • Hives
Findings • Hypotension • Itching
• Hives

• Immediate cessation of
• Immediate cessation of transfusion
transfusion
• Epinephrine
• Antihistamines
Treatment • Antihistamines, oxygen, fluids &
• Resume transfusion if patient is
vasopressors
otherwise asymptomatic

This hemodynamically stable patient developed isolated urticaria during a transfusion, which is
consistent with an urticarial transfusion reaction. Urticarial transfusion reactions are
common and typically mild. Pathogenesis involves histamine release due to preformed
recipient IgE antibodies reacting against a soluble allergen in the donated plasma, or vice
versa (ie, donor IgE against recipient allergen).

Symptoms are limited to hives and/or itching and develop hours after transfusion initiation. In
contrast, hives due to anaphylaxis typically occur within minutes of transfusion initiation and
develop in conjunction with wheezing, angioedema, and/or hypotension.

Management of an urticarial transfusion reaction includes immediate cessation of the


transfusion and administration of diphenhydramine (ie, antihistamine) if urticaria is
extensive. Diagnosis is confirmed if symptoms improve and no signs of anaphylaxis
develop. The transfusion can then be resumed without additional evaluation.

(Choice A) Transfusion-transmitted bacterial infection (TTBI) is an uncommon, severe


complication of transfusion that presents with fever, chills, and hypotension from a
contaminated blood product (most commonly platelets). Cultures should be obtained from the
recipient as well as from the transfused product to identify the pathogen and potential source of
infection, respectively. This patient has no symptoms of TTBI, and therefore testing is not
warranted.
(Choices B, D, and E) Acute hemolytic transfusion reaction is a rare but potentially fatal
reaction caused by mismatched donor and recipient blood (ie, ABO incompatibility) due to
clerical error. Classic symptoms include fever, flank pain, hypotension, and dark/pink urine (ie,
hemoglobinuria), none of which are seen in this patient. Evaluation typically reveals positive
direct antibody (Coombs) testing. In cases complicated by disseminated intravascular
coagulation, schistocytes on peripheral blood smear and prolonged PT and PTT are also
seen. This patient's petechiae (pinpoint, nonblanching macules) and ecchymoses are consistent
with her established thrombocytopenia and were present prior to transfusion.

Educational objective:
Urticarial transfusion reactions are common and caused by preformed recipient IgE antibodies
reacting against a soluble allergen in the donated plasma (or vice versa). Patients have isolated
urticaria that improves with transfusion cessation and diphenhydramine administration. If no
signs of anaphylaxis are present (eg, wheeze, angioedema, hypotension), the transfusion can be
resumed without additional evaluation.
*inhibitor development in hemophilia A

Factor VIII deficiency (hemophilia A) is an X-linked recessive bleeding disorder associated


with recurrent hemarthrosis (bleeding into joints), as seen in this patient. Treatment is factor
VIII replacement therapy (eg, recombinant factor VIII infusions through a port). However,
inhibitor development occurs in approximately 25% of patients with severe hemophilia A (ie,
frequent, spontaneous bleeds) as a complication of treatment. The immune system recognizes
the infused factor as foreign, forming antibodies that interfere with factor function.

Therefore, an inhibitor should be suspected when a patient with hemophilia A on factor


replacement therapy develops increased bleeding frequency or has hemorrhage refractory to
treatment. In addition, because factor VIII belongs to the intrinsic coagulation pathway,
prolonged PTT, but normal PT, occurs if factor VIII is inhibited. Regular screening for
inhibitor development is performed when patients with severe hemophilia receive factor
infusions as well as when an inhibitor is clinically suspected.

Treatment of an acute bleed in a patient with inhibitor development often involves bypassing
products (eg, recombinant activated factor VII, activated prothrombin complex concentrates);
such agents work downstream in the coagulation cascade to promote clotting without the need
for factor VIII.

(Choices A and E) Acquired von Willebrand disease and disorders of platelet function present
with bruising and mucosal bleeding rather than with hemarthrosis.

(Choice B) Fibrinogen (factor I) is in the common final coagulation pathway, and dysfunction
causes prolonged PT and PTT. This patient's PT is normal.

(Choice C) Heparin-induced thrombocytopenia is due to an autoantibody against heparin and


typically presents with thrombosis and decreased platelet count, which are not seen in this
patient.

Educational objective:
Inhibitor development occurs in 25% of patients with severe factor VIII deficiency. It should
be considered in a patient with bleeding refractory to replacement therapy or with increased
bleeding frequency.
*Hematologic manifestations of SLE

Hematologic manifestations of SLE


Common mechanisms Uncommon mechanisms
• Anemia of chronic disease
• SLE nephritis • Medications
• Iron deficiency anemia • MAHA
Anemia
(gastrointestinal loss) • Aplastic anemia
• Immune hemolytic anemia

• Medications
• Immune-mediated destruction • Bone marrow
Leukopenia
dysfunction

• Medications
• Immune-mediated destruction
Thrombocytopenia • TTP (↑ consumption)

MAHA = microangiopathic autoimmune hemolytic anemia; SLE = systemic lupus


erythematosus; TTP = thrombotic thrombocytopenic purpura.

This patient's presentation (joint pains, oral ulcer, hypertension, pancytopenia, impaired renal
function) suggests possible systemic lupus erythematosus (SLE), a systemic autoimmune
disorder. Her severe thrombocytopenia (platelet count <50,000/mm3), manifesting with
common associated features of bleeding gums and heavy menstrual periods, is likely due to
SLE. In the United States, SLE occurs most commonly in African American, Hispanic, and
Asian women. Hematologic abnormalities occur in over half of SLE patients and include:

• Anemia, resulting from immune destruction, but also from coexisting renal disease or
chronic inflammation
• Leukopenia (eg, neutropenia), often immune-mediated and correlating with disease
activity
• Thrombocytopenia, most commonly due to immune thrombocytopenic purpura (ITP)

The presence of autoantibodies is the primary mechanism underlying the most common
cytopenias in SLE. Thrombocytopenia is most often caused by antiplatelet antibodies, which
trigger increased destruction of platelets via immunoglobulin binding in peripheral blood
vessels and subsequent macrophage phagocytosis. Less commonly, thrombocytopenia can
occur in SLE-associated hypercoagulable states in which platelets are consumed within thrombi
(eg, thrombotic thrombocytopenic purpura, associated with schistocytes; disseminated
intravascular coagulation, which can occur in antiphospholipid antibody syndrome).
*leukemoid reaction

Comparison of leukemoid reaction & chronic myeloid leukemia (CML)


Leukemoid reaction CML
Elevated
Leukocyte count >50,000/mm3 (>50 × 109)
(often >100,000/mm3 [>100 × 109])
Cause Severe infection BCR-ABL fusion
LAP score High Low
More mature Less mature
Neutrophil precursors
(metamyelocytes > myelocytes) (metamyelocytes < myelocytes)
Absolute basophilia Not present Present
LAP = leukocyte alkaline phosphatase.

This patient presents with manifestations of acute infectious diarrhea possibly due to
Clostridiodes difficile (recent antibiotic use, severe diarrhea, high fever, left lower quadrant
pain). His leukocyte count is dramatically elevated, with a predominance of late neutrophil
precursors (metamyelocytes, bands) rather than early precursors (promyelocytes,
myelocytes). This, combined with a high leukocyte alkaline phosphatase (LAP) score,
suggests leukemoid reaction (LR), characterized by a leukocytosis of >50,000/mm3. Severe
infections can cause LR due to the mobilization of mature and immature leukocytes from the
bone marrow. Given the severity of the infection, it may take >24 hours for clinical
improvement with antibiotics.

LR must be differentiated from chronic myelogenous leukemia (CML), a myeloproliferative


disorder that also presents with neutrophilia and dramatic leukocytosis. Unlike LR, CML is
characterized by a low LAP score (due to cytochemically abnormal neutrophils) and a
peripheral smear that features absolute basophilia and early neutrophil precursor cells (Choice
B).

Educational objective:
Leukemoid reaction (LR) occurs as a response to severe infection and is marked by leukocyte
counts >50,000/mm3. Compared to chronic myeloid leukemia, LR is characterized by the
presence of a high alkaline phosphatase score, a greater proportion of late neutrophil precursors
(metamyelocytes, bands), and a lack of absolute basophilia.
*Stroke in sickle cell disease

Stroke in sickle cell disease


• Ischemic stroke
o Intimal hyperplasia & stenosis (vasculopathy)
o Adhesion of sickled red blood cells to vasculature
Pathogenesis • Hemorrhagic stroke
o Weakened cerebral vessels
o Cerebral aneurysm rupture

• Focal neurologic symptoms (eg, hemiparesis)


• Seizure
Presentation
• Altered mental status

• MRI of the head: ischemic stroke (large vessel region or watershed area)
Imaging • CT scan of the head: hemorrhagic stroke

• Exchange transfusion
Treatment • Simple transfusion if exchange transfusion is unavailable

• Transcranial Doppler screening during childhood


• Hydroxyurea
Primary prevention
• ± Chronic transfusions

This child with sickle cell disease (SCD) has sudden-onset unilateral weakness without intracranial
hemorrhage, findings concerning for an ischemic stroke.

SCD increases the risk of ischemic stroke due to intimal hyperplasia and stenosis of large arteries, as well as
occlusion of vasculature by adherent, stiff, sickled red blood cells. Vascular and oxidative injury leads to
recruitment of inflammatory cells and activation of the coagulation cascade. Precipitating factors may include
acute drops in oxygenation or hemoglobin (often seen in pain crises). This patient is at increased risk of SCD
complications due to not taking hydroxyurea, which decreases frequency of pain crises by increasing fetal
hemoglobin.

Treatment of stroke in SCD patients is exchange transfusion (ie, slowly removing the patient's blood and
replacing it with allogeneic blood), which lowers the relative percentage of sickled cells compared to healthy
red blood cells. By decreasing RBC sickling, exchange transfusion increases the oxygen-carrying capacity of
the blood and improves hyperviscosity, limiting further occlusion. Exchange transfusion also decreases the
risk of stroke recurrence. Simple transfusion (ie, packed red blood cell transfusion) should be performed when
exchange transfusion is not available.
*heparin-induced thrombocytopenia (HIT)
This patient's markedly elevated aPTT is suggestive of unfractionated heparin use. The
presentation of new-onset thrombocytopenia and arterial thrombosis in the setting of heparin
use is due to heparin-induced thrombocytopenia (HIT). The 2 forms of HIT differ in onset,
clinical course, and severity of disease.

Type 1 HIT occurs due to a nonimmune direct effect of heparin on platelet activation and
usually presents within the first 2 days of heparin exposure. The platelet count then normalizes
with continued heparin therapy and there are no clinical consequences.

Type 2 HIT is a more serious immune-mediated disorder due to antibodies to platelet factor 4
(PF4) complexed with heparin. This leads to platelet aggregation, thrombocytopenia, and
thrombosis (both arterial and venous). Platelet counts usually drop >50% from baseline, with a
nadir of 30,000-60,000/µL. Type 2 HIT usually presents 5-10 days after the initiation of
heparin therapy and may lead to life-threatening consequences (eg, limb ischemia, stroke).

(Choice A) Enoxaparin typically does not prolong the activated partial thromboplastin time (it
may in rare cases cause mild prolongation). However, the risk of HIT is lower with enoxaparin
than with unfractionated heparin therapy.

(Choices B and C) The risk of developing HIT is negligible with factor X-a inhibitors such as
fondaparinux and rivaroxaban.

(Choice E) Oral warfarin is typically co-administered with intravenous unfractionated heparin


(or low-molecular-weight heparin) in the initial treatment of venous thromboembolism. It is not
used alone due to initial hypercoagulable state from transient protein C depletion and risk of
thrombosis and skin necrosis. Warfarin typically causes prothrombin time prolongation
(normal in this patient) and does not affect platelet count.

Educational objective:
Heparin-induced thrombocytopenia should be suspected in patients receiving heparin
anticoagulation who have thrombocytopenia, thrombosis (arterial or venous), or a >50% drop in
the platelet count from baseline 5-10 days after the initiation of treatment.
*Polycythemia vera

Polycythemia vera
• ↑ Blood viscosity
o Hypertension
o Erythromelalgia (burning cyanosis in hands/feet)
o Transient visual disturbances
Manifestations
• ↑ RBC turnover (gouty arthritis)
• Aquagenic pruritus
• Bleeding

• Facial plethora (ruddy cyanosis)


Examination • Splenomegaly

• Elevated hemoglobin
• Leukocytosis & thrombocytosis
Laboratory • Low erythropoietin level
• JAK2 mutation positive
findings
• Thrombosis
Complications • Myelofibrosis & acute leukemia

• Phlebotomy
Treatment • Hydroxyurea (if ↑ risk of thrombus)

RBC = red blood cell.

Polycythemia vera (PV) is a clonal myeloproliferative disorder characterized by


erythrocytosis. Most cases are associated with a JAK2 mutation. Patients are often
asymptomatic at diagnosis but may present with symptoms related to increased blood viscosity
(eg, transient visual disturbances, hypertension, thrombosis) and increased red blood cell
(RBC) turnover (gouty arthritis). Itching after warm showers (aquagenic pruritus) is
common. Physical examination often shows facial plethora (ruddy cyanosis) and
splenomegaly. Unlike other causes of erythrocytosis (eg, chronic hypoxia, erythropoietin
[EPO]-producing tumors), patients with PV usually have normal oxygen saturations and low
EPO levels. Complete blood count often shows an increase in all 3 cell lines.

Serial phlebotomy is the mainstay of treatment for PV; it creates a relative iron deficiency and
reduces hematocrit levels to normal range. Bone marrow suppressive drugs (eg, hydroxyurea)
may be added in patients at high risk of thrombosis (eg, previous thrombosis, advanced age).

The terms polycythemia and erythrocytosis are often used interchangeably; however, not all
polycythemia is truly erythrocytosis. Polycythemia implies a laboratory finding of elevated
hematocrit and may be diagnosed as relative (usually due to a reduction in plasma volume) or
absolute (due to increased RBC production). Only absolute polycythemia involves
erythrocytosis (increased RBC mass). Absolute polycythemia/erythrocytosis is seen in patients
with PV and secondary polycythemia (eg, chronic hypoxia, EPO-secreting tumors).
*Thrombotic thrombocytopenic purpura (TTP)

Thrombotic thrombocytopenic purpura


• ↓ ADAMTS13 level → uncleaved vWF multimers → platelet
trapping & activation
Pathophysiology
• Acquired (autoantibody) or hereditary

• Hemolytic anemia (↑ LDH, ↓ haptoglobin) with schistocytes


• Thrombocytopenia (↑ bleeding time, normal PT/PTT)

Sometimes with:
Clinical features
• Renal failure
• Neurologic manifestations
• Fever

• Plasma exchange
• Glucocorticoids
Management • Rituximab
• Caplacizumab

LDH = lactate dehydrogenase; vWF = von Willebrand factor.

Thrombotic thrombocytopenic purpura (TTP) is marked by small vessel thrombi that consume platelets,
shear red blood cells, and cause end organ damage (primarily renal and CNS). This patient has several
manifestations of the classic TTP pentad, which includes:

• Thrombocytopenia.
• Microangiopathic hemolytic anemia (MAHA).
• Renal insufficiency.
• Neurologic changes (eg, acting "strangely").
• Fever (not always present).

TTP is usually due to an acquired autoantibody to ADAMTS13, a protease that cleaves von Willebrand factor
(vWF) off the endothelial surface. As ADAMTS13 levels fall (due to the antibody), vWF
multimer accumulation on the endothelial wall traps platelets at areas of high shearing force (eg, small
arterioles, capillaries), leading to thrombus formation.

Most cases arise in middle age and are idiopathic; however, chronic infections (eg, HIV), malignancy, or
certain medications can increase the risk for TTP. Diagnosis is based on clinical and laboratory
data. Thrombocytopenia (platelet count <30,000/mm3) and MAHA (eg, increased levels of indirect bilirubin,
aspartate aminotransferase [AST], alanine aminotransferase [ALT], lactate dehydrogenase, and reticulocytes)
should always raise suspicion. A peripheral blood smear showing intravascular hemolysis (eg, schistocytes,
helmet cells, triangle cells) supports the diagnosis

TTP must be treated urgently with plasma exchange. Patients should also receive high-dose glucocorticoids
and rituximab to suppress autoantibody production by B cells. Caplacizumab (monoclonal antibody that
blocks the vWF binding site for the ADAMTS13-autoantibody) is frequently added for severe disease.
*different blood smear patterns

This patient has single, round, blue inclusions within red blood cells (RBCs) on peripheral smear, consistent
with Howell-Jolly bodies. These bodies are retained RBC nuclear remnants that are typically removed by the
spleen. The presence of Howell-Jolly bodies usually indicates either physical absence of the spleen (ie,
asplenia) due to congenital absence or surgical removal or functional hyposplenism due to splenic
autoinfarction (eg, sickle cell disease), infiltrative disorders of the spleen (eg, sarcoidosis), or splenic congestion
(eg, thrombosis).

Both asplenia and functional hyposplenism increase the risk of sepsis because the spleen is normally
responsible for clearing bacteria, particularly encapsulated organisms. However, splenectomy may be indicated
in cases of chronic hemolysis.
(Choice A) In glucose-6-
phosphate dehydrogenase deficiency disease, hemoglobin becomes oxidized and precipitates into insoluble
Heinz bodies. Peripheral smear shows bite cells, and Heinz bodies appear as peripheral blue inclusions with
special staining (eg, crystal violet), not with Wright staining as in this patient.
(Choice B) The inability to produce alpha (alpha thalassemia) or beta (beta thalassemia) globin chains causes
hypochromic, microcytic RBCs and target cells (due to decreased hemoglobin production), which can be seen
on peripheral smear.
(Choice C) Mechanical RBC damage occurs when RBCs shear within the vascular system. This occurs in
hemolytic uremic syndrome, disseminated intravascular coagulation, and thrombotic thrombocytopenic
purpura. Schistocytes (fragmented RBCs) are seen on peripheral blood smear.
(Choice E) Precipitation of ribosomal ribonucleic acid is seen in patients with lead poisoning. In contrast to
the single, blue, peripherally-located inclusion characteristic of a Howell-Jolly body, basophilic stippling
appears as multiple, scattered, blue granules within the RBC cytoplasm.

Educational objective:
Howell-Jolly bodies are nuclear remnants within red blood cells typically removed by the spleen. Their
presence strongly suggests asplenia or functional hyposplenism
Choice A) Burr cells (also known as echinocytes) are spiculated appearing red blood cells (RBCs) with
serrated edges that can be seen in liver disease and end-stage renal disease (ESRD). However, burr cells are
less common in the acute kidney injury seen in SRC.
(Choice D) Spur
cells (acanthocytes) are RBCs with irregularly sized and spaced projections that are most commonly seen in
liver disease.
*Iron deficiency anemia in young children
Iron deficiency anemia in young children
• Prematurity
• Lead exposure
• Age <1
o Delayed introduction of solids (ie, exclusive

Risk factors breastfeeding after 6 months)


o Cow's, soy, or goat's milk
• Age >1
o >24 oz (>700 mL) per day of cow's milk
o <3 servings/day of iron-rich foods

• Screening hemoglobin at age 1


Diagnosis • Hemoglobin <11 g/dL (<6.83 mmol/L), ↓ MCV, ↑ RDW

• Empiric trial of iron supplementation


Treatment
MCV = mean corpuscular volume; RDW = red blood cell distribution width.

Iron deficiency is the most common nutritional deficiency in children. An infant's iron stores
are affected by the maternal iron stores, prenatal or perinatal hemorrhage, and the gestational
age of the infant at delivery; also, prematurity is a risk factor for iron deficiency. Term infants
usually have adequate iron stores for the first 6 months of life. After 6 months, inadequate
dietary intake becomes the most important cause of iron deficiency. Early introduction or
excessive intake of cow's milk is problematic as cow's milk has low iron content and
bioavailability and can cause occult intestinal blood loss in infants. Children should not be
started on cow's milk until age 1 year, and children age ≥1 year should consume <24
ounces/day (ie, consumption of >24 ounces [700 mL]/day is a risk factor for iron
deficiency). A typical glass is about 8 ounces.

Children with iron deficiency are often asymptomatic and may not have telltale symptoms such
as pica, fatigue, or pallor. Therefore, universal screening is recommended by age 1 year, and
the diagnosis is typically based on a complete blood count, typically showing a low
hemoglobin, low mean corpuscular volume, and elevated red blood cell distribution width. The
reticulocyte count is classically low (because the bone marrow with iron deficiency cannot
increase reticulocyte production), although a low (<2%) reticulocyte index (reticulocyte count ×
hematocrit / normal hematocrit) is a better indicator of hypoproliferative anemia. A peripheral
smear would be expected to show microcytic hypochromic erythrocytes, although it is usually
not needed in the initial workup. Further testing (eg, iron studies) is not necessary in children
with the classic presentation. The most cost-effective approach to treatment is empiric oral
iron therapy. Hemoglobin should be rechecked in 4 weeks; if the hemoglobin level has risen 1
g/dL, the oral iron therapy should be continued for 2-3 months after the hemoglobin normalizes
to replete iron stores.
(Choice A) Blood transfusions are rarely necessary in the treatment of dietary iron deficiency
anemia, even when the hemoglobin is as low as 4 g/dL. Iron therapy should be sufficient.

(Choices B, C, and E) If the anemia does not resolve with empiric iron therapy and
appropriate dietary changes, other causes must be considered. Hemoglobin electrophoresis (for
hereditary anemias), colonoscopy (for lower gastrointestinal tract bleeding), or serum creatinine
measurement (for renal disease) may be indicated.

Educational objective:
Iron deficiency anemia is common in infants and toddlers who drink excessive amounts of
cow's milk. In addition to a decreased hemoglobin level, a low mean corpuscular volume and
red blood cell count are also seen. Treatment consists of empiric oral iron therapy.
*PE common sources
This patient's clinical presentation (eg, acute chest pain worse with inspiration, tachycardia, S2
suggestive of pulmonary hypertension) with CT angiogram findings of a thrombus in the right
pulmonary artery is diagnostic of pulmonary embolus (PE). The deep veins of the lower
extremities are the most frequent source. Lower extremity deep vein thrombosis (DVT) is
divided into 2 categories:

• Proximal/thigh (eg, iliac, femoral, popliteal): These DVTs are the source of >90% of
acute PEs, probably due to their large caliber and proximity to the lungs.
• Distal/calf: Although most DVTs occur in the calf vein, distal/calf DVTs are less likely
than proximal/thigh DVTs to embolize to the lungs and cause PE and they are more
likely to spontaneously resolve (Choice A).

(Choice C) Thrombi originating from the renal veins are most common in patients with
nephrotic syndrome. The prevalence of PE in patients with nephrotic syndrome is 10%-30%.

(Choice D) Thrombi originating in the right atrium are more common in patients with
intracardiac devices (eg, pacemaker) and are less likely to occur spontaneously.

(Choice E) Spontaneous upper extremity DVT is generally uncommon (1%-4%) and even less
likely to cause PE. Common risk factors for upper extremity DVT include prothrombotic state,
indwelling catheter, and anatomical venous obstruction (eg, cervical rib, bony overgrowth after
clavicular fracture).

Educational objective:
The proximal deep leg veins are the most common source of symptomatic pulmonary
embolism. Other less common sources of emboli include the calf veins, renal veins, pelvic
veins, upper extremity veins, and right heart.
*prevention of Rh D alloimmunization
This woman's O-negative blood type indicates that she is Rh negative, and the anti-D antibody titer of 1:32
reflects that she is alloimmunized (ie, sensitized). Alloimmunization occurs when the mother is Rh negative
and has an Rh-positive fetus. Her first pregnancy, especially the placental abruption, put her at risk for
alloimmunization. To prevent the maternal immune system from developing anti-D antibodies, anti-D immune
globulin is first administered at 28 weeks gestation and repeated within 72 hours of delivery.

A standard dose of 300 µg at 28 weeks gestation can usually prevent alloimmunization. However, ~50% of Rh-
negative women will need a higher dose after delivery, placental abruption, or procedures. The Kleihauer-
Betke (KB) test is commonly used to determine the dose. Red blood cells from the maternal circulation are
fixed on a slide. The slide is exposed to an acidic solution and adult hemoglobin lyses, leaving "ghost"
cells. The dose of anti-D immune globulin is calculated from the percentage of remaining fetal hemoglobin.

(Choice B) The standard dose of anti-D immune globulin at 28 weeks gestation in an uncomplicated pregnancy
is usually adequate as the risk of alloimmunization is very low before this time. However, if the patient
developed placental abruption earlier in pregnancy, the KB test should be performed to determine whether a
higher dose is indicated. Other indications for anti-D immune globulin include amniocentesis, chorionic villus
sampling, and external cephalic version.

(Choice C) Prophylaxis between pregnancies is rarely necessary. It is possible but uncommon for sensitization
to occur from inadvertent intravenous transfusion of Rh-positive blood due to routine blood bank cross-
matching. Sensitization could also occur during an unrecognized spontaneous abortion of an Rh-positive fetus,
but most women would experience bleeding.

(Choice D) Delayed administration of anti-D immune globulin after 72 hours postpartum may allow
sensitization to occur. However, the timing of this patient's anti-D immune globulin within 72 hours after
delivery is appropriate.

(Choice E) The patient's antibody titer reflects Rh alloimmunization that developed from the first
pregnancy. If an undisclosed new partner is Rh positive, the fetus may also be Rh positive and at risk for
hemolytic disease of the newborn. However, if an undisclosed new partner is Rh negative, the fetus will not be
at risk for hemolysis.

Educational objective:
Anti-D immune globulin should be administered to any Rh D-negative mother who delivers an Rh D-positive
baby. The standard dose is usually adequate at 28 weeks gestation. After delivery or procedures, the
Kleihauer-Betke test is used to determine whether a higher dose is needed due to the increased risk of fetal
blood cells entering the maternal circulation.
*Iron deficiency anemia & thalassemias

Iron deficiency anemia & thalassemias


Iron deficiency α-Thalassemia β-Thalassemia
Parameter
anemia minor minor
MCV ↓ ↓ ↓
RDW ↑ Normal Normal
RBCs ↓ Normal/↑ Normal/↑
Peripheral Microcytosis,
Target cells Target cells
smear hypochromia
Serum iron ↓ Iron & ferritin Normal/↑ iron & ferritin Normal/↑ iron & ferritin
studies ↑ TIBC (RBC turnover) (RBC turnover)
Response to iron
↑ Hemoglobin No improvement No improvement
supplementation
Hemoglobin
Normal Normal ↑ Hemoglobin A2
electrophoresis
MCV = mean corpuscular volume; RBCs = red blood cells; RDW = red blood cell distribution
width; TIBC = total iron-binding capacity.

Iron deficiency and thalassemia are the most common causes of microcytic anemia (mean
corpuscular volume <80 µm3). Both present with fatigue and pallor; however, a history of
prolonged bleeding (eg, heavy menses) or poor dietary iron intake raises suspicion for iron
deficiency anemia (IDA).

Distinguishing laboratory findings of IDA include decreased erythrocyte count (inadequate


synthesis) and elevated red cell distribution width due to variable iron levels available
throughout the day. In addition, IDA may be associated with reactive thrombocytosis
(platelets >400,000/mm3) in response to low red blood cell count. This change is due to
megakaryocytes and erythrocytes sharing a common progenitor cell.

Although thalassemia should be on the differential diagnosis for a patient of Greek ethnicity
with a family history concerning for an inherited thalassemia, this patient also has signs of IDA
(heavy menses, decreased erythrocytes, thrombocytosis). Therefore, iron studies are the best
next step. Correction of IDA is necessary prior to evaluation of thalassemia with hemoglobin
electrophoresis because concomitant iron deficiency can lower hemoglobin A2 and mask the
diagnosis (Choice B).

(Choice A) Bone marrow biopsy should be considered in patients with more than 2 abnormal
cell lines (eg, anemia, leukopenia, thrombocytopenia) to evaluate for acute leukemia or aplastic
anemia. Because this patient has isolated anemia and related symptoms, bone marrow biopsy is
not yet indicated. Iron studies are also a noninvasive first step in contrast to bone marrow
biopsy.
(Choice D) Lead poisoning can cause microcytic anemia in children with risk factors for lead
exposure (eg, housing built before 1978, eating dirt/paint chips). In contrast, this patient's home
was built in the 1980s.

(Choice E) Vitamin B12 deficiency is a macrocytic anemia more prevalent in patients with a
vegan diet.

Educational objective:
Iron studies are performed prior to hemoglobin electrophoresis in patients with microcytic
anemia suspicious for iron deficiency (eg, heavy menses, low erythrocyte count, reactive
thrombocytosis) and concomitant thalassemia (eg, Greek origin, family history).

Laboratory findings in thalassemia minor typically demonstrate mild hematocrit reduction


(>28%), significantly low MCV (55-75 µm3), and disproportionally elevated red blood cell
(RBC) counts. The disproportionately elevated RBC count (eg, normal or high rather than
low) and decreased MCV help distinguish thalassemia minor from iron deficiency anemia,
which does not typically cause microcytosis until hemoglobin is <10 g/dL (Choice F). Patients
with thalassemia minor show no improvement in hemoglobin with iron supplementation and do
not require specific treatment.
*Hereditary spherocytosis
Hereditary spherocytosis
• Usually autosomal dominant
Epidemiology
• Hemolytic anemia
• Jaundice
Clinical presentation
• Splenomegaly

• ↑ MCHC
• Negative Coombs test
• Spherocytes on peripheral smear
Laboratory findings
• ↑ Osmotic fragility on acidified glycerol lysis test
• Abnormal eosin-5-maleimide binding test

• Folic acid supplementation


• Blood transfusion
Treatment
• Splenectomy

MCHC = mean corpuscular hemoglobin concentration.

This infant has anemia, jaundice, and splenomegaly, concerning for hereditary spherocytosis,
a common cause of hemolytic anemia in persons of Northern European descent. It is typically
an autosomal dominant disorder caused by a defect of red blood cell membrane proteins (most
commonly ankyrin), resulting in unstable, round (rather than biconcave) red blood cells.

These fragile spherocytes cannot traverse through tight spaces in the microcirculation, such as
the spleen, which leads to the classic triad of hemolytic anemia, jaundice, and
splenomegaly. Patients can have symptoms at any time of life, including the newborn
period. Specifically, neonates may present with jaundice on the first day of life and persistent
jaundice refractory to standard management (eg, phototherapy).

Laboratory findings in spherocytosis include anemia, increased mean corpuscular


hemoglobin concentration (MCHC), indirect hyperbilirubinemia, and negative Coombs
test. The increased MCHC is due to membrane loss and cellular dehydration. Reticulocytes are
elevated to compensate for hemolysis, and spherocytes are usually present on peripheral
smear. The diagnosis is confirmed by abnormal acidified glycerol lysis (osmotic fragility) and
eosin-5-maleimide binding tests.
*metastatic prostate cancer bone lesions & it's DDx
This elderly man with several months of progressive back pain has focal lumbar tenderness,
increased alkaline phosphatase, and L1/L2 sclerotic lesions suggesting osteoblastic bone disease
due to metastatic prostate cancer.

Prostate adenocarcinoma is the most common cancer in men and often presents with
manifestations of metastatic disease. Symptoms usually arise after spread to the axial skeleton
(vertebral bodies, ribs) with resultant progressive lower back pain or acute functional issues
(eg, motor weakness, incontinence) from pathologic fracture or spinal cord impingement.

Unlike many other forms of metastatic cancer to the spine, prostate cancer usually causes pure
osteoblastic lesions. This leads to normal or low serum calcium (unlike osteolytic disease),
elevations in alkaline phosphatase, and imaging evidence of focal, sclerotic bone
lesions. Further evaluation with a radionuclide bone scan and prostate-specific antigen test is
required. A prostate biopsy is likely needed.

(Choice A) Colorectal cancer often causes abdominal symptoms (eg, pain, change in bowel
habits, bleeding) and usually spreads to the liver and lungs. Osteoblastic bone lesions would be
atypical.

(Choice C) Multiple myeloma is a plasma cell neoplasm that often presents with bone pain due
to lytic lesions (not blastic lesions), hypercalcemia (not normal calcium), anemia, and renal
disease.

(Choice D) Paget disease is a common disorder of bone remodeling that is often asymptomatic
and diagnosed after incidentally discovering an elevated alkaline phosphatase. Radiographs
typically reveal osteolytic lesions or mixed osteolytic/osteoblastic lesions (not osteoblastic
alone).

(Choice E) Chronic kidney disease often causes hyperphosphatemia (as in this patient),
hypocalcemia, and secondary hyperparathyroidism and is frequently associated with renal
osteodystrophy. Manifestations include increased bone turnover, with radiographic evidence of
widespread osteopenia and subperiosteal bone reabsorption (not focal, sclerotic lesions).

(Choice F) Vertebral osteomyelitis often causes slowly progressive back pain; however,
imaging typically reveals vertebral body destruction and collapse of the disc space (not sclerotic
lesions).

Educational objective:
Prostate cancer may initially present with symptoms related to metastatic disease. Spread to the
axial skeleton is common, and manifestations typically include slowly progressive lower back
pain or functional impairment (eg, motor weakness, incontinence). Prostate cancer usually
causes osteoblastic bone lesions with normal or low calcium, elevated alkaline phosphatase, and
radiographic evidence of focal, sclerotic lesions.
*Von Willebrand disease (vWD)

Von Willebrand disease (vWD) is the most common inherited bleeding disorder and is typically transmitted in
an autosomal dominant fashion. Therefore, a family history of prolonged bleeding is an important diagnostic
clue. Most patients are asymptomatic, but a minority (~1%) have easy bruising, skin bleeding, and mucosal
bleeding (eg, menstruation), which can occasionally result in iron deficiency (microcytic anemia) due to chronic
blood loss.

VWD is associated with impaired quantity/activity of von Willebrand factor (vWF), a glycoprotein produced
by endothelial cells and platelets. VWF contributes to hemostasis by:

1. Binding platelets to subendothelial components at the site of vascular injury and aggregating platelets to
each other, leading to the formation of a platelet plug. Therefore, patients with vWD have impaired
platelet-endothelial binding and platelet aggregation, which prolongs bleeding times.
2. Acting as a carrier protein for factor VIII, which is part of the intrinsic (contact activation) coagulation
cascade and contributes to fibrin clot formation. Because factor VIII degrades rapidly when not bound
to vWF, patients with vWD generally have low factor VIII levels, which may prolong activated
PTT. In contrast, PT is normal because the extrinsic coagulation pathway is unaffected.
*pernicious anemia

This patient's symptomatic anemia (light-headedness, palpitations) with macrocytosis (eg, mean
corpuscular volume [MCV] >100 µm3), glossitis (smooth tongue), and lower extremity
neurologic findings (paresthesia, diminished reflexes) suggests vitamin
B12 deficiency. Vitamin B12 absorption relies on the salivary glands (R-protein), stomach
(intrinsic factor, gastric acid), pancreas (proteases), and ileum (absorption). Although damage
to any of these structures can impair uptake, the most common cause of vitamin B12 deficiency
is pernicious anemia.
Pernicious anemia is marked by antibody-mediated destruction of intrinsic factor, a protein
carrier produced by gastric parietal cells and required for vitamin B12 absorption. Antibodies
also directly target gastric parietal cells, leading to atrophy of the acid/pepsin-producing parts of
the stomach. Over time, the atrophic gastric mucosa undergoes intestinal-type metaplasia,
which dramatically increases the risk of gastric cancer. Therefore, patients with pernicious
anemia require esophagogastroduodenoscopy at initial diagnosis and with any suspicious
symptoms (eg, positive fecal occult blood test, mid-epigastric abdominal pain).
*Hodgkin lymphoma

Hodgkin lymphoma
• Bimodal peak incidence: age 15-35 & >60
Epidemiology • Association with EBV in immunosuppression

• Painless lymphadenopathy
• Mediastinal mass
Manifestations • B symptoms (ie, fever, sweats, weight loss)
• Pruritus

• Lymph node biopsy


Diagnosis • Reed-Sternberg cells on histology

EBV = Epstein-Barr virus.


This young patient's subacute fever, pruritus, fatigue, and painless cervical and supraclavicular
lymphadenopathy (LAD) raise strong suspicion for Hodgkin lymphoma (HL). Cases generally
arise in a bimodal distribution, with peak incidence in patients age 15-35 and >60.

HL usually arises in a single lymph node and disseminates to contiguous nodes via the
lymphatic and thoracic ducts. Patients generally present with painless peripheral LAD,
particularly in the cervical and supraclavicular chains. Significant mediastinal LAD also
occurs in ~50% of cases and may lead to cough, shortness of breath, or chest pain. Most
patients have long-standing B symptoms (eg, fever, night sweats, weight loss) and pruritus (a
paraneoplastic process classically associated with HL and likely due to cytokine and interleukin
release).

Individuals with unexplained LAD and findings concerning for malignancy (eg, supraclavicular
LAD) require excisional lymph node biopsy. Malignant Reed-Sternberg cells admixed with
nonneoplastic inflammatory cells are diagnostic of HL.
*Fanconi anemia

Fanconi anemia
• Inherited DNA repair defect
Pathophysiology • Bone marrow failure

• Short stature
• Hypo-/hyperpigmented macules
Clinical findings • Abnormal thumbs
• Genitourinary malformations

• Pancytopenia
Laboratory findings • Positive chromosomal breakage testing

• Hematopoietic stem cell transplant


Treatment

This patient most likely has Fanconi anemia (FA), the most common congenital cause of
aplastic anemia (ie, bone marrow failure, pancytopenia). FA is typically an autosomal
recessive disorder caused by a DNA repair defect. Damaged, unstable DNA impairs normal
hematopoietic stem cell production and can also lead to increased susceptibility to malignancy
(eg, leukemia).

Patients usually present in childhood with signs of thrombocytopenia (eg, bleeding, bruising),
as in this case, with progression to leukopenia (eg, infections) and anemia (eg, fatigue) over
time. Anemia is usually macrocytic due to fetal erythropoiesis that occurs during periods of
chronic hematopoietic stress. Additional characteristic features of FA include short stature,
hyper- or hypopigmentation, and absent or hypoplastic thumbs, as seen in this
patient. Additional abnormalities of the hand may include polydactyly or a flat thenar
eminence.

Diagnosis of FA is made by demonstrating chromosomal breakage following DNA exposure to


interstrand crosslinking agents. Definitive treatment is hematopoietic stem cell transplantation.
*superior vena cava (SVC) syndrome
This patient's signs and symptoms are consistent with superior vena cava (SVC) syndrome, a
condition where obstruction of the SVC impedes venous return from the head, neck and arms to
the heart. Signs and symptoms include dyspnea, venous congestion, and swelling of the head,
neck and arms. Malignancy is the most common cause of obstruction (i.e. lung cancer, non-
Hodgkin lymphoma), accounting for >60% of cases. This patient's 40 year smoking history and
recent 10 pound weight loss make lung cancer a likely diagnosis. Chest x-ray (Choice C) can
identify the cause of SVC syndrome in >80% of cases. Abnormalities on chest x-ray warrant
follow-up with chest CT and histology to determine the tumor type and to guide therapy.

(Choice A) Echocardiogram is not part of the work-up for SVC syndrome. Echocardiography
can be helpful in suspected cardiac tamponade, which presents with Beck's triad of jugular
venous distention, distant heart sounds and hypotension. Tamponade does not cause arm or
face swelling or engorgement of the anterior chest wall veins.

(Choice B) Unexplained unilateral arm swelling warrants a Doppler to rule out


DVT. However, simultaneous swelling of the face and bilateral arms indicates venous
obstruction in the SVC, which upper extremity Doppler would not demonstrate.

(Choices D & E) Measurement of the 24-hour urinary protein excretion can help assess for
nephrotic syndrome, and measurement of the serum albumin level can help assess for liver
disease. These conditions can cause generalized edema, but not edema isolated to the upper
half of the body.

Educational objective:
Malignancy is the most common cause of SVC syndrome. Lung cancer (particularly small cell
lung cancer) and NHL are often implicated. Other possible causes include fibrosing
mediastinitis (secondary to histoplasmosis or Tb infection) or thrombosis secondary to
indwelling central venous devices. When the history and physical examination are suggestive,
chest x-ray is warranted.
*anemia of chronic disease (ACD) ttt
This patient has inflammatory arthritis, which is most consistent with early rheumatoid
arthritis. She has moderate anemia with low serum iron and total iron-binding capacity (TIBC)
consistent with anemia of chronic disease (ACD). Other characteristic laboratory findings of
ACD include normochromic normocytic red blood cells, mildly decreased transferrin saturation,
and normal to elevated ferritin. In contrast, iron-deficiency anemia features elevated TIBC and
low ferritin. Erythropoietin levels are variable, but ACD patients usually have a low
reticulocyte count relative to anemia severity, suggesting impaired red blood cell
production. ACD pathophysiology is thought to involve iron trapping within macrophages,
leading to reduced serum iron concentrations and poor iron availability for hemoglobin
synthesis. Decreased erythropoietin production and poor marrow response to erythropoietin
may also play a role.

ACD is commonly associated with chronic inflammatory diseases (eg, infections, cancer,
autoimmune disorders) but can also be observed in heart disease, diabetes mellitus, and acute
inflammation. Treating the underlying inflammatory disorder will often improve the
anemia. Common agents used in rheumatoid arthritis treatment include methotrexate,
hydroxychloroquine, and tumor necrosis factor inhibitors (eg, infliximab,
etanercept). Erythropoietin or darbepoetin treatment may benefit ACD patients unresponsive to
underlying inflammatory disorder treatment (Choice B). Packed red blood cell transfusions are
rarely required but can be considered after all other treatment interventions have failed (Choice
F).

(Choice D) Serum iron concentrations are low in ACD patients. However, iron
supplementation is not beneficial due to impaired iron utilization in ACD.

(Choices A and C) B-complex vitamins (pyridoxine, folic acid, cyanocobalamin, nicotinic


acid) are not deficient in ACD; supplementation will not improve the anemia of ACD.

(Choice G) Hereditary spherocytosis is treated via splenectomy; this can be occasionally used
for treating immune thrombocytopenic purpura and chronic idiopathic myelofibrosis with
refractory anemia. However, splenectomy will not improve this patient's anemia.

Educational objective:
Anemia of chronic disease is a disorder of iron utilization that most commonly occurs in the
setting of chronic inflammation. It is characterized by a normocytic anemia with decreased
serum iron, decreased total iron-binding capacity, decreased iron saturation, and
normal/elevated serum ferritin. Treating the underlying inflammatory disorder will often
improve the anemia.
*Anemia & Pregnancy
During pregnancy, a mild physiologic anemia occurs due to hemodilution (increased plasma
volume), but mean corpuscular volume (MCV) does not typically change. Therefore, pregnant
patients with normocytic, mild anemia do not require additional evaluation. In contrast,
microcytic and macrocytic anemias indicate pathology.

This patient at 9 weeks gestation has microcytic anemia (eg, MCV <80 µm3). Serum iron
studies (eg, iron, ferritin, total iron binding capacity [TIBC]) can help determine the underlying
etiology:

• Iron deficiency is most common cause of microcytic anemia in pregnancy due to


increased demand for red blood cell production. This patient's normal serum iron level
and normal TIBC make iron deficiency unlikely.
• Anemia of chronic disease is associated with high ferritin (an inflammatory marker) and
low iron.
• Lead toxicity inhibits heme synthesis and causes sideroblastic anemia. Lead toxicity is
associated with high serum iron and ferritin because lead inhibits ferrochelatase, causing
iron to accumulate; in contrast, this patient's serum iron and ferritin are normal (Choice
C).
• Hemoglobinopathy causes anemia due to either abnormal globin production (eg, alpha- or
beta-thalassemia) or hemoglobin structure (eg, Hgb S, C, E).

In patients with microcytic anemia but normal serum iron studies,as seen in this patient,
hemoglobinopathy is most likely. The best next step is hemoglobin electrophoresis, which
would confirm the diagnosis and potentially change management. Hemoglobinopathy traits are
inherited in an autosomal recessive manner. Therefore, if the mother is identified as a carrier,
the father is also tested; and if both parents are carriers, genetic counseling (regarding
implications for the offspring) and fetal testing (eg, chorionic villus sampling) are offered.

(Choices A and E) Folate and vitamin B12 requirements increase in pregnancy, and deficiency
of either causes anemia. Folate deficiency is common with malnutrition or antiepileptic drug
use; vitamin B12 deficiency may be seen with poor gastrointestinal absorption (eg, gastric
bypass) or inadequate dietary intake of vitamin B12–rich animal products (eg,
vegetarianism). However, folate and vitamin B12 deficiencies cause macrocytic (MCV ≥100
µm3) anemia.

(Choice D) Hypothyroidism typically causes anemia via decreased red blood cell mass with a
normocytic MCV (80-100 µm3) or in association with pernicious anemia (ie, macrocytic). In
addition, TSH screening is not recommended for asymptomatic pregnant patients.

Educational objective:
Pregnant patients with microcytic anemia and normal iron studies require evaluation for
hemoglobinopathy with hemoglobin electrophoresis.
*Acute myeloid leukemia

Acute myeloid leukemia


• Most common adult acute leukemia
Background • Median age 65

• Fatigue is common (other B symptoms unusual)


• Often presents with symptoms from cytopenias:
o Fatigue, weakness (anemia)
o Bleeding, bruising (thrombocytopenia)
Manifestations
o Infection (granulocytopenia)
• Hepatosplenomegaly/lymphadenopathy rare
• Disseminated intravascular coagulation (if APML)

• Cytopenias (leukocytes may be ↑, normal or ↓)


• Elevated lactate dehydrogenase
Laboratory
• Peripheral smear - usually myeloblasts with Auer rods

• Bone marrow biopsy - usually hypercellular with myeloid blasts


Diagnosis
APML = acute promyelocytic leukemia.

This patient with fatigue, prolonged bleeding, and easy bruising has pancytopenia,
coagulopathy, and elevated lactate dehydrogenase, strongly suggesting acute promyelocytic
leukemia (APML), a form of acute myeloid leukemia (AML).

AML is a clonal myeloproliferative disorder characterized by the accumulation of blastic or


immature myeloid cells in the bone marrow and peripheral blood. Patients often develop
fatigue and symptoms related to >1 cytopenias, including fatigue/weakness (anemia),
bleeding/bruising (thrombocytopenia), and/or infection (granulocytopenia). Examination may
show pallor and ecchymosis, but lymphadenopathy and hepatosplenomegaly are
rare. Leukocyte count may be elevated (sometimes >100,000/mm3), normal, or low.

One unique type of AML is APML, which is characterized by life-threatening coagulopathy due
to disseminated intravascular coagulation (prolonged PT/active PTT,
hypofibrinogenemia). In APML, bone marrow biopsy would reveal atypical promyelocytes
*Diamond-Blackfan anemia

Diamond-Blackfan anemia
• Congenital erythroid aplasia
Pathogenesis
• Craniofacial abnormalities
• Triphalangeal thumbs
Clinical findings
• Increased risk of malignancy

• Macrocytic anemia
• Reticulocytopenia
Laboratory findings
• Normal platelets, white blood cells

• Corticosteroids
Treatment • Red blood cell transfusions

This infant has macrocytic anemia, craniofacial anomalies, and triphalangeal thumbs consistent with
Diamond-Blackfan anemia (DBA). DBA is caused by a congenital (inherited or sporadic) defect of erythroid
progenitor cells, which leads to increased apoptosis of red blood cells, resulting in profound anemia. Most
patients have other anomalies, such as short stature, cleft palate, and webbed neck.

DBA typically presents in infancy with progressive pallor and poor feeding due to anemia, as seen in this
patient. The heart rate increases to meet oxygen demands, and a faint systolic ejection (flow) murmur may be
heard on examination due to increased turbulence across the valves.

Laboratory testing differentiates the type of anemia. In contrast to Fanconi anemia (pancytopenic bone marrow
failure), DBA is a pure red cell aplasia with reticulocytopenia and normal platelet and white blood cell counts
(Choice B). Treatment mainly involves corticosteroids or transfusion therapy in refractory cases.

(Choice C) Idiopathic aplastic anemia is an acquired disease that results in pancytopenia, not pure red cell
aplasia. Common triggers include chemicals (eg, benzene, phenylbutazone), drugs (eg, chloramphenicol,
sulfonamides), infectious agents (eg, viral hepatitis), and ionizing radiation.

(Choice D) Although iron deficiency is a common cause of microcytic anemia in infancy, it is unlikely in this
patient with an elevated mean corpuscular volume.

(Choice E) Hemolytic disease of the newborn due to Rh incompatibility occurs when an Rh-negative mother
previously exposed to Rh-positive cells gives birth to an Rh-positive infant. The mother of this infant is Rh-
positive (blood type O+) and the patient is Rh-negative (blood type A−); therefore, there is no risk of Rh
incompatibility for the patient.

(Choice F) Wiskott-Aldrich syndrome is an X-linked disorder characterized by eczema, thrombocytopenia, and


hypogammaglobulinemia, findings not present in this patient.

Educational objective:
Diamond-Blackfan anemia is a congenital, pure red cell aplasia characterized by macrocytic anemia and several
congenital abnormalities, such as cleft palate, webbed neck, and triphalangeal thumbs.
*Drug-induced, immune-mediated hemolytic anemia

Drug-induced, immune-mediated hemolytic anemia


• Drug coats erythrocytes → IgG binding → splenic destruction of RBCs
(extravascular hemolysis)
Pathogenesis • Drug triggers immune complexes → complement-mediated destruction
of RBCs (intravascular hemolysis)

• Sudden onset (within hours of exposure)


• Anemia: fatigue, pallor, dyspnea
• Hemolysis: jaundice, dark urine, abdominal or back pain
o ↑ Reticulocytes, indirect bilirubin & LDH
Manifestations
o ↓ Haptoglobin
o Spherocytes on peripheral blood smear
o + Direct Coombs test (anti-IgG, anti-C3)

• Discontinue offending drug


• Transfusion (if severe)
Treatment
• ± Glucocorticoids, IVIG

IVIG = intravenous immunoglobulin; LDH = lactate dehydrogenase; RBCs = red blood cells.

This patient received amoxicillin and subsequently developed symptomatic anemia (eg, fatigue,
exertional dyspnea), splenomegaly, and jaundice, raising strong suspicion for drug-induced
hemolytic anemia. Penicillins (eg, amoxicillin) and cephalosporins are common triggers of
drug-induced hemolytic anemia because these medications bind to the surface of erythrocytes,
creating a hapten for IgG attachment. IgG-coated erythrocytes are then partially or wholly
phagocytized by splenic macrophages, leading to extravascular hemolysis. Complement-
mediated intravascular hemolysis may also occur.

Patients generally show signs of anemia (eg, fatigue, pallor, dyspnea) and hemolysis (eg,
jaundice, dark urine) within 2-4 days of medication exposure. Laboratory assessment typically
reveals indirect hyperbilirubinemia, elevated lactate dehydrogenase, decreased haptoglobin, and
reticulocytosis (due to the generation of new erythrocytes in response to anemia). Most
patients have normocytic anemia with a normal or slightly elevated mean corpuscular volume
due to the presence of reticulocytes. Splenomegaly can also occur due to increased red blood
cell sequestration in the spleen.

The diagnosis is made with direct antiglobulin (Coombs) testing, which identifies IgG or C3
(a complement fragment) attached to the erythrocyte surface. Because penicillin- and
cephalosporin-induced hemolysis is primarily triggered by drug binding to the erythrocyte
surface, withdrawal of the medication generally leads to resolution. Therefore, amoxicillin
should be discontinued in this patient; a different class of antibiotics could be started if
needed.

(Choices B, C, and E) Investigation for other causes of hemolytic anemia is not generally
necessary when there is a clear inciting factor such as recent administration of a medication
known to trigger hemolysis (eg, amoxicillin). However, in patients who do not have resolution
of hemolysis after medication cessation, additional workup may be required; this often includes
antinuclear antibody testing for systemic lupus erythematosus, bone marrow biopsy for
leukemia, and/or liver function testing for viral hepatitis.

(Choice D) Hemoglobin electrophoresis can diagnose sickle cell disease, which causes
hemolytic anemia and splenomegaly. However, most cases present in early childhood with
vasoocclusive pain crises. This patient with no chronic medical history who developed
hemolysis after taking amoxicillin is much more likely to have drug-induced hemolysis.

(Choice F) Splenectomy can improve platelet count in recalcitrant idiopathic


thrombocytopenia. Splenectomy is not advised for drug-induced hemolysis because the
hemolysis typically resolves rapidly with drug cessation.

Educational objective:
Drug-induced hemolytic anemia is often triggered by penicillin and cephalosporin
medications. It is marked by evidence of extravascular hemolytic anemia such as fatigue,
weakness, splenomegaly, dark urine, and jaundice. Discontinuation of the offending drug
usually results in complete resolution of symptoms within days. No additional workup is
generally required.
*Vitamin K deficiency

Vitamin K deficiency
• Inadequate dietary intake (eg, malnutrition)
• Disorders of fat malabsorption
o Cystic fibrosis
o Biliary atresia
Risk
factors • Disorders of intestinal inflammation
o Celiac disease
o Inflammatory bowel disease
• Decreased production by bacterial flora (eg, frequent antibiotic use)

• Easy bruising
Clinical • Mucosal bleeding
features • Gastrointestinal bleeding

Laboratory • ↑ PT & INR


findings • If severely deficient, ↑ PTT

This patient with cystic fibrosis (CF) has bruising and epistaxis likely due to deficiency of
vitamin K, a fat-soluble vitamin. Patients with CF are at increased risk for vitamin K
deficiency because defective chloride transport causes viscous exocrine secretions to block
pancreatic ducts, leading to pancreatic insufficiency. Impaired secretion of pancreatic
enzymes (eg, lipase) results in the malabsorption of fats and fat-soluble vitamins (ie, A, D, E,
and K). In patients with CF, nonadherence to pancreatic enzyme replacement and fat-soluble
vitamin supplementation, as seen here, increases the risk of bleeding diatheses.

Because vitamin K normally acts as an important cofactor in the hepatic activation of


coagulation factors II, VII, IX, and X, deficiency results in low levels of activated vitamin K–
dependent coagulation factors. PT is prolonged due to deficiency of activated factor VII;
activated PTT is typically normal but can be prolonged with severe deficiency.

Symptoms include easy bruising and mucosal bleeding (eg, epistaxis), as seen in this
patient. Acute bleeding due to vitamin K deficiency is treated with vitamin K replacement.
*acute vs chronic DIC

Acute vs chronic disseminated intravascular coagulation (DIC)


Acute DIC Chronic DIC
Sepsis

Common etiologies Severe trauma Malignancy (eg, pancreatic)

Obstetric complications
Coagulation studies Prolonged Often normal
Platelets Low Often normal
Fibrinogen Low Often normal
D-dimer High High
Bleeding risk Very high Mildly increased
Thromboembolism risk Mildly increased Very high

In this patient with asymmetric leg swelling suggestive of deep venous thrombosis (DVT),
several features are concerning for chronic disseminated intravascular coagulation (DIC) in
the setting of cancer. His long history of smoking combined with unintentional weight loss and
nagging epigastric and back pain raises suspicion for pancreatic cancer. Mucin-producing
tumors (eg, pancreatic, gastric, ovarian, breast) often secrete tissue factor into the blood, which
intermittently triggers the coagulation cascade and can lead to chronic DIC, a consumptive
coagulopathy.

As with other consumptive coagulopathies, the risk of arterial or venous thrombosis (eg, DVT)
is significantly increased in patients with chronic DIC; in addition, periodic mucocutaneous
bleeding (eg, gums, nose) can occur, as seen in this patient. However, because patients with
chronic DIC are usually able to compensate for intermittent coagulation and fibrinolysis via
increased production of coagulation factors, platelets, and fibrinogen, they may be
asymptomatic and frequently have normal platelet counts and coagulation times.

In contrast to chronic DIC, acute DIC usually arises in the setting of sustained procoagulant
exposure (eg, lipopolysaccharide in sepsis) and is marked by massive activation of the
coagulation cascade, with rapid consumption of platelets, coagulation factors, and
fibrinogen. Most cases present with acute bleeding rather than thrombosis.
*transfusion-related acute lung injury (TRALI)

Transfusion reactions associated with hypotension


Reaction Onset* Cause Clinical features
• Shock,
Seconds to Recipient anti-IgA angioedema/urticaria &
Anaphylaxis
minutes antibodies respiratory distress

• Respiratory distress &


Transfusion- Donor noncardiogenic
Minutes to pulmonary edema
related acute lung antileukocyte
hours • Bilateral pulmonary
injury antibodies
infiltrates

• Fever, flank pain,


Minutes to ABO
Acute hemolysis hemoglobinuria & DIC
hours incompatibility
Bacterial • Fever, chills, septic
Minutes to
Bacterial sepsis contamination of shock & DIC
hours
donor product
*Time after transfusion initiation.

DIC = disseminated intravascular coagulation.

This patient developed respiratory distress, hypotension, and pulmonary infiltrates during blood
transfusion, findings concerning for transfusion-related acute lung injury (TRALI). TRALI
is an uncommon but potentially fatal transfusion reaction in which neutrophils activated by a
component in the transfused blood (eg, anti-leukocyte antibodies) cause damage to pulmonary
microvasculature. Risk factors for TRALI include smoking, alcohol use disorder, and critical
illness.

Damage to the capillary endothelium results in inflammatory pulmonary edema. Hypoxia


and dyspnea develop within 6 hours after transfusion initiation. Tachycardia, hypotension,
and fever may also be present. Chest x-ray reveals bilateral pulmonary infiltrates.

Treatment of TRALI includes immediate transfusion cessation and respiratory supportive


care; most patients require ventilatory support. Some patients recover within 24-48 hours, but
the mortality rate is about 50% in previously critically ill patients.
*Multiple myeloma

Multiple myeloma
• Plasma cell neoplasm produces monoclonal paraprotein
Pathophysiology (immunoglobulin)

• Bone pain, fractures


• Constitutional symptoms (weight loss, fatigue)
Manifestations
• Recurrent infections

• Normocytic anemia
• Renal insufficiency
Laboratory • Hypercalcemia (constipation, muscle weakness)
• Monoclonal paraproteinemia (M-spike)

• Osteolytic lesions/osteopenia (osteoclast activation)


Radiology

This patient with constipation, back pain, and laboratory evidence of anemia, renal
insufficiency, and hypercalcemia likely has multiple myeloma (MM). MM is a plasma cell
neoplasm that commonly presents with manifestations related to bone marrow infiltration (bone
pain, fractures, hypercalcemia, anemia) or elevations in serum monoclonal protein (renal
insufficiency). At the time of diagnosis, approximately 25% of patients with MM have
hypercalcemia. Typically, this is a result of osteolytic bone destruction by the neoplastic
plasma cells. Although hypercalcemia may be asymptomatic with a calcium level <12 mg/dL,
fatigue, constipation, and depression are common. This patient's constipation is likely a result
of hypercalcemia.

Patients with suspected MM should have serum/urine protein electrophoresis (M-spike), a


peripheral blood smear (rouleaux), and a serum free light chain analysis. If these screening tests
suggest MM, diagnosis can be confirmed by bone marrow biopsy.
*Acute splenic sequestration

Acute splenic sequestration


• Complication of sickle cell disease seen in early childhood
• Vasoocclusion within spleen, causing trapping of red blood cells &
Pathophysiology
platelets

• Abdominal pain
• Palpable splenomegaly
Clinical features • Signs of anemia (tachycardia, pallor, fatigue)
• Hypotensive shock

• Acute drop in hemoglobin


Laboratory • Reticulocytosis
findings • Thrombocytopenia

• Isotonic fluid resuscitation


• Red blood cell transfusion
Treatment
• ± Splenectomy

This patient has splenomegaly with associated left-sided abdominal tenderness and signs of
acute anemia (eg, fatigue, pale mucosa, flow murmur). This presentation most likely represents
acute splenic sequestration crisis associated with sickle cell disease.

Splenic sequestration is a potentially life-threatening complication of sickle cell disease that


may be the presenting feature in some children. Because sickled red blood cells (RBCs)
become trapped in the splenic sinuses, a large volume of blood pools in the spleen. The spleen
rapidly enlarges (ie, tender splenomegaly), and hemoglobin drops acutely from baseline,
resulting in fatigue, pallor, tachycardia, and even shock (eg, hypotension, weak pulses). Platelet
counts are often low because of splenic sequestration, and reticulocyte production in the bone
marrow increases in response to decreased circulating RBCs. Scleral icterus reflects underlying
hemolysis.

Hemoglobin electrophoresis showing the presence of hemoglobin S (HbS) (beta globin gene
mutation) confirms the diagnosis of sickle cell disease. HbS polymerization in its deoxygenated
state causing RBCs to form a sickle shape.
*Disseminated intravascular coagulation

Disseminated intravascular coagulation


• Sepsis
• Severe traumatic injury
Major causes • Malignancy
• Obstetric complications

• Procoagulant excessively triggers coagulation cascade →


Pathophysiology • Formation of fibrin-/platelet-rich thrombi & fibrinolysis →
• Bleeding & organ damage (eg, kidneys, lungs)

• Thrombocytopenia
• Prolonged PT & PTT
Laboratory findings • ↓ Fibrinogen
• ↑ D-dimer
• Microangiopathic hemolytic anemia (schistocytes)

This patient's oozing from venipuncture and surgical sites and reduced urine output after a
serious traumatic injury raises strong suspicion for disseminated intravascular coagulation
(DIC), a consumptive coagulopathy. Traumatic injury increases the risk of DIC due to
endothelial (exposes tissue factor) and tissue damage (releases procoagulant proteins and
phospholipids); other common causes of DIC include sepsis, malignancy, and obstetrical
complications.

DIC is marked by:

• Overactivation of the coagulation cascade, leading to the formation of fibrin- and


platelet-rich thrombi and the consumption of coagulation factors (prolonged PT/PTT),
platelets (thrombocytopenia), and fibrinogen.
• Subsequent fibrinolysis (to break up the clots), which increases fibrin degradation
products (eg, D-dimer). Anticoagulation proteins (eg, protein C/S) are also consumed.

Patients with acute DIC usually develop bleeding from venipuncture/surgical sites,
ecchymosis, and petechiae. Organ damage (eg, renal insufficiency [reduced urine output in the
setting of intravascular fluids]) is also commonly seen
*Common causes of macrocytic anemia

Common causes of macrocytic anemia*


• Folate deficiency
• Vitamin B12 deficiency
• Myelodysplastic syndromes
• Acute myeloid leukemias
• Drug-induced (eg, hydroxyurea, zidovudine,
chemotherapy agents)
• Liver disease
• Alcohol use disorder
• Hypothyroidism

*Mean corpuscular volume >100 μm3.

This patient's presentation is consistent with macrocytic anemia, usually suggested by


abnormal red blood cell (RBC) indices, including an elevated (ie, >100 µm3) mean
corpuscular volume (MCV), elevated mean corpuscular hemoglobin (amount of hemoglobin),
and normal mean corpuscular hemoglobin concentration (low values seen with microcytosis;
elevated values seen with spherocytosis or some hemolytic anemias).

Macrocytic anemia is due to 2 primary etiologies, which the peripheral blood smear and
reticulocyte count findings can help distinguish:

• Megaloblastic causes (eg, impaired DNA synthesis due to deficiency in B12 or folate,
crucial one-carbon donors in nucleotide production): MCV values >110-115 µm3 are
almost exclusively associated with these anemias, which are also typically characterized
by low reticulocyte count (inability to efficiently generate new erythrocytes).
• Nonmegaloblastic causes (eg, alcohol use, hypothyroidism, drugs, liver disease): These
are associated with increased, normal, or decreased reticulocyte count.

Other findings suggestive of megaloblastosis include hypersegmented neutrophils (mean lobe


count >5), RBCs of various sizes (anisocytosis) and shapes (poikilocytosis), and basophilic
stippling (nonspecific; ribosomal remnants due to impaired erythrocyte maturation). Hemolysis
markers (eg, lactate dehydrogenase, indirect bilirubin) may be slightly elevated (increased
intramedullary and peripheral hemolysis), and mild thrombocytopenia and leukopenia can occur
(increased apoptosis).

Patients in whom megaloblastic anemia is suspected should undergo serum vitamin B12 and
folate level testing; methylmalonic acid and homocysteine levels can be added for those with
inconclusive or borderline results. Workup for nonmegaloblastic causes can be undertaken
when initial evaluation is unrevealing.
(Choice A) Lead poisoning is associated with basophilic stippling but would cause microcytic
(MCV <80 µm3) rather than macrocytic anemia. Basophilic stippling can also be due to
thalassemia, alcohol use, or vitamin deficiency.

(Choice B) Bone marrow biopsy helps diagnose myelodysplasia, which usually causes
cytopenia in ≥1 cell lines. Although patients can develop macrocytosis, bone marrow biopsy is
invasive and would not be performed prior to laboratory workup.

(Choice C) Direct antiglobulin test can help diagnose autoimmune hemolytic


anemias. Patients typically have hemoglobin of 7-10 g/dL with elevated reticulocyte count,
bilirubin, and lactate dehydrogenase. The haptoglobin is also markedly decreased, and
peripheral smear usually shows spherocytes. However, it would not cause very high MCV.

(Choice D) Measuring iron levels is useful for evaluating patients with microcytic anemia
(MCV <80 µm3).

Educational objective:
Macrocytic anemia can be due to megaloblastic (eg, B12 or folate deficiency) or
nonmegaloblastic (eg, alcohol use disorder, hypothyroidism, drugs, liver disease)
etiologies. Peripheral smear, reticulocyte count, and B12 and folate levels can aid diagnosis.
*Hereditary spherocytosis

Hereditary spherocytosis
• Usually autosomal dominant
Epidemiology
• Hemolytic anemia
• Jaundice
Clinical presentation
• Splenomegaly

• ↑ MCHC
• Negative Coombs test
• Spherocytes on peripheral smear
Laboratory findings
• ↑ Osmotic fragility on acidified glycerol lysis test
• Abnormal eosin-5-maleimide binding test

• Folic acid supplementation


• Blood transfusion
Treatment
• Splenectomy

MCHC = mean corpuscular hemoglobin concentration.

This infant has jaundice, hemolytic anemia, and elevated mean corpuscular hemoglobin
concentration (MCHC) consistent with hereditary spherocytosis (HS). HS is due to an
inherited defect in red blood cell (RBC) membrane proteins (eg, spectrin, ankyrin), which
results in fragile, sphere-shaped RBCs that become trapped and hemolyzed in the spleen.

Newborns with HS often have hyperbilirubinemia that requires phototherapy. Although


physiologic jaundice is expected in the first week of life due to increased RBC turnover (shorter
fetal RBC lifespan) and decreased bilirubin clearance, severe or prolonged neonatal jaundice
(>1 week) raises suspicion for inherited hemolytic anemia. Reticulocytosis on bloodwork
confirms hemolysis, and elevated MCHC reflects spherocytosis due to membrane loss and
RBC dehydration. Other laboratory findings include low to normal mean corpuscular volume,
negative direct Coombs test, and spherocytes on peripheral smear.

Treatment is primarily supportive. RBC or exchange transfusion may be considered for infants
with severe hemolysis.

(Choice A) ABO incompatibility refers to hemolysis when a mother with blood type O
transfers anti-A and/or anti-B antibodies to her fetus with a nonmatching blood type (ie, A, B,
AB). In this case, maternal blood type is AB; therefore, the mother has no anti-A or anti-B
antibodies to cause hemolysis of fetal RBCs.
(Choice B) Beta thalassemia is caused by a defect in the beta globin chain. It is common
among patients of Mediterranean heritage and can result in microcytic, nonimmune, hemolytic
anemia. However, it does not typically present until after age 6 months when fetal hemoglobin
decreases. In addition, MCHC is usually decreased in beta thalassemia.

(Choice C) Breastfeeding failure jaundice presents in the first week of life with indirect,
nonhemolytic hyperbilirubinemia due to inadequate breastmilk intake, resulting in significant
weight loss. This patient's jaundice has persisted despite appropriate weight gain.

(Choice D) Glucose-6-phosphate dehydrogenase (G6PD) deficiency can cause hemolysis and


prolonged jaundice in the newborn period but is an X-linked disorder more commonly seen in
male patients. In addition, MCHC is typically normal in G6PD deficiency.

Educational objective:
Hereditary spherocytosis (HS) is an inherited disorder caused by a defect in red blood cell
membrane proteins, leading to spherocyte formation and hemolysis. Hemolytic anemia with an
elevated mean corpuscular hemoglobin concentration in an infant with prolonged neonatal
jaundice is suggestive of HS.
*Immunologic blood transfusion reactions

Immunologic blood transfusion reactions


Transfusion reaction Onset* Cause Key features
• Angioedema,
Recipient anti-IgA hypotension, respiratory
Within seconds antibodies directed distress/wheezing,
Anaphylactic
to minutes against donor blood shock
IgA • IgA-deficient recipient

• Fever, flank pain,


hemoglobinuria
Within ABO incompatibility • Disseminated
Acute hemolytic
1 hr (often clerical error) intravascular coagulation
• Positive Coombs test

Febrile nonhemolytic Cytokine


Within • Fever & chills
(most common accumulation during
1-6 hr
reaction) blood storage
Recipient IgE against
Within • Urticaria
Urticarial blood product
2-3 hr
component
• Respiratory distress
• Noncardiogenic
Transfusion-related Within Donor anti-leukocyte pulmonary edema with
acute lung injury 6 hr antibodies bilateral pulmonary
infiltrates

• Often asymptomatic
• Laboratory evidence of
hemolytic anemia
Within days to Anamnestic antibody
Delayed hemolytic • Positive Coombs test,
weeks response
positive new antibody
screen

• Rash, fever,
gastrointestinal
Graft versus host Within weeks Donor T lymphocytes
symptoms, pancytopenia

*Time after transfusion initiation.

This patient likely has an acute hemolytic transfusion reaction, a rare but life-threatening reaction typically
caused by transfusion of mismatched blood (ie, ABO incompatibility), secondary to clerical
error. Pathogenesis involves recipient antibodies reacting with donor blood antigens, causing complement
fixation and hemolysis as well as a proinflammatory reaction.

Therefore, patients develop fever, chills, and hypotension within 1 hour after transfusion initiation;
nonspecific flank pain is also characteristic. Hemolysis causes hemoglobinuria and may lead to acute renal
failure (due to acute tubular necrosis), disseminated intravascular coagulation (eg, bleeding from intravenous
site, as in this patient), and shock. Key laboratory findings include a positive direct Coombs test; pink or dark
brown plasma and urine (due to hemolysis); and a repeat type and crossmatch confirming a mismatch.

Management includes immediate cessation of transfusion, aggressive intravenous fluid administration, and
supportive care.

(Choice B) Delayed hemolytic transfusion reactions result from an anamnestic antibody response to a red
blood cell antigen to which the patient was previously sensitized (eg, pregnancy). In contrast to this case,
patients typically develop extravascular hemolysis days to weeks after transfusion.

(Choice C) Transfusion-transmitted bacterial infection (TTBI), due to bacterial contamination of blood


products, is characterized by fever, tachycardia, and hypotension around 30 minutes after transfusion
completion. Flank pain is not associated with TTBI. In addition, TTBI most commonly occurs with platelet
(rather than red blood cell) transfusions because platelets are stored at room temperature, which increases their
susceptibility to bacterial growth.

(Choice D) Febrile nonhemolytic transfusion reaction is a common, benign reaction that occurs within 1-6
hours after transfusion due to the release of accumulated cytokines in stored blood. Patients develop transient
fever and chills. Hypotension and flank pain do not occur.

(Choice E) Individuals with IgA deficiency are at risk of developing anaphylactic reaction to blood products
due to preformed anti-IgA antibodies (IgE or IgG) that react against donor blood IgA. Angioedema,
hypotension, and respiratory distress (eg, wheezing) occur within minutes after transfusion initiation. This
patient's fever and lack of wheezing or rash make anaphylaxis less likely.

Educational objective:
Acute hemolytic transfusion reaction is an uncommon, life-threatening reaction due to transfusion of
mismatched blood (eg, ABO incompatibility), which causes fever, flank pain, and hemoglobinuria within 1 hour
of transfusion initiation. Continued hemolysis can lead to acute renal failure, disseminated intravascular
coagulation, and shock.
*B12 VS Folate deficiency
Chronic alcohol use is associated with folate deficiency due to limited dietary intake. In
addition, vegetarians are at risk of developing vitamin B12 deficiency, particularly if dairy
product consumption is limited; vegans are also at increased risk as they do not consume any
dairy products. This patient who is a vegetarian and uses alcohol daily likely has combined
folic acid and vitamin B12 deficiency; deficiency of either can cause megaloblastic anemia due
to effects on DNA synthesis. However, vitamin B12 deficiency also causes neurologic deficits,
including loss of proprioception and vibration sense mostly in the lower extremities, likely due
to a defect in myelin formation in the dorsal columns. Other associated abnormalities include
memory deficits, irritability, and dementia.

Response to folate and vitamin B12 repletion occurs in a predictable manner, with resolution of
the anemia within 1-2 months. Therefore, in patients with combined deficiency, replenishing
folic acid can partially correct hematologic abnormalities but is unlikely to improve neurologic
symptoms (and could in fact lead to their worsening, for unclear reasons) if it is not
accompanied by vitamin B12 supplementation.

As a result, this patient who received folate supplementation will still have neurologic
manifestations of vitamin B12 deficiency (eg, loss of proprioception), although hematologic
abnormalities (eg, megaloblastosis) are less likely to be present
*Rectus sheath hematoma

Rectus sheath hematoma


• Abdominal trauma, forceful abdominal contractions (eg, coughing)
• Anticoagulation
Risk factors
• Older age, female sex

• Acute-onset abdominal pain with palpable abdominal mass


Clinical • Blood loss anemia, leukocytosis
features • ± Nausea, vomiting, fever

• Hemodynamically stable: serial monitoring of CBC, reverse


Management anticoagulation and transfuse blood products when appropriate
• Unstable: angiography with embolization, surgical ligation

CBC = complete blood count.

This patient taking apixaban developed acute abdominal pain associated with a palpable
abdominal mass and anemia, suggesting the development of a rectus sheath hematoma (RSH).

RSH typically occurs due to rupture of the inferior epigastric artery, which supplies blood to
the lower aspect of the rectus abdominis muscle. The rectus sheath, which contains this muscle,
does not extend posteriorly below the arcuate line; therefore, bleeding below this line (eg, lower
aspect of rectus abdominis muscle) is relatively uncontained and can result in significant
hemorrhage with hematoma formation. RSH is often seen with blunt trauma or forceful
abdominal contractions (eg, severe coughing), particularly in patients receiving
anticoagulation therapy.

Manifestations include acute abdominal pain, often associated with rebound or guarding, and
a palpable abdominal wall mass. The mass does not cross the midline and does not change
with movement of the lower extremities (Fothergill sign). Nausea, vomiting, and fever may
also occur. Laboratory findings usually include anemia and leukocytosis (likely due to
inflammation). Abdominal CT confirms the diagnosis. Hemodynamically stable patients are
usually managed conservatively (eg, serial monitoring of CBC, reversal of anticoagulation
when appropriate); however, those with significant bleeding or hemodynamic instability (eg,
shock) may require angiography with embolization or surgery.
*acute graft-versus-host disease (GVHD)
This patient's profuse, watery diarrhea and maculopapular rash raise strong suspicion for acute graft-versus-
host disease (GVHD), an immune-mediated, multisystem inflammatory condition that occurs in 35%-50% of
patients who undergo allogeneic hematopoietic stem cell transplantation (HSCT). Acute GVHD arises when
donor T cells (particularly cytotoxic T cells) identify antigens on host epithelial cells as foreign and
subsequently generate a strong proinflammatory response.

Patients with acute GVHD generally develop symptoms within 100 days of transplantation, including the
following:

• Maculopapular rash that is often painful and can become confluent (resembling Stevens-Johnson
syndrome)
• Profuse, watery diarrhea that has a secretory pattern (eg, persistent, unrelated to eating, occurs at
night) and is often associated with crampy abdominal pain, nausea, and vomiting
• Liver inflammation with damage to the biliary tract epithelium, leading to elevated bilirubin, alkaline
phosphatase, and transaminases

Acute GVHD is usually diagnosed by biopsy (eg, colonoscopy with biopsy) after common infections such as
Clostridioides difficile and cytomegalovirus are ruled out. Glucocorticoids are the mainstay of therapy.

(Choice A) GVHD is almost entirely mediated by T cells; studies have found that removal of donor T cells
almost completely eliminates the risk of GVHD (but increases the risk of graft rejection and disease
recurrence). B cells and the humoral antibody response play a small role in the pathogenesis of GVHD. A
classic example of antibody-dependent cellular cytotoxicity is natural killer cell activation by antibodies (which
is used in some forms of cancer immunotherapy).

(Choice C) Cryptosporidiosis is a common cause of profuse secretory diarrhea and most often manifests in
those with impaired cellular or humoral immunity. However, a maculopapular rash would be atypical, and this
patient, who has been hospitalized since transplant and has been eating a low-microbial diet, is at a lower risk.

(Choice D) Conditioning chemotherapy is administered prior to transplantation to destroy active immune cells
(immunoablative) or to destroy hematopoietic stem cells in the bone marrow to create room for transplanted
stem cells (myeloablative). Although chemotherapy is often associated with a few days of nausea and vomiting,
gastrointestinal side effects generally resolve within a week; symptoms 3 weeks later would be very atypical.

(Choice E) Patients who undergo solid organ or HSCT are at increased risk for lymphoproliferative
disorders. Most cases present many years (not weeks) after transplantation with extranodal or CNS
lesions. Profuse watery diarrhea and a maculopapular rash would be atypical.

Educational objective:
Acute graft-versus-host disease is a common complication of allogenic hematopoietic stem cell
transplantation. It arises when donor T-lymphocytes recognize antigens on host epithelial cells as foreign and
initiate a strong inflammatory response. Patients usually manifest symptoms within 100 days of transplant,
including a maculopapular rash; profuse, watery diarrhea; and signs of hepatobiliary inflammation.
*Acute hemolytic transfusion reaction

Acute hemolytic transfusion reaction


• ABO incompatibility
Pathogenesis • Intravascular hemolysis

• Onset within minutes to 24 hr of transfusion


• Fever, chills, hypotension
Clinical findings
• Hemoglobinuria, flank pain

• Positive direct Coombs test


Laboratory findings • Hemolysis (eg, ↑ LDH, ↑ indirect bilirubin)

• Acute kidney failure


Complications • Disseminated intravascular coagulation

LDH = lactate dehydrogenase.


This patient with fever, flank pain, and dark red urine 10 minutes after transfusion is having an
acute hemolytic transfusion reaction (AHTR), a rare but potentially fatal reaction due to
transfusion of mismatched blood (eg, ABO incompatibility). The most common etiology is
clerical error (eg, patient misidentification).

AHTR typically occurs within minutes to hours of transfusion due to host antibodies attacking
donor blood antigens. The result is massive red blood cell destruction (ie, intravascular
hemolysis), which leads to toxic hemoglobin buildup in the kidney, renal tubular cell injury,
and subsequent acute renal failure. Therefore, findings that raise suspicion for AHTR
include flank pain and dark red urine (ie, hemoglobinuria) as well as fever, tachycardia, and
hypotension (due to inflammatory cytokine release) shortly following a transfusion. Patients
may also have elevated lactate dehydrogenase levels (from hemolysis) and disseminated
intravascular coagulation (eg, oozing intravenous site).

Diagnosis of AHTR is with a positive direct Coombs test, reflecting antibody-coated red blood
cells. Management includes immediate cessation of transfusion, aggressive intravenous fluid
administration, and supportive care.

(Choice A) Blood culture and antibiotics are indicated for a transfusion-transmitted bacterial
infection (TTBI) or for urinary tract infection (UTI), which can present with fever, tachycardia,
and hypotension. However, TTBI typically develops over hours (not minutes), and flank pain
and hemoglobinuria are usually not present. UTI can cause flank pain (eg, pyelonephritis) and
hematuria, but AHTR is a much more likely explanation of this patient's sudden-onset
hypotension and acute disseminated intravascular coagulation within minutes of a transfusion.

(Choices B and E) Cystoscopy and renal ultrasound can identify a urinary tract obstruction or
renal mass, which may present with flank pain and dark red urine. However, these conditions
do not cause hypotension or acute disseminated intravascular coagulation.

(Choice D) IgA levels are measured in patients with a suspected anaphylactic transfusion
reaction, which typically presents within seconds to minutes of transfusion initiation. Patients
with this reaction have angioedema and respiratory distress, which are not seen in this patient.

Educational objective:
Acute hemolytic transfusion reaction is a rare but potentially fatal condition due to transfusion
of mismatched blood (eg, ABO incompatibility). Patients can develop fever, hypotension, flank
pain, hemoglobinuria, and disseminated intravascular coagulation within minutes to hours of
transfusion. Diagnosis is with a positive direct Coombs test
*Common etiologies of pediatric stroke

Common etiologies of pediatric stroke


• Sickle cell disease
• Prothrombotic disorders
• Congenital cardiac disease
• Bacterial meningitis
• Vasculitis
• Focal cerebral arteriopathy
• Head/neck trauma

This patient presents with acute, focal neurologic deficits, which should raise concern for
stroke. Clinical features of stroke typically include focal weakness, hemiparesis, aphasia,
seizures, or altered mental status. Although stroke is more common in older adults, it can occur
in children, and the most common cause of pediatric stroke is sickle cell disease (SCD). In
addition, this patient's history of adoption from Nigeria, a country with a high prevalence of
SCD, raises the likelihood of SCD. SCD is an autosomal recessive disorder in which a
mutation in the beta globin chain of hemoglobin results in hemoglobin polymerization, red
blood cell deformation, and microvascular occlusion. Diagnosis of SCD is made via
hemoglobin electrophoresis, which can confirm the presence of sickle hemoglobin.

Chronic vasoocclusion can result in endothelial damage, intimal proliferation, and eventual
vascular stenosis, which increases the risk of cerebral ischemia, resulting in stroke. If concern
for stroke exists, an MRI should be performed to confirm the diagnosis.

(Choice A) The inflammation present in bacterial meningitis can result in vascular thrombosis
and cerebral ischemia. Diagnosis is confirmed by bacterial culture of the cerebrospinal
fluid. In the absence of fever, neck stiffness, or photophobia, meningitis is unlikely.

(Choice B) Cardiac disorders, most commonly congenital, can cause ischemic stroke via many
mechanisms, including cerebral ischemia from decreased cardiac output, paradoxical emboli
(venous clot shunted across septum to arterial circulation), infective endocarditis with septic
emboli, and intracardiac thrombus. Echocardiography can assess cardiac anatomy and function,
but cardiac disorders are unlikely without known cardiac history or murmur.

(Choice C) Seizures can be followed by Todd paralysis, which is focal weakness or paresis
that typically resolves in minutes to hours after the event. Electroencephalography can assess
for intermittent seizures or generalized slowing. Seizure is less likely in this patient given her
garbled speech and no history of seizures.

(Choice E) Mitochondrial myopathies, specifically mitochondrial encephalopathy with lactic


acidosis and stroke-like episodes (MELAS), can present with acute neurologic changes,
including hemiparesis. Muscle biopsy can confirm this diagnosis. These myopathies are much
less common than SCD. Further noninvasive workup (eg, serum lactate/pyruvate, creatine
kinase) must be performed before biopsy even if MELAS is suspected.

(Choice F) Urea cycle disorders result in hyperammonemia, which can cause acutely altered
mental status. Most urea cycle disorders are diagnosed in the newborn period, and
hyperammonemia leads to generalized confusion, not focal weakness.

Educational objective:
Sickle cell disease is the most common cause of pediatric stroke, and diagnosis can be
confirmed by hemoglobin electrophoresis.
*Iron studies in microcytic anemia

Iron studies in microcytic anemia


Transferrin
Transferrin
MCV Iron Ferritin saturation
(TIBC)
(iron/TIBC)
Iron deficiency ↓ ↓ ↓ ↑ ↓
Thalassemia ↓↓ Normal/↑ Normal/↑ Normal/↓ Normal/↑
Anemia of
chronic disease Normal/↓ ↓ Normal/↑ ↓ Normal/↓
(inflammation)
MCV = mean corpuscular volume; TIBC = total iron binding capacity.

This patient's history and laboratory results are most likely due to iron deficiency, the most common nutritional
deficiency in infants. Iron deficiency is typically due to the introduction of cow, goat, or soy milk before age 1
and inadequate consumption of iron-rich foods. In older children and adults, the cause is usually chronic blood
loss from the gastrointestinal tract.

The characteristic laboratory findings of iron deficiency anemia are shown in the table. Anisocytosis (red blood
cells of unequal size) is often the first finding on peripheral smear, and the increased red blood cell
distribution width (RDW) is a quantification of this abnormality. Gradually, the mean corpuscular volume
(MCV) and mean corpuscular hemoglobin (MCH) decline, correlating with microcytosis and hypochromia
on peripheral smear. The iron stores become depleted, resulting in a low reticulocyte count, decreased
transferrin saturation (serum iron/total iron binding capacity [TIBC]), and increased TIBC.

(Choices A and D) Thalassemia is associated with normal or elevated iron and ferritin levels due to high red
blood cell turnover. Reticulocyte count and total bilirubin are increased as a result of hemolysis.

(Choice B) Anemia of chronic disease is associated with decreased TIBC. Serum ferritin is often increased
due to ongoing inflammation.

(Choice C) Autoimmune hemolysis typically causes an increased reticulocyte count due to the bone marrow
response in an attempt to compensate for red blood cell destruction. The breakdown of red blood cells also
causes indirect hyperbilirubinemia, which is not seen in this patient.

(Choices E and H) Folate and vitamin B12 deficiency cause megaloblastic anemia, which is characterized by an
elevated MCV, elevated MCH, and normal mean corpuscular hemoglobin concentration.

(Choice G) Sideroblastic anemia is characterized by increased serum iron levels and normal TIBC.

Educational objective:
The characteristic laboratory findings of iron deficiency anemia are decreased mean corpuscular volume
(MCV), increased red blood cell distribution width (RDW), decreased serum iron, decreased transferrin
saturation, and increased total iron binding capacity (TIBC). The peripheral smear will show small,
hypochromic red blood cells
*Hemolytic uremic syndrome

Hemolytic uremic syndrome


• Escherichia coli serotype O157:H7 or Shigella dysenteriae
Pathogenesis • Vascular damage & microthrombi formation

• Preceding bloody diarrhea


• Fatigue, pallor
Clinical features • Bruising, petechiae
• Oliguria, edema

• Hemolytic anemia (schistocytes, ↑ bilirubin)


• Thrombocytopenia
Laboratory findings
• Acute kidney injury (↑ BUN, ↑ creatinine)

• Fluid & electrolyte management


Treatment
• Blood transfusions
• Dialysis

BUN = blood urea nitrogen.

This patient has normocytic anemia, thrombocytopenia, and edema following a diarrheal illness, findings that
are suggestive of hemolytic uremic syndrome (HUS). HUS most commonly occurs 5-10 days after an acute,
often bloody, diarrheal illness caused by a Shiga toxin-producing strain of Escherichia coli (serotype
O157:H7).

The Shiga toxin induces endothelial damage in renal glomeruli, leading to platelet activation, microthrombi
formation, and microangiopathic hemolytic anemia. The mechanical intravascular shearing of red blood
cells (RBCs) can result in jaundice or scleral icterus (from elevated indirect bilirubin) as well as schistocytes (ie,
fragmented RBCs) on peripheral smear, as seen in this patient. Patients may also have signs of
thrombocytopenia (eg, bruising) and acute kidney injury (eg, edema, elevated blood urea nitrogen and
creatinine).

Treatment of HUS is supportive and includes fluid and electrolyte management, blood transfusions, and
dialysis. Treatment of the prodromal diarrhea with antibiotics or antimotility agents (eg, loperamide), as in this
case, can increase the risk of developing HUS and is not recommended.

This patient has fatigue, edema, and anuria after a bloody diarrheal illness; these findings raise concern for
hemolytic uremic syndrome (HUS). HUS most commonly occurs after infection due to a Shiga toxin-
producing strain of Escherichia coli (serotype O157:H7). This toxin invades and destroys the colonic epithelial
lining, producing abdominal pain and bloody diarrhea, as seen in this patient.

HUS typically presents about a week after the diarrheal illness with signs of anemia (eg, fatigue, pallor),
thrombocytopenia (eg, bruising), and acute kidney injury (eg, oliguria, edema). The shearing of red blood
cells by platelet microthrombi leads to microangiopathic hemolytic anemia with schistocytes on peripheral
smear. Hemolysis is evidenced by decreased haptoglobin and increased indirect bilirubin, which can cause
scleral icterus (as in this case) as well as jaundice (Choice E).

Treatment of HUS is supportive (eg, fluid/electrolyte management, blood transfusions, dialysis). Most patients
improve within a few weeks.

(Choice A) A decreased ferritin level is seen in iron deficiency anemia (eg, chronic gastrointestinal bleeding),
which is unlikely in this patient with acute bloody diarrhea that is now resolved. Patients with HUS have
normal or increased ferritin levels because ferritin is an acute-phase reactant that rises with inflammation.

(Choice C) A decreased reticulocyte count in the setting of anemia indicates defective bone marrow (eg, viral
suppression, aplastic anemia, malignancy). Reticulocyte counts are increased with hemolytic anemia.

(Choice D) Disseminated intravascular coagulation occurs in severely ill patients (eg, sepsis) and is
characterized by microangiopathic hemolytic anemia, thrombocytopenia, and abnormal coagulation studies (ie,
increased PT). HUS is more likely in this child with a preceding bloody diarrheal illness; PT is normal in HUS.
(Choice F) The direct Coombs test is positive in patients with autoimmune hemolytic anemia, which can
present with fatigue and indirect hyperbilirubinemia. However, signs of acute renal injury (eg, edema, anuria)
would not be seen. Direct Coombs test is negative in HUS because red blood cells are mechanically destroyed.

Educational objective:
Hemolytic uremic syndrome, which is most commonly caused by a Shiga toxin-producing strain of Escherichia
coli (serotype O157:H7), is characterized by microangiopathic hemolytic anemia, thrombocytopenia, and acute
kidney injury.
*ESA-induced hypertension
This patient with advanced chronic kidney disease (CKD) and new-onset nausea and headache
has retinal hemorrhages and a significantly elevated blood pressure consistent with
hypertensive emergency. Hypertension is common in CKD patients and can be due to a
variety of causes that include medication nonadherence; sodium and water retention; increased
activity of the renin-angiotensin and sympathetic nervous systems; and an adverse effect of
erythropoiesis-stimulating agents (ESAs) (eg, epoetin, darbepoetin).

Up to 30% of patients receiving an ESA for CKD-induced anemia develop new or worsening
hypertension, which typically occurs 2-8 weeks after treatment initiation. Hypertension is
generally mild but can be severe, as in this patient. The mechanism by which ESAs cause
hypertension appears to be independent of increased blood volume and likely involves systemic
vasoconstriction caused by:

• Activation of erythropoietin receptors on vascular smooth muscle cells


• Heightened sensitivity of alpha receptors
• Depletion of endothelium-derived nitric oxide

The risk of ESA-induced hypertension is increased by large doses and a rapidly rising
hemoglobin soon after ESA administration, as in this patient whose hemoglobin increased from
8.3 to 11 g/dL in 3 weeks. Following acute treatment of hypertension, subsequent ESA doses
should be decreased with a goal of slowly increasing hemoglobin
*Management of sickle cell anemia

Management of sickle cell anemia


• Vaccination
• Penicillin (until age 5)
Maintenance • Folic acid supplementation
• Hydroxyurea (for patients with recurrent vaso-occlusive crises)

• Hydration
• Analgesia
Acute pain crises
• +/- Transfusion

The hallmark of sickle cell disease (SCD) is recurrent, acute, painful episodes due to vaso-
occlusion. Acute management consists of hydration and analgesia with nonsteroidal anti-
inflammatory drugs and opioids.

Hydroxyurea is indicated in patients with frequent pain crises, history of acute chest syndrome,
and/or severe symptomatic anemia. The major beneficial effect of hydroxyurea is to increase
fetal hemoglobin. Sickled hemoglobin is therefore proportionally decreased, resulting in
reduced polymerization of red blood cells (RBCs) and fewer episodes of vaso-
occlusion. Hydroxyurea also decreases the risk of acute chest syndrome and the need for blood
transfusions. The primary dose-limiting side effect of hydroxyurea is myelosuppression (eg,
neutropenia, anemia, thrombocytopenia), but it is otherwise relatively safe.

Educational objective:
Hydroxyurea, which works by increasing fetal hemoglobin levels, is indicated in patients with
sickle cell disease who have frequent vaso-occlusive crises. It is a relatively safe therapy with a
dose-limiting side effect of myelosuppression (neutropenia, anemia, thrombocytopenia).
*Microcytic/hypochromic anemias
This patient presents with microcytic/hypochromic anemia; therefore, the differential diagnosis
should include:

• Iron deficiency - decreased intake or increased blood loss


• Defective utilization of storage iron - anemia of chronic disease
• Reduced globin production - thalassemia and other hemoglobinopathies
• Reduced heme synthesis - lead poisoning, sideroblastic anemia

Iron deficiency is the most common cause of microcytic anemia. The other conditions
mentioned above are relatively uncommon. Iron studies help to confirm the diagnosis of iron-
deficiency anemia, as well as rule out other causes. If the diagnosis of iron-deficiency anemia
is confirmed, the underlying cause should be carefully sought.

Microcytic/hypochromic
Findings in Iron Studies
anemias
Typically depressed serum iron level, increased total iron
1. Iron-deficiency anemia
binding capacity (TIBC), and decreased serum ferritin level
2. Thalassemias Normal to high serum iron and ferritin levels
3. Anemia of chronic disease Below normal TIBC; Normal or increased serum ferritin level
4. Sideroblastic anemia Normal to high serum iron and ferritin values

(Choice A) Bone marrow sampling can be employed to determine the stored iron, but
measurement of the serum ferritin level is typically used as the indicator of iron reserves. If
myelodysplasia is strongly suspected, bone marrow sampling may be necessary.

(Choices C and D) Measurement of the serum folate level and the Schilling test are used in the
work-up of patients with megaloblastic anemia.

(Choice E) The HbA2 level is frequently elevated in patients with thalassemia.

Educational Objective:
Iron studies are indicated in patients who present with microcytic/hypochromic anemia. These
are useful in confirming the diagnosis of iron-deficiency anemia, which is the usual cause of
microcytic/hypochromic anemia, and in ruling out other causes.
*Upper extremity deep venous thrombosis

Upper extremity deep venous thrombosis


• Central catheter or PICC line
• Young, athletic males (spontaneous)
Epidemiology
• Thoracic outlet obstruction

• Unilateral arm or forearm edema


• Pain/heavy sensation
Manifestations • Erythema
• Dilated subcutaneous collateral veins

• Doppler or duplex ultrasonography


Diagnosis
• 3 months of anticoagulation
Treatment
PICC = peripherally inserted central catheter.

This patient's arm swelling, pain, and mild erythema developed in the setting of a peripherally
inserted central catheter (PICC), raising strong suspicion for upper extremity deep venous
thrombosis (DVT). All upper extremity central catheters (eg, internal jugular, subclavian) are
associated with increased risk of DVT due to endothelial trauma during insertion and improper
positioning of the catheter tip; the risk of DVT is somewhat higher with PICCs because they
travel long distances through narrow peripheral veins (often occupying >50% of total vein
caliber). Hospitalized patients, particularly those with a hypercoagulable state (eg,
malignancy), are most likely to develop PICC-related DVT.

Upper extremity DVT is most common within 7-14 days of PICC insertion. The primary
manifestation is arm or forearm swelling. Pain and mild erythema may also occur. In
contrast to PICC infection, patients with DVT usually have a normal appearing catheter
insertion site (eg, no purulence). The diagnosis is made with compression or duplex
ultrasonography, which visualizes the subclavian, axillary, and brachiocephalic
veins. Although upper extremity DVT is associated with a relatively low risk of pulmonary
embolism (~6%), treatment with 3 months of anticoagulation is recommended. The PICC line
can often remain in place.
*acquired methemoglobinemia

This patient has new-onset hypoxia by pulse oximetry (85%) during an endoscopic procedure,
with a significant difference between the oxygen saturation value estimated on blood gas
analysis and that obtained on pulse oximetry (ie, large oxygen saturation gap). He likely has
acquired methemoglobinemia due to anesthetic use.

Some medications - most commonly topical anesthetics (eg, benzocaine), dapsone, and
nitrates (in infants) - cause the iron component of hemoglobin to be oxidized, thereby forming
methemoglobin, which cannot bind oxygen. The remaining normal hemoglobin also has an
increased affinity for oxygen, resulting in less oxygen delivery to tissues. Because
methemoglobin absorbs light at distinct wavelengths, pulse oximetry commonly is ~85% (as
seen in this patient) regardless of the true oxygen saturation. In parallel, blood gas analysis
frequently returns a falsely elevated result for oxygen saturation (eg, 99% in this patient) as it
provides an estimate based only on the PaO2, not on effective hemoglobin-oxygen
binding. These inaccurate readings create the large oxygen saturation gap.

Cyanosis can occur when methemoglobin comprises ~10% of total hemoglobin, but hypoxia
symptoms (eg, headache, lethargy) occur only when levels surpass 20%. At levels >50%, there
is risk of severe symptoms (eg, altered mental status, seizures, respiratory depression) and
death. Treatment involves discontinuing the causative agent and administering methylene blue,
which helps reduce iron to its normal state.

(Choice A) Anaphylaxis (eg, due to medication) can lead to respiratory compromise and
hypoxia but is associated with symptoms such as wheezing, stridor, and dyspnea.

(Choices C and G) Although atelectasis and mucus plugging can lead to transient hypoxemia
due to a collapsed portion of lung, the hypoxemia is commonly reversible with supplemental
oxygen and would not explain the oxygen saturation gap. Procedural hypoventilation should
also correct with supplemental oxygen.
(Choice D) This patient has clear lung fields and normal heart sounds, making pulmonary
edema less likely. Pulmonary edema also typically improves with supplemental oxygen and is
not characterized by a large oxygen saturation gap.

(Choice E) Patients with liver disease can have large-volume ascites and severe muscle
wasting, which may lead to decreased diaphragmatic excursion and resultant
hypoxia. However, this patient has only mild ascites on ultrasound and no evidence of muscle
wasting.

(Choice F) Hepatopulmonary syndrome results from arteriovenous shunting in the lungs and
can occur after years of liver disease. Patients frequently exhibit platypnea (induced dyspnea in
the upright position) and orthodeoxia (hypoxia in the upright position). This syndrome would
not explain the patient's large oxygen saturation gap.

Educational objective:
Acquired methemoglobinemia results from the oxidization of iron in hemoglobin, which is most
commonly due to topical anesthetic agents or dapsone. It presents with hypoxia, a characteristic
pulse oximetry reading of ~85%, and a large oxygen saturation gap.
*Management of hemorrhage due to trauma

Management of hemorrhage due to trauma


• Control bleeding from any compressible site
Control • Early transfer to location (trauma center, operating room, angiography
bleeding suite) for definitive care & control of hemorrhage

• Transfuse blood products early in resuscitation


Give blood • Massive transfusion protocol*: ratio of 1:1:1 FFP/pRBCs/platelets to
products early minimize risk of coagulopathy

• Limit to ~1 L if patient is hypotensive


Limit use
• Switch to blood products as soon as available
of crystalloids
*Massive transfusion protocol likely required in patients with ≥2 of the following: SBP ≤90 mm Hg,
pulse ≥120/min, positive FAST examination, and penetrating mechanism of injury.

FAST = Focused Assessment with Sonography for Trauma; FFP = fresh frozen plasma; pRBCs = packed
red blood cells; SBP = systolic blood pressure.

This patient has multiple sites for blood loss: hemothorax (decreased breath sounds, 1.5 L of blood from chest
tube), hemoperitoneum (tender abdomen, positive focused assessment with sonography for trauma [FAST]
examination), and femur fracture (left thigh deformity). In the setting of blood loss, a systolic blood pressure
(SBP) <90 mm Hg, along with tachycardia >120/min, altered mental status (eyes closed, moaning), and delayed
capillary refill (cold, clammy extremities), suggests at least 30% loss of circulating blood volume, generally
classified as class III hemorrhage. This presentation is consistent with hemorrhagic shock from bleeding at
noncompressible sites (chest and abdomen).

Type-specific blood is the optimal choice for transfusion; however, depending on several factors (eg, trauma vs
nontrauma center, presence of antibodies), it can take significant (eg, >20 min) time to prepare (eg, after type
and cross-matching). Patients in hemorrhagic shock, such as this one, can exsanguinate in that time (Choice
E). Therefore, in such situations, group O, Rh D-negative blood should be administered unless type-specific
blood is immediately available.

Most hospitals keep O Rh D-negative blood (universal donor) ready for use in emergencies. To prevent
hemolytic disease of the newborn in future pregnancies, it is particularly important that this type of blood be
given to women of childbearing age, as in this patient, and to young girls. In men and in women past
childbearing age, O Rh D-positive blood can be given (Choice D).

When there is concern for massive transfusion requirements (eg, SBP ≤90/mm Hg, pulse ≥120/min, positive
FAST, penetrating mechanism of injury), blood products should be given in a 1:1:1 fresh frozen plasma/packed
red blood cells/platelets ratio to minimize risk of coagulopathy (eg, widespread tissue damage, lactic acidosis).

(Choice A) In acutely injured trauma patients, excess crystalloid use exacerbates the lethal, trauma-induced
triad of hypothermia, acidosis, and coagulopathy, increasing mortality. Research supports limited crystalloid
administration (eg, 1 L) in patients with hemorrhagic shock when blood is not immediately available.

(Choice B) Vasopressors are appropriate for patients with severe peripheral vasodilation due to distributive (eg,
septic, neurogenic) shock. Patients with hemorrhagic shock should receive volume resuscitation with blood
transfusion while awaiting definitive management of the hemorrhage (eg, surgical intervention). Vasopressor
administration without appropriate resuscitation can worsen tissue perfusion.

Educational objective:
Blood transfusion should be initiated early in patients with hemorrhagic shock. Group O, Rh D-negative blood
(universal donor) should be transfused while waiting for type-specific blood to be available.

This patient with penetrating abdominal trauma has signs of hemorrhagic shock (eg, hypotension, tachycardia,
cool extremities) and, likely, ongoing hemorrhage (eg, free intraperitoneal fluid on Focused Assessment with
Sonograph for Trauma). Such patients once received early, aggressive crystalloid (eg, normal saline)
resuscitation to restore intravascular volume and normal blood pressure. Now, balanced resuscitation (aka,
damage-control resuscitation) is used; it includes the following measures:

• Limiting use of crystalloids (eg, ≤1 L), which dilute existing coagulation (eg, clotting) factors and
platelets, thereby increasing coagulopathy
• Replacing lost intravascular volume with blood products (rather than crystalloids), transfused in a
ratio similar to that of whole blood (eg, 1:1:1 ratio of packed red blood cells/plasma/platelets)
• Permitting hypotension (ie, permissive hypotension) to limit ongoing hemorrhage and/or prevent clot
disruption and rebleeding

With balanced resuscitation, blood products are administered only as needed to maintain a blood pressure (eg,
mean arterial pressure ~65 mm Hg) sufficient for tissue perfusion, until definitive hemorrhage control (eg,
surgical intervention) can be achieved.

In addition to coagulopathy, aggressive crystalloid resuscitation has been associated with the following
detrimental effects:

• Hypothermia: Room temperature fluids are cooler than body temperature and can cause hypothermia
(which worsens coagulopathy), especially when administered in large volumes (Choice B).
• Acidosis: Large volumes of rapidly administered normal saline can cause non–anion gap
hyperchloremic metabolic acidosis (vs alkalosis), which can further worsen the performance of
coagulation factors (Choice D).
• Increased mortality: Overuse of crystalloids worsens the "lethal triad" (hypothermia, acidosis,
coagulopathy) already present in severely injured trauma patients, increasing the risk for mortality.

(Choice A) The trauma-induced inflammatory response can cause capillary leakage and diffuse pulmonary
edema, sometimes resulting in acute respiratory distress syndrome (ARDS). Crystalloids have been shown to
cause a dose-dependent increase (vs decrease) in ARDS; they have a lower oncotic pressure than blood and can
leak more easily into the pulmonary interstitium.

Educational objective:
Large-volume crystalloid resuscitation increases coagulopathy, hypothermia, and mortality in trauma
patients. Balanced resuscitation, which restricts crystalloid use and uses blood products to maintain a blood
pressure just sufficient for tissue perfusion (ie, permissive hypotension) until hemorrhage is controlled, can
decrease these adverse effects.
*Ddx of mediastinal masses

This patient has a large, anterior mediastinal mass that is causing cough, shortness of breath, and postural
wheezing due to mass-effect compression of other mediastinal structures, including the trachea. The differential
diagnosis of anterior mediastinal masses includes the 4 Ts: thymoma, teratoma (and other germ cell tumors
[GCTs]), "terrible" lymphoma, and thyroid neoplasm. When serum hormone levels of alpha fetoprotein (AFP)
and β-hCG are elevated, a malignant GCT is likely.

Malignant GCTs occur predominantly in young men and can usually be distinguished by differences in serum
hormone levels:

• Seminomas, which are more common than nonseminomatous GCTs, may cause elevated serum β-hCG
in a minority of cases, but AFP is almost always normal (Choice C).
• Nonseminomatous GCTs, which include yolk sac tumors, choriocarcinoma, embryonal carcinoma, and
mixed GCTs that contain a mixture of different cell types, cause an elevated AFP and/or β-hCG in the
majority (~85%) of patients.
In contrast, benign teratomas of the mediastinum typically do not cause elevation of either AFP or β-
hCG. They can also often be distinguished from other GCTs on imaging by the presence of fat, bone, or fluid.

Biopsy is typically performed to confirm the diagnosis prior to treatment. In addition, testicular ultrasound is
performed to determine whether a primary testicular tumor is present because management and prognosis differ
among primary mediastinal and metastatic (eg, testicular) GCTs.

(Choice A) Lymphoma can arise in any of the mediastinal compartments, including the anterior
mediastinum. However, it is not associated with elevated β-hCG and AFP. In addition, patients with high-
grade (ie, fast-growing) lymphoma often have systemic symptoms (eg, fever, weight loss, night sweats).

(Choice D) Small cell neuroendocrine tumors may present as a lung mass, but an isolated anterior mediastinal
mass would be unusual.

(Choice E) Thymomas are a common cause of anterior mediastinal masses. However, they do not cause
elevated β-hCG and AFP. In addition, they usually occur in middle-aged patients and often lead to
paraneoplastic syndromes such as myasthenia gravis (with abnormal levels of anti–acetylcholine receptor
antibodies).

Educational objective:
The differential diagnosis for anterior mediastinal mass includes the 4 Ts: thymoma, teratoma (and other germ
cell tumors), "terrible" lymphoma, and thyroid neoplasm. Seminomas may cause elevated β-hCG, but alpha
fetoprotein (AFP) is essentially always normal. In contrast, most nonseminomatous germ cell tumors cause
elevated AFP and/or β-hCG.

.............................................................................................

This patient with vague chest discomfort, frequent upper respiratory infections, and a middle mediastinal mass
most likely has a bronchogenic cyst.

Mediastinal masses are categorized according to their location within the anterior, middle, or posterior
mediastinal compartments. Most sources define the middle compartment as the space between the anterior
pericardium and the ventral surface of the thoracic spine; it contains the heart and great vessels, trachea and
main bronchi, esophagus, pericardium, and lymph nodes. Masses that arise from these structures may include
the following:

• Lymphoma or lymphadenopathy (most common), with a broad differential of mediastinal


lymphadenopathy that includes sarcoidosis, metastatic lung cancer, and granulomatous infections
• Benign cystic tumors, including bronchogenic cysts, pericardial cysts, and enteric cysts
• Vascular masses, including aortic aneurysms and aortic arch abnormalities
• Esophageal tumors, including large esophageal cancers with extraesophageal invasion

Bronchogenic cysts, the most common type of benign cystic tumor, typically arise from anomalous budding of
the foregut during congenital development of the tracheobronchial tree. In infancy or adulthood, affected
patients may become symptomatic with chest discomfort and nonspecific respiratory symptoms such as
recurrent coughing or frequent respiratory infections. CT scan of the chest with intravenous contrast can
usually be used to confirm the diagnosis.

(Choice B) Neuroblastoma and other neurogenic tumors (eg, schwannoma, neurofibroma) that occur at the
level of the thoracic vertebrae can present with chest discomfort. However, they are generally located in the
posterior (vs the middle) mediastinum. MRI is often used to evaluate spinal involvement of neurogenic tumors.
(Choices C, D, and E) Retrosternal thyroid tissue, teratoma, and thymoma can occur as mediastinal masses
and cause chest discomfort and compressive symptoms (eg, cough, dyspnea); however, these masses typically
arise in the anterior (vs the middle) mediastinum. In addition, paraneoplastic syndromes (myasthenia gravis
being the most common) are common in patients with thymoma.

Educational objective:
Mediastinal masses are categorized according to their compartments (ie, anterior, middle, posterior). The
middle compartment contains the heart and great vessels, trachea and main bronchi, esophagus, pericardium,
and lymph nodes. Middle compartment masses may include bronchogenic cysts, which arise from congenital,
anomalous budding of the foregut and can cause chest discomfort and nonspecific respiratory symptoms.
*sickle cell trait
This patient underwent a long plane flight and developed a splenic infarction, an uncommon
condition that usually arises in the setting of acute splenic artery (or subbranch) occlusion;
possible underlying etiologies include a hypercoagulable state, embolic disease, or
hemoglobinopathy. In this patient, a mild hemoglobinopathy such as sickle cell trait is most
likely given :

1- the mild intravascular hemolysis (eg, elevated reticulocyte count, indirect


hyperbilirubinemia),

2- normal hemoglobin level,

3- negative past medical history for pain crises (making sickle cell disease unlikely).

Sickle hemoglobin mutations are often linked to Sub-Saharan Africa but are also common in
other regions that have a high burden of malaria, including Central/South America, the
Caribbean, the Middle East, the Mediterranean, and India. Most patients with sickle cell trait
(eg, one altered beta-hemoglobin chain) are asymptomatic and considered benign carriers;
however, certain stressors, such as flying at high altitude or dehydration due to alcohol
consumption, can sometimes lead to intravascular hemolysis, tissue ischemia (eg, splenic
infarction), and/or vasoocclusive pain. Hemoglobin electrophoresis is diagnostic.

Electrophoresis patterns in sickle cell syndromes


HbA HbA2 HbF HbS HbC
Normal ++++ + + None None
Sickle cell trait +++ + + +++ None
Sickle cell anemia (SCA) None + + ++++ None
SCA on hydroxyurea None + ++ +++ None
Hemoglobin SC disease None + + +++ +++

Sickle cell disease is a hemoglobinopathy with an autosomal recessive inheritance. When both
parents carry the trait, their children are at risk for inheriting the disease. This patient's
hemoglobin electrophoresis pattern and normal complete blood count are consistent with being
a carrier for the sickle cell trait.

Individuals with sickle cell trait are generally asymptomatic and can lead a healthy
life. Although patients are at increased risk for renal issues, the most common of these is
painless microscopic or gross hematuria that results from sickling in the renal
medulla. Hyposthenuria (impairment in concentrating ability) is also common and can present
as nocturia and polyuria. Less commonly, there may be an increased risk of urinary tract
infections, particularly during pregnancy (Choice G). Splenic infarctions (Choice F) are
uncommon but can occur at high altitudes.

(Choices A, B, D, and E) Other complications of sickle cell disease are unlikely to occur with
sickle cell trait.

Educational objective:
Most patients with sickle cell trait lead normal, healthy lives. Painless hematuria is the most
common complication.

(Choice A) 24-hour ECG monitoring can diagnose atrial fibrillation, which can cause splenic
infarction due to atrial thromboembolism. However, atrial fibrillation would be unusual in a
24-year-old and does not typically cause reticulocytosis or bilirubinemia.

(Choice B) Blood cultures can diagnose infective endocarditis, which can cause splenic
infarction due to septic thromboembolism. However, most patients are ill (eg, fever, chills,
anorexia, weight loss) and cardiac murmur is usually present on examination.

(Choice C) Bone marrow biopsy can diagnose hematopoietic malignancies such as


myelofibrosis, which may cause splenic infarction due to venous occlusion from congestive
splenomegaly. However, myelofibrosis typically presents in older adults and causes
splenomegaly (a hallmark feature not seen in this patient).

(Choice D) Factor V Leiden mutation is a common cause of thrombosis in middle-aged adults


and typically causes venous thromboembolism, which can occasionally lead to splenic
infarction. However, signs of intravascular hemolysis are not typically present.

Educational objective:
Splenic infarction usually occurs in the setting of splenic artery (or subbranch) occlusion due to
hypercoagulable states, embolic disease, or hemoglobinopathy (eg, sickle cell disease or
trait). Although patients with sickle cell trait are generally asymptomatic, they occasionally
develop splenic infarction and intravascular hemolysis in the setting of certain stressors such as
air travel or dehydration. Diagnosis of most hemoglobinopathies is made by hemoglobin
electrophoresis.
*Modified Wells criteria for DVT

Modified Wells criteria for pretest probability


of deep venous thrombosis
• Previously documented DVT
• Active cancer
• Recent immobilization of the legs
• Recently bedridden >3 days
Score 1 point for • Localized tenderness along vein distribution
each feature • Swollen leg
present • Calf swelling >3 cm compared to other leg
• Pitting edema
• Collateral superficial nonvaricose veins
• Alternate diagnosis more likely (−2 points)

• 0 points = Low probability


Total score for • 1 or 2 points = Moderate probability
clinical probability • ≥3 points = High probability

DVT = deep venous thrombosis.

This patient with a history of injection drug use has right leg edema, erythema, and warmth. Although these
symptoms often occur in a localized manner in injection drug users due to skin and soft tissue infection, the
presence of symptoms in the entire leg, increased thigh and calf diameters, and injection marks in the right groin
raises strong suspicion for acute iliofemoral deep venous thrombosis (DVT).

Common risk factors for first DVT include prolonged immobility or hospitalization, recent major surgery,
obesity, malignancy, oral contraceptive/hormone replacement use, and pregnancy. However, patients who
inject drugs into the femoral vein also are at high risk for DVT due to iliofemoral venous wall trauma,
chemical irritation, and/or infection.

Manifestations of lower extremity DVT generally include unilateral leg swelling, pain, and warmth; physical
examination often reveals dilated superficial veins and increased diameter of the calf or thigh when compared
to the unaffected leg. Initial evaluation typically includes D-dimer (in those with low or moderate pretest
probability) and/or lower extremity Doppler ultrasound examination (in those with high pretest probability).

(Choice A) Acute lymphangitis is a common complication of injection drug use due to inoculation of skin flora
into subcutaneous tissue. However, this condition is generally marked by tender red streaks up the lymphatic
channels with painful, swollen, erythematous regional lymphadenitis. The presence of diffuse right leg
symptoms and increased calf or thigh diameter enlargement makes DVT far more likely.

(Choice C) Chronic lymphatic filariasis often results in limb lymphedema due to long-standing lymphatic
inflammation. However, most cases occur in sub-Saharan Africa, Southeast Asia, the Caribbean, and the Indian
subcontinent. In addition, this patient with injection marks in the right groin is far more likely to have acute
DVT due to iliofemoral vein injury.

(Choice D) Lymphogranuloma venereum is a tropical and subtropical disease caused by serovars of Chlamydia
trachomatis; it is marked by genital ulcers and inguinal lymphadenitis.
(Choice E) Necrotizing myositis is a rare condition generally marked by severe systemic toxicity (eg, high
fever, hypotension, toxic appearance) and rapidly progressive skin changes (eg, discoloration, gangrene). This
patient's normal vital signs and slow-onset symptoms make this highly unlikely.

Educational objective:
Patients who inject drugs into the femoral vein can develop iliofemoral deep venous thrombosis. This disorder
is typically marked by unilateral leg edema, warmth, and erythema with evidence of dilated superficial veins
and increased calf/thigh diameter on examination

Management:

This patient with recent acute deep vein thrombosis (DVT) has life-threatening gastrointestinal bleeding (ie,
hypotension, major transfusion requirement) soon after anticoagulation was initiated. Anticoagulation therapy
can unmask sources of bleeding. Given the patient's active/uncontrolled bleeding sources (small bowel
angiodysplasias not accessible by endoscopy), resumption of anticoagulation would lead to potentially severe
rebleeding.

However, lack of anticoagulation in this patient with recent acute DVT increases her risk for pulmonary
embolism (PE). Inferior vena cava (IVC) filters trap clots in transit from the legs to the lungs. They are an
option for patients with acute DVT and absolute contraindications to anticoagulation (eg, intracranial
hemorrhage, active/uncontrollable bleeding source).

Although IVC filters protect against PE (relative risk ~0.5), they increase the long-term risk of recurrent DVT
(relative risk ~2). Prevention of PE is considered the more pressing clinical priority, especially in this patient
with decreased cardiopulmonary reserve (ie, severe chronic lung disease). Short-term complications of IVC
filters include bleeding and infection at the puncture site.

(Choice A) Due to its quickly titratable and reversible kinetics, unfractionated heparin is an attractive short-
term strategy for patients requiring anticoagulation who have non–life-threatening bleeding sources that are
under control and accessible for intervention in the event of rebleeding (eg, surgical site). It would not be
appropriate for this patient who just experienced life-threatening hemorrhage and has ongoing, inaccessible,
active bleeding.

(Choice C) Although the patient's existing popliteal DVT may spontaneously dissolve after several weeks, the
clot itself is a thrombogenic nidus, tending to stimulate its own propagation. Furthermore, the risk of PE is very
high (>50%) with unprotected DVT. Therefore, observation without intervention would be unsafe. The option
of an IVC filter should be discussed with the patient.

(Choice D) Compression stockings are used for post-thrombotic syndrome, which can produce pain, swelling,
and lower extremity ulceration from chronic venous insufficiency following DVT. However, they are not
effective in preventing or treating existing acute DVT.

(Choice E) Aspirin is sometimes offered for secondary prevention of DVT after a patient completes a full
course of anticoagulation. In contrast, aspirin monotherapy has a trivial therapeutic effect as primary treatment
for acute DVT and may increase the risk of gastrointestinal rebleeding.

Educational objective:
Inferior vena cava filters are an option for patients with acute deep vein thrombosis and absolute
contraindications to anticoagulation (eg, intracranial hemorrhage, active and difficult-to-treat source of major
bleeding). They trap clots in transit, reducing the risk of clinically significant pulmonary embolism.
*type 2 HIT

Clinical features of type 2 heparin-induced


thrombocytopenia
Suspected with heparin exposure >5 days & any
of the following:

• Platelet count reduction >50% from


baseline
Clinical signs • Arterial or venous thrombosis
• Necrotic skin lesions at heparin injection
sites
• Acute systemic (anaphylactoid) reactions
after heparin

• Serotonin release assay: gold standard


Diagnostic confirmatory test
• Start treatment in suspected cases prior to
evaluation
confirmatory tests

• Stop all heparin products


• Start a direct thrombin inhibitor (eg,
Therapy argatroban) or fondaparinux (synthetic
pentasaccharide)

Heparin-induced thrombocytopenia (HIT) is a life-threatening complication of heparin


therapy. Heparin induces a conformational change in a platelet surface protein (platelet factor
4), which exposes a neoantigen. In patients with HIT, HIT antibodies form in response to the
neoantigen and bind to the surface of platelets, causing platelet aggregation, thrombocytopenia
(or drop in platelets >50%), and a prothrombotic state. In patients receiving heparin
subcutaneously (eg, enoxaparin), a classic thrombotic complication of HIT is skin necrosis at
the abdominal injection site.

HIT is diagnosed by immunoassay (only if high titer) or functional assay (eg, serotonin release
assay [gold standard]). Treatment involves immediately discontinuing all heparin products
(including heparin flushes) and initiating an alternate anticoagulant (eg, argatroban,
fondaparinux). If HIT is suspected, treatment should not be delayed for laboratory confirmation
as the risk of venous and arterial thrombosis is as high as 50% in untreated HIT.
(Choice A) Warfarin is an oral anticoagulant that can cause an acquired protein C deficiency,
which may manifest as skin necrosis. However, this patient has not received any oral
anticoagulation. Warfarin is often used for chronic anticoagulation maintenance in HIT but not
until argatroban or fondaparinux are initiated and platelets are >150,000/mm 3.

(Choice C) Delayed hypersensitivity reactions typically cause skin erythema, vesicles, or


bullae. More severe reactions are associated with systemic symptoms, such as fever. This
patient has necrotic lesions likely at the site of her low-molecular-weight heparin injections,
making HIT much more likely.

(Choice D) Cholesterol embolization can occur due to plaque embolization into small arteries
following coronary angiography. Cholesterol embolization classically causes skin findings (eg,
livedo reticularis [mottled erythema], gangrene, cyanosis) that most commonly affect the lower
extremities. With necrotic lesions on the abdominal wall, HIT is much more likely.

(Choice E) Viral reactivation is common with herpes zoster. Lesions are typically unilateral,
in a single dermatome, and quite painful. They tend to be vesicular, not necrotic.

Educational objective:
Heparin-induced thrombocytopenia (HIT) should be suspected in patients on heparin who
develop thrombocytopenia or thrombotic complications. In patients receiving heparin
subcutaneously (eg, enoxaparin), a classic thrombotic complication is skin necrosis at the
abdominal injection site. HIT is treated by discontinuing all heparin products and initiating an
alternate anticoagulant (eg, argatroban, fondaparinux).
*vitamin K deficiency
This patient's clinical presentation suggests an acquired bleeding disorder, most likely due to
vitamin K deficiency. Vitamin K is a fat-soluble vitamin that plays an important role in
hemostasis by serving as a cofactor in the enzymatic carboxylation of glutamic acid residues on
prothrombin complex proteins. The body obtains vitamin K exogenously from intestinal
absorption of dietary Vitamin K and endogenously from bacterial production of vitamin K in
the intestine. Vitamin K deficiency is most commonly due to inadequate dietary intake,
intestinal malabsorption, or hepatocellular disease causing loss of storage sites. The liver can
normally store a 30-day supply of vitamin K, but an acutely ill person with underlying liver
disease can become vitamin K deficient in as little as 7-10 days.

This patient received no enteral nutrition postoperatively and was given a broad-spectrum
antibiotic, and so both natural sources of vitamin K were compromised. In addition, he also
likely has alcoholic liver disease, further limiting his vitamin K stores. Vitamin K deficiency
decreases plasma levels of all the prothrombin complex proteins (factors 2, 7, 9, 10, and protein
C and S). This initially increases the prothrombin time (PT), followed by prolongation of the
partial thromboplastin time (PTT). Administration of vitamin K rapidly replenishes the stores
in 8-10 hours, and fresh frozen plasma may be used for the management of acute hemorrhage in
the interim.

Educational objective:
Vitamin K deficiency is usually due to inadequate dietary intake, intestinal malabsorption, or
hepatocellular disease. An acutely ill patient with underlying liver disease can become vitamin
K deficient in 7-10 days. Laboratory studies usually show prolonged prothrombin time
followed by prolonged partial thromboplastin time
*Posttransplantation lymphoproliferative disorder

Posttransplantation lymphoproliferative disorder


• Immunosuppression following solid-organ or stem cell
transplantation → suppressed cytotoxic T-cell
immunosurveillance → unchecked viral replication →
Pathogenesis immortalized lymphocytes or plasma cells
• Epstein-Barr virus causes >95% of cases, but other human
herpesviruses (eg, HHV8) can also trigger the disease

• Fever
• Lymphadenopathy & hepatosplenomegaly
Manifestations • Leukopenia
• Masses in nonlymphatic tissue

• High viral titers


Diagnosis • Biopsy evidence of lymphoid or plasma cell proliferation

• Reduce immunosuppression
• B-cell immunotherapy (eg, rituximab)
Treatment
• Epstein-Barr virus titer monitoring

This patient developed a febrile illness with lymphadenopathy and leukopenia while taking
immunosuppressive medications following solid-organ transplantation. Although many
pathogens can cause febrile illness in the setting of immunocompromise, the presence of an
elevated Epstein-Barr virus (EBV) titer suggests posttransplantation lymphoproliferative
disorder (PTLD).

PTLD is a plasmocytic or lymphoid proliferation that occurs in the weeks or months following
solid-organ or hematopoietic stem cell transplantation. It is triggered by the use of
immunosuppressive medications that impair cytotoxic T-cell immunosurveillance, which
leads to the unchecked replication of oncogenic viral pathogens in transplanted donor tissue or
host cells. Most cases (>95%) are caused by the reactivation of EBV, a human herpesvirus
that establishes life-long latent infection in B and T lymphocytes following initial infection.

EBV is an oncogenic virus because it produces proteins (eg, latent membrane protein) that lead
to B-cell proliferation and generates prosurvival nuclear transcription factors (EBNA) that lead
to B-cell immortalization. Subsequent cell spread through the reticuloendothelial system
results in lymphadenopathy, hepatosplenomegaly, bone marrow infiltration (eg, leukopenia,
anemia), and/or B-symptoms. Diagnosis is often suspected due to a markedly elevated EBV
titer, but biopsy may be required for confirmation.
*Hemophilic arthropathy

Hemophilia A & B
• X-linked recessive
Inheritance
• Delayed/prolonged bleeding after mild trauma
o Hemarthrosis, intramuscular hematomas
Clinical o Gastrointestinal or genitourinary tract bleeding
features o Intracranial hemorrhage
• Complications: hemophilic arthropathy

• ↑ Activated PTT
• Normal platelet count & PT
Laboratory
• Absent or ↓ factor VIII (hemophilia A) or factor IX
findings
(hemophilia B) activity

• Factor replacement
Treatment • Desmopressin for mild hemophilia A

This patient's history of excessive bleeding after a dental procedure is concerning for hemophilia, an X-linked
recessive bleeding disorder caused by deficiency of Factor VIII (hemophilia A) or Factor IX (hemophilia
B). Disease severity varies based on degree of factor activity. Often, patients present with severe joint swelling
and pain following little or no trauma due to spontaneous bleeding into a joint (hemarthrosis). Recurrent
hemarthroses, as seen in this patient, can result in long-term complications such as hemophilic arthropathy.
Hemophilic arthropathy refers to joint damage caused by intra-articular bleeding. Hemosiderin deposition
within the joint triggers synovial inflammation, which leads to fibrosis and destruction of cartilage and
bone. Chronic, worsening joint pain and swelling are accompanied by limited mobility on
examination. Although severe hemophilic arthropathy may be visible on x-ray, MRI allows for earlier detection
and characterization of the degree of joint damage. Early prophylaxis with factor concentrates can
significantly reduce the risk of developing arthropathy.
*Neonatal polycythemia

Neonatal polycythemia
• Hematocrit >65% in term infants
Definition
• Increased erythropoiesis from intrauterine hypoxia: maternal diabetes,
hypertension, or smoking; intrauterine growth restriction
• Erythrocyte transfusion: delayed cord clamping, twin-twin transfusion
Causes
• Genetic/metabolic disease: hypothyroidism/hyperthyroidism, genetic
trisomy (13, 18, 21)

• Asymptomatic (most common)


• Ruddy skin
• Hypoglycemia, hyperbilirubinemia
Clinical
• Respiratory distress, cyanosis, apnea
presentation
• Irritability, jitteriness
• Abdominal distension

• Intravenous fluids
• Glucose
Treatment
• Partial exchange transfusion

Polycythemia is defined as a hematocrit level >65% (>2 standard deviations above the mean)
in term neonates. Causes include excessive transfusion (eg, delayed umbilical cord clamping,
twin-to-twin transfusion syndrome), intrauterine hypoxia (eg, poor placentation due to
maternal diabetes or hypertension, smoking), or genetic conditions (eg, trisomy 21). In this
patient, maternal preeclampsia caused placental dysfunction, resulting in intrauterine hypoxia
and growth restriction.

Other than a ruddy/plethoric appearance, most neonates with polycythemia are


asymptomatic. As the hematocrit level rises due to postbirth fluid shifts, the increased blood
viscosity may cause decreased blood flow to organs. Common consequences include lethargy,
irritability, and jitteriness. Severe symptoms include respiratory distress, tachypnea, poor
feeding, and cyanosis. The increased red blood cell mass can lead to hypoglycemia and
hypocalcemia due to increased cellular uptake. Treatment of polycythemia is hydration by oral
feeds or glucose-containing parenteral fluids.
*Immune thrombocytopenia (ITP)

Immune thrombocytopenic purpura


• Commonly acquired form of thrombocytopenia
• Autoantibody formation
Epidemiology • Often recent viral infection or comorbidity (eg, HIV, HCV,
CLL)

• Frequently asymptomatic
• Mucocutaneous bleeding (eg, menorrhagia, epistaxis)
Manifestations • Ecchymoses, petechiae, purpura
• Severe hemorrhage is rare

• Diagnosis of exclusion
• Normal coagulation tests, platelet morphology (peripheral
Diagnosis smear)
• HIV, HCV testing (commonly induces ITP)

• Observation if no bleeding & platelets >30,000/mm3


Treatment • Corticosteroids if platelets <30,000/mm3
• IVIG & platelet transfusion if hemorrhage

CLL = chronic lymphocytic leukemia; HCV = hepatitis C virus; ITP = immune


thrombocytopenic purpura; IVIG = intravenous immunoglobulin.

Immune thrombocytopenia (ITP) is an acquired disorder marked by the formation of


autoantibodies to platelet surface glycoproteins, which increases platelet clearance in the spleen
and inhibits megakaryocytosis. In ITP, platelet counts fall to <100,000/mm3, leading to
prolonged skin/mucosal bleeding (eg, menstrual, nasal), ecchymosis, and petechiae.
ITP is a diagnosis of exclusion; therefore, other causes of thrombocytopenia (eg, drug-induced
thrombocytopenia, liver disease, hypersplenism, microangiopathic disorders, bone marrow
abnormalities) must be ruled out. In ITP, leukocyte count, erythrocyte count, and coagulation
studies are normal; peripheral blood smear usually shows normal platelet morphology (both
normal-sized and large platelets may be seen due to increased platelet turnover).

ITP is categorized as either primary (no discernible cause) or secondary (due to an underlying
condition). HIV and hepatitis C are the most common secondary causes, so testing for these 2
conditions is generally performed. Additional testing is warranted when specific signs or
symptoms of another disorder are present. This patient's myalgias and arthralgias raise concern
for an autoimmune disease (eg, systemic lupus erythematosus); therefore, antinuclear
antibody testing is advisable.
(Choice B) Ascorbic acid (vitamin C) deficiency causes impaired collagen synthesis and can
lead to ecchymosis and palpable purpura. However, most cases occur in severely malnourished
individuals or those who use alcohol or illicit drugs.

(Choice C) Factor VIII deficiency can cause similar bleeding symptoms but is associated with
prolonged PTT. Hemophilia A, an X-linked disorder (seen only in males), causes severe factor
VIII deficiency.

(Choice D) Hemoglobin electrophoresis is used to diagnose thalassemia, a genetic hemoglobin


disorder associated with anemia, microcytosis, and target cells. Hemoglobin disorders do not
usually cause thrombocytopenia.

(Choice E) Platelet aggregation testing evaluates for rare disorders of platelet function and is
generally indicated in those who have normal platelet count and prolonged bleeding times. This
patient's thrombocytopenia makes ITP far more likely; platelet aggregation testing would not be
indicated.

(Choice F) Von Willebrand disease (vWD) is associated with impaired platelet aggregation,
which occasionally causes bruising and prolonged mucosal bleeding. However, platelet count
and PT are normal; PTT is sometimes prolonged due to increased destruction of factor VIII.

Educational objective:
Immune thrombocytopenia (ITP) is an autoimmune disorder associated with isolated
thrombocytopenia. Although most cases are primary, a minority of individuals have ITP due to
an underlying condition (eg, HIV, hepatitis C). Testing for HIV and hepatitis C virus is
recommended for all patients with ITP; additional testing is needed when signs and symptoms
of other disorders (eg, systemic lupus erythematosus) are present.
*PET scan & Hodgkin lymphoma
This young patient with hilar and mediastinal lymphadenopathy and B symptoms (eg, fevers,
weight loss) likely has Hodgkin lymphoma, a lymphoid neoplasm with peak incidence in early
adulthood and in those age >60. Most cases come to clinical attention due to painless
lymphadenopathy in the cervical and/or supraclavicular chains or after chest radiograph shows
a mediastinal mass. B symptoms are present in approximately 40% of cases.

The diagnosis is generally made by excisional lymph node biopsy; those with no clear evidence
of peripheral lymphadenopathy on physical examination usually undergo imaging to identify a
potential biopsy site. Positron emission tomography (PET) scan with 18-
fluorodeoxyglucose (FDG), an analog of glucose, is often the imaging test of
choice. Neoplastic cells (eg, supraclavicular and mediastinal lymph nodes in this patient) have
a high metabolic rate and readily take up radiotracer on PET scan. However, radiotracer also
pools in healthy organs with high glucose requirements such as the brain, kidneys, and liver;
excretion of radiotracer also leads to enhancement throughout the urinary collecting system.

(Choice A) Glioblastoma multiforme would cause focal FDG uptake in the brain lesion and
does not generally lead to cough or spread to the hilar or mediastinal lymph nodes.

(Choice C) Renal cell carcinoma usually metastasizes to the pulmonary parenchyma, which
would be seen on PET scan.

(Choice D) Small cell lung cancer frequently spreads to the ipsilateral hilar and mediastinal
lymph nodes; however, cases are rare in young adults, and a pulmonary mass is generally seen
on imaging.

(Choice E) Transitional cell carcinoma would cause increased uptake of tracer in the urinary
bladder, but abdominal lymphadenopathy would be seen rather than hilar or mediastinal
lymphadenopathy.

Educational objective:
A positron emission tomography scan using a radiolabeled glucose analog can identify areas of
enhanced metabolic activity (eg, cancer, infection). However, the tracer will also pool in
healthy organs with high glucose requirements such as the brain, liver, and kidneys; uptake will
also be seen in the urinary collecting system due to radiotracer excretion.
*notes on DVT case evaluation
This patient has deep venous thrombosis (DVT) of the popliteal and femoral veins. Risk
factors for venous thromboembolism (VTE) include inherited (eg, factor V Leiden,
prothrombin gene mutation, protein C deficiency) and acquired (eg, immobilization, surgery,
malignancy, medications) factors. Initial evaluation is directed at identifying major causes of
VTE or factors that can increase the risks of anticoagulation therapy; evaluation should include
careful history, physical examination, and limited diagnostic testing (eg, blood counts, serum
chemistries, coagulation studies, fecal occult blood testing, chest x-ray).

In the absence of any clear provoking factors (eg, recent surgery, immobilization), patients with
a first episode of VTE, such as this man, should be referred for age-appropriate cancer
screening (eg, colonoscopy) to evaluate for malignancy as a potential risk factor for VTE. In
addition, any suspicious symptoms of malignancy (eg, weight loss, unexplained pain) warrant
consideration for more extensive cancer evaluation (eg, CT scan of the abdomen) (Choice B).

(Choice D) Testing for an inherited thrombophilia (eg, factor V Leiden; deficiency of protein
C, protein S, or antithrombin III) is generally considered only if the patient's history is
suggestive of a genetic predisposition (eg, recurrent VTE; multiple or unusual sites of
thrombosis; family history of VTE, particularly at a young age).
*paroxysmal nocturnal hemoglobinuria (PNH)

Clinical features of paroxysmal nocturnal hemoglobinuria


• Hemolysis → fatigue

Clinical • Cytopenias (impaired hematopoiesis)


manifestations
• Venous thrombosis (intraabdominal, cerebral veins)

• Complete blood count (hypoplastic/aplastic anemia,


thrombocytopenia, leukopenia)
• Elevated lactate dehydrogenase and low haptoglobin
(hemolysis)
Workup
• Indirect hyperbilirubinemia
• Urinalysis (hemoglobinuria)
• Flow cytometry (absence of CD55 & CD59)

• Iron & folate supplementation


• Eculizumab (monoclonal antibody that inhibits
Treatment
complement activation)

The most likely diagnosis in this case is paroxysmal nocturnal hemoglobinuria (PNH). PNH is an clonal
stem cell disorder characterized by intravascular and extravascular hemolysis and hemoglobinuria. The
disease is due to an acquired genetic defect that results in lack of the glycosylphosphatidylinositol anchor,
which connects proteins, including CD55 and CD59, to the cell surface. These proteins normally inhibit the
activation of complement on red blood cells, but their absence allows the complement membrane attack
complex to form and results in hemolysis.

On average, patients have manifestations in the third decade of life resulting from one of the following:

• Hemolysis leading to hemoglobinuria


• Cytopenias - fatigue and dyspnea from anemia
• Hypercoagulable state (eg, portal vein thrombosis, hepatic vein thrombosis)

This patient's anemia and low haptoglobin accompanied by elevated bilirubin and lactate dehydrogenase (LDH)
are all consistent with intravascular hemolysis. The key to the diagnosis is the patient's hepatic vein thrombosis
in conjunction with these signs of hemolysis. The pathophysiology of thrombosis is unclear but may be related
to release of procoagulant molecules as a result of hemolysis and possibly to changes in complement
levels. Unusual sites of thrombosis may be noted, and cerebral and intraabdominal vein involvement is
classic. Abdominal pain may be due to severe hemolysis, thrombosis, or effects on smooth muscle.

Flow cytometry tests are used to confirm the diagnosis by assessing for the absence of the CD55 and CD59
proteins on the surface of the red blood cells.
*Drug-induced neutropenia

Drug-induced neutropenia
• Methimazole, propylthiouracil
• Trimethoprim-sulfamethoxazole
Common triggers • Clozapine
• Sulfasalazine

• Immune mediated: drug or metabolite binds to


neutrophils & triggers a cell-mediated or humoral
Mechanism response
• Bone marrow toxicity: drug or metabolite directly or
indirectly damages granulocyte precursors

• Usually within 3 months of drug exposure


• Oropharyngeal ulceration ± fever
Manifestations
• Other cell lines unaffected

• Discontinue inciting medication


• Antibiotics in setting of treatable infection
Management
• Consider granulocyte colony-stimulating factor

Patients with alcohol use disorder and cirrhosis may develop leukopenia due to a variety of
conditions. Two of the most common etiologies are alcohol-induced bone marrow toxicity and
vitamin B12/folate deficiency. However, these causes tend to be slow in onset and to cause mild
neutropenia (eg, absolute neutrophil count nadir >1000/mm3) (Choices A and E). Given the
recent initiation of trimethoprim-sulfamethoxazole for spontaneous bacterial peritonitis
prophylaxis, the most likely etiology of this patient's severe neutropenia (absolute neutrophil
count <500/mm3) is drug-induced neutropenia.

Drug-induced neutropenia is generally triggered by direct bone marrow toxicity in a dose-


dependent manner (eg, chemotherapy) or an idiosyncratic reaction due to the attachment of a
drug or drug metabolite to the neutrophil surface, which triggers a cell-mediated or humoral
immune response. Most cases present with oropharyngeal ulcerations and fever. The other
cell lines (eg, erythrocytes, platelets) are generally normal unless a different condition is
altering rates of production or destruction; in this case, the patient likely has mild macrocytic
anemia due to alcohol toxicity or vitamin B12/folate deficiency and mild thrombocytopenia
due to hypersplenism.

The next best step would be to stop trimethoprim-sulfamethoxazole, which should resolve the
neutropenia within 1-3 weeks. Administering granulocyte colony-stimulating factor may
speed resolution.
(Choice C) Although pancytopenia (due to bone marrow suppression) can be caused by sepsis,
this patient does not have clinical evidence of sepsis (eg, hypotension, tachycardia). In
addition, sepsis does not typically cause severe neutropenia out of proportion to other blood cell
counts.

(Choice D) Hypersplenism can cause thrombocytopenia but does not usually cause
leukopenia. In addition, severe neutropenia (absolute neutrophil count <500/mm3) would be
very uncommon.

Educational objective:
Trimethoprim-sulfamethoxazole can cause drug-induced neutropenia, which typically presents
with sudden, severe neutropenia, oropharyngeal ulcers, and fever. The other cell lines are
usually unaffected, but they may be reduced due to other ongoing conditions.
*pyridoxine-dependent acquired sideroblastic anemia
The most likely diagnosis in this patient is acquired sideroblastic anemia. Sideroblastic
anemias consist of different types of anemias characterized by defects in heme synthesis (eg, in
mitochondria) and ring sideroblasts (nucleated erythroblasts with mitochondrial iron granules
surrounding the nucleus). They can be congenital (presenting in children or adolescents) or
acquired (most common). Although acquired sideroblastic anemias may be related to clonal
abnormalities (eg, myelodysplastic syndrome), they are frequently due to reversible etiologies
(eg, alcohol, malnutrition, medications) that lead to pyridoxine-dependent impairment in the
early steps of protoporphyrin synthesis within mitochondria.

This patient is on isoniazid (INH), a well-known pyridoxine (vitamin B6) antagonist that can
cause acquired sideroblastic anemia. It frequently manifests as microcytic hypochromic
anemia, simulating iron deficiency anemia.

• As can sometimes be seen with iron deficiency anemia, usually 2 groups of red blood
cells (RBCs) can be demonstrated on microscopy: hypochromic and normochromic
("dimorphic" RBC population), likely reflecting the defective heme synthesis.
• Unlike iron deficiency anemia, iron studies typically reveal increased serum iron
concentration and normal or decreased total iron binding capacity due to mitochondrial
mishandling of iron (explaining the mitochondrial iron granules seen in ring
sideroblasts); in contrast, iron deficiency anemia is characterized by decreased serum iron
concentration and high total iron binding capacity.

In this patient with an easily identifiable cause of vitamin B6 deficiency (ie, INH),
administration of pyridoxine is the most reasonable next step. Bone marrow sampling can
confirm the diagnosis by demonstrating ring sideroblasts (a specific finding in sideroblastic
anemia), but it is not necessary in this patient (Choice A).

(Choices C and D) Folic acid or the combination of folic acid with vitamin B12 is employed in
patients with macrocytic anemia (which would cause an elevated mean corpuscular volume).

Educational objective:
Sideroblastic anemia usually manifests as microcytic/hypochromic anemia simulating iron-
deficiency anemia, but iron studies reveal elevated serum iron level and normal or decreased
total iron binding capacity. In patients with an identifiable cause of vitamin B6 deficiency
(alcohol, medications), the administration of pyridoxine can easily correct the problem
*Delayed hemolytic transfusion reaction

Delayed hemolytic transfusion reaction


• Anamnestic antibody response: antibody

Pathogenesis production against minor RBC antigen to


which patient was previously* exposed

• Onset >24 hr to a month after transfusion


• Often asymptomatic
Clinical • Fatigue, dyspnea, tachycardia
findings • Jaundice
• Low-grade fever

• Hemolytic anemia
o ↑ Indirect bilirubin, LDH, reticulocyte

Laboratory count
o ↓ Hemoglobin, haptoglobin
findings
• New positive direct antiglobulin (Coombs)
test

• Supportive (eg, fluids)


Management
• Review transfusion history & prior antibody
screens
Prevention • Transfuse when necessary with extended-
antigen cross-matched blood

*Prior transfusion (eg, sickle cell disease), pregnancy, or


transplant.

LDH = lactate dehydrogenase; RBC = red blood cell.

This patient with sickle cell disease has symptomatic anemia, indirect hyperbilirubinemia, and
low-grade fever days after receiving a blood transfusion. These findings are concerning for a
delayed hemolytic transfusion reaction (DHTR).

DHTRs occur in patients who have been previously exposed to a foreign red blood cell (RBC)
antigen (eg, prior transfusion, pregnancy). The antigen is minor (non-ABO) and typically
causes a low, often undetectable antibody response after initial exposure (ie, alloimmunization,
similar to that seen in a Rhesus-negative primigravida with a Rhesus-positive fetus). However,
on subsequent exposure via repeat transfusion, an anamnestic antibody response occurs in
which memory B cells rapidly produce more antibodies; these antibodies bind to donor RBCs,
causing extravascular hemolysis.

Sickle cell disease patients are at high risk of DHTRs due to frequent exposure to minor
antigens via repeated transfusions. Onset of DHTR is >24 hours up to a month after
transfusion. Most patients are asymptomatic. However, sickle cell disease patients are more
likely to have symptomatic, severe anemia due to sickle cell-related intravascular hemolysis in
addition to DHTR-related extravascular hemolysis. Symptomatic DHTR causes fatigue,
dyspnea, jaundice, and low-grade fever, with laboratory evidence of hemolysis, including
elevated indirect bilirubin, lactate dehydrogenase, and reticulocyte count as well as decreased
haptoglobin. Because these results may be present at baseline in sickle cell disease patients, a
new positive direct antiglobulin (Coombs) test is important for diagnosing DHTR.

Management is supportive. Prevention includes thorough review of prior antibody screens and
use of extended-antigen cross-matched blood when transfusion is required.

(Choice B) Patients with sickle cell disease often have iron overload, not deficiency, due to
frequent transfusions. In addition, iron deficiency anemia is associated with a low reticulocyte
count.

(Choice C) Splenic sequestration in young children with sickle cell disease causes
splenomegaly and a rapid drop in hemoglobin with compensatory reticulocytosis. This patient's
age is inconsistent with this diagnosis.

(Choices D and E) Transfusion-transmitted parvovirus infection in sickle cell disease can


cause a transient aplastic crisis with severe, acute anemia. However, reticulocyte count is low
due to decreased erythropoiesis. Transfusion-transmitted viral hepatitis causing jaundice and
hepatomegaly is rare due to blood donor screening, and it does not cause anemia.

Educational objective:
Sickle cell disease patients are at high risk of delayed hemolytic transfusion reactions due to
alloimmunization from frequent transfusions. Reactions develop >24 hours after transfusion
and can cause hemolytic anemia. Diagnosis is confirmed with a newly positive Coombs test.
*Aplastic crisis

Acute, severe anemia in sickle cell disease


Cause Reticulocytes Key features
• Transient arrest of erythropoiesis
Aplastic crisis ↓ • Secondary to infection (eg, parvovirus B19)

• Splenic vasoocclusion → rapidly enlarging


Splenic sequestration spleen

crisis • Occurs in children prior to autosplenectomy

Sickle cell disease (SCD) is characterized by chronic anemia with compensatory


reticulocytosis. Patients with SCD have increased red blood cell destruction at baseline and are
at risk for acutely worsening, severe anemia. An acute drop in hemoglobin accompanied by a
low reticulocyte count (<1%) without splenomegaly is consistent with an aplastic crisis,
which is characterized by a sudden halt in red blood cell production (erythropoiesis). Anemia is
typically severe (<6 g/dL) and patients may have pallor, fatigue, and a functional systolic
murmur (due to hyperdynamic blood flow). White blood cell and platelet counts are typically
normal.

The most common cause of an aplastic crisis is parvovirus B19, which infects and destroys
erythrocyte precursors. Aplastic crises generally present before age 15 as parvovirus outbreaks
are most common in school-age children. Although erythema infectiosum (ie, fever, rash) is the
classic presentation in children, parvovirus B19 infection can result in an aplastic crisis with no
preceding viral symptoms. Blood transfusions are the mainstay of treatment.
*Notes on Hodgkin lymphoma (HL)

This patient's lymph node biopsy reveals a mixed inflammatory infiltrate with a Reed-
Sternberg (RS) cell—a large cell with 2 nucleoli in separate nuclear lobes, creating an
"owl's eye" appearance. RS cells are pathognomonic for Hodgkin lymphoma (HL), a
germinal B-cell malignancy that has peak prevalence during adolescence (age 15-19) and in
those age >50.

Adults with HL most often have:

• Painless lymphadenopathy (LAD) that is typically nontender, rubbery, and firm; it


most often occurs in the cervical and axillary areas.
• A mediastinal mass that is often asymptomatic at diagnosis but may cause chest pain,
cough, or shortness of breath.
• B-symptoms that may include fever, night sweats, and weight loss.
Hepatomegaly and/or splenomegaly (reflecting HL involvement of these organs) can also be
seen. Other, less common symptoms in adults include generalized pruritus and severe,
generalized pain following alcohol ingestion.

Complete blood count can be normal (or near normal) in early disease, but many patients with
HL have leukocytosis, eosinophilia, and/or ≥1 cytopenia (eg, anemia). Lactate dehydrogenase
and inflammatory markers (eg, ESR) are often quite elevated due to increased cancer cell
turnover or tissue destruction. Diagnosis is confirmed when excisional lymph node biopsy
reveals RS cells (usually 0.1%-10% of total cells) admixed with an inflammatory cell
infiltrate. A PET scan, which uses a radiolabeled glucose analogue to identify areas of
increased metabolic activity (eg, tumors), is required for staging.
*Notes on G6PD deficiency

Medications that often trigger hemolysis in G6PD deficiency*


Diaminodiphenyl sulfone (dapsone)

Isobutyl nitrite

Avoid Nitrofurantoin

Rasburicase

Primaquine
Acetaminophen

Acetylsalicylic acid (aspirin)

Chloramphenicol

Chloroquine

Colchicine

Diphenhydramine (Benadryl)

Use with caution Glyburide

Isoniazid

L-Dopa

Quinine

Sulfamethoxazole

Trimethoprim

Vitamin K
*This list is not exhaustive; it includes medications commonly used in the clinical setting.

G6PD = glucose-6-phosphate dehydrogenase.

This patient's dysuria, lower abdominal pain, urinary incontinence, and urinalysis findings
suggest a urinary tract infection, for which antibiotic therapy is indicated. However, she likely
has glucose-6-phosphate dehydrogenase (G6PD) deficiency given her prior hospitalization
for hemolytic anemia with low baseline G6PD levels. G6PD deficiency is an inherited
disorder associated with episodes of hemolysis during oxidative stress. In these patients,
oxidative medications such as nitrofurantoin are particularly likely to trigger hemolysis and
must be avoided. Other common oxidative triggers include infections (leukocytes release
highly oxidative reactive oxygen metabolites) and fava beans (metabolized into highly oxidative
compounds).

G6PD deficiency is X-linked and primarily affects male patients. However, female patients
who have homozygous mutations or skewed lyonization of heterozygous mutations
(overexpression of erythrocytes with the mutated X-chromosome) can also express the disorder.

(Choices A, B, C, and D) Penicillin-class medications (eg, amoxicillin) and cephalosporins


(eg, cefixime, cefpodoxime, cephalexin) are generally safe to use in patients with G6PD
deficiency.

Educational objective:
Glucose-6-phosphate dehydrogenase deficiency is an X-linked disorder associated with
episodes of hemolytic anemia during oxidative stress. In order to prevent hemolysis, oxidative
medications such as nitrofurantoin should be avoided
*Chronic lymphocytic leukemia
Chronic lymphocytic leukemia
• Lymphadenopathy (cervical, supraclavicular, axillary)
• Hepatosplenomegaly
Clinical features • Mild thrombocytopenia & anemia
• Often asymptomatic

• Severe lymphocytosis & smudge cells


• Flow cytometry
Diagnosis
• Lymph node & bone marrow biopsy not generally needed

• Median survival 10 years


• Worse prognosis with:
o Multiple chain lymphadenopathy
Prognosis
o Hepatosplenomegaly
o Anemia & thrombocytopenia

• Infection
• Autoimmune hemolytic anemia
Complications
• Secondary malignancies (eg, Richter syndrome)

This patient with a fever, productive cough, and right lower lobe infiltrate likely has bacterial
pneumonia. Most bacterial infections cause a mild to moderate leukocytosis with a
predominance of neutrophils. This patient's complete blood count demonstrates a dramatic
leukocytosis, primarily with lymphocytes. In an elderly patient, severe lymphocytosis
combined with hepatosplenomegaly, lymphadenopathy, and bicytopenia (anemia,
thrombocytopenia) strongly suggests underlying chronic lymphocytic leukemia
(CLL). Although patients with CLL have high white blood cell counts, their immune systems
are abnormal; hypogammaglobulinemia and defects in cell signaling increase the risk and
severity of infections (a major complication of CLL).

CLL is diagnosed by flow cytometry (showing a clonality of mature B cells).


*different blood smear patterns

This patient has single, round, blue inclusions within red blood cells (RBCs) on peripheral smear, consistent
with Howell-Jolly bodies. These bodies are retained RBC nuclear remnants that are typically removed by the
spleen. The presence of Howell-Jolly bodies usually indicates either physical absence of the spleen (ie,
asplenia) due to congenital absence or surgical removal or functional hyposplenism due to splenic
autoinfarction (eg, sickle cell disease), infiltrative disorders of the spleen (eg, sarcoidosis), or splenic congestion
(eg, thrombosis).

Both asplenia and functional hyposplenism increase the risk of sepsis because the spleen is normally
responsible for clearing bacteria, particularly encapsulated organisms. However, splenectomy may be indicated
in cases of chronic hemolysis.
(Choice A) In glucose-6-
phosphate dehydrogenase deficiency disease, hemoglobin becomes oxidized and precipitates into insoluble
Heinz bodies. Peripheral smear shows bite cells, and Heinz bodies appear as peripheral blue inclusions with
special staining (eg, crystal violet), not with Wright staining as in this patient.

Heinz bodies, which appear as small inclusions within an erythrocyte, are aggregates of denatured hemoglobin
and are commonly seen in patients with hemolysis due to glucose-6-phosphate dehydrogenase deficiency and
thalassemia. When phagocytes extract this rigid precipitate, they form characteristic bite cells
(Choice B) The inability to produce alpha (alpha thalassemia) or beta (beta thalassemia) globin chains causes
hypochromic, microcytic RBCs and target cells (due to decreased hemoglobin production), which can be seen
on peripheral smear.
(Choice C) Mechanical RBC damage occurs when RBCs shear within the vascular system. This occurs in
hemolytic uremic syndrome, disseminated intravascular coagulation, and thrombotic thrombocytopenic
purpura. Schistocytes (fragmented RBCs) are seen on peripheral blood smear.
(Choice E) Precipitation of ribosomal ribonucleic acid is seen in patients with lead poisoning. In contrast to
the single, blue, peripherally-located inclusion characteristic of a Howell-Jolly body, basophilic stippling
appears as multiple, scattered, blue granules within the RBC cytoplasm.

Educational objective:
Howell-Jolly bodies are nuclear remnants within red blood cells typically removed by the spleen. Their
presence strongly suggests asplenia or functional hyposplenism
Choice A) Burr cells (also known as echinocytes) are spiculated appearing red blood cells (RBCs) with
serrated edges that can be seen in liver disease and end-stage renal disease (ESRD). However, burr cells are
less common in the acute kidney injury seen in SRC.
(Choice D) Spur
cells (acanthocytes) are RBCs with irregularly sized and spaced projections that are most commonly seen in
liver disease.
*hemoglobin Barts
Alpha thalassemia
Genotype Disorder Clinical features
1 gene loss
Alpha thalassemia minima Asymptomatic, silent carrier
(αα/α−)
2 gene loss
Alpha thalassemia minor Mild microcytic anemia
(αα/−−) or (α−/α−)
3 gene loss
Hemoglobin H disease Chronic hemolytic anemia
(α−/−−)
Hydrops fetalis: anasarca,
4 gene loss Alpha thalassemia major,
high-output cardiac failure,
(−−/−−) hemoglobin Barts disease
death in utero

Normal hemoglobin is a heterotetramer composed of 4 chains:

• 2 alpha and 2 beta chains in children and adults (adult hemoglobin), or


• 2 alpha and 2 gamma chains in fetuses and infants (fetal hemoglobin)

In patients with alpha-thalassemia, ≥1 alpha genes are deleted. When 3 genes are lost (α-/--), a
condition known as hemoglobin H disease, the nonalpha chains accumulate into homotetramers;
in children and adults, these homotetramers are composed of beta chains (hemoglobin
H) whereas, in fetuses and infants, they are composed of gamma chains (hemoglobin Barts), as
seen in this patient.

Hemoglobin H disease usually presents in infancy with chronic hemolysis due to a shorter red
blood cell (RBC) lifespan and increased splenic sequestration. Elevated erythrocyte count
and reticulocytosis occur as the bone marrow attempts to replenish hemolyzed
RBCs. However, RBCs are microcytic (mean corpuscular volume <80 µm3) and hypochromic
because hemoglobin accounts for the majority of RBC volume. Characteristic findings on
peripheral smear include abundant target cells as a result of the reduced RBC volume.

The presence of gamma tetramers (hemoglobin Barts) on hemoglobin electrophoresis should


raise suspicion for alpha-thalassemia; it is classically associated with hydrops fetalis, the most
severe form of alpha-thalassemia, which is due to 4 alpha gene loss (--/--) and typically leads to
death in utero.

(Choice A) Increased destruction of RBCs seen in microangiopathic hemolytic anemia results


in a decreased erythrocyte count and schistocytes on peripheral smear.

(Choice B) Patients with hereditary spherocytosis have a defective RBC membrane protein that
results in spherocytes on peripheral smear. Erythrocyte count is decreased due to destruction of
spherocytes in the spleen.
(Choice C) Secondary erythrocytosis is characterized by an increase in immature erythrocytes
(polychromasia) in response to hypoxia or tumor. Hemoglobin electrophoresis would not show
gamma tetramers.

(Choice D) Iron deficiency causes a microcytic, hypochromic anemia similar to


thalassemia. However, decreased RBC production due to inadequate iron stores results in a low
erythrocyte count and pencil cells on peripheral smear.

Educational objective:
The presence of gamma tetramers (hemoglobin Barts) on hemoglobin analysis is consistent with
alpha-thalassemia. Laboratory studies typically reveal a microcytic anemia, an elevated
erythrocyte count, and target cells on peripheral smear.
*Glucose-6-phosphate dehydrogenase deficiency

Glucose-6-phosphate dehydrogenase deficiency


• Hemolytic anemia due to oxidative stress
(infection, sulfa drugs, fava beans)
Epidemiology • X-linked: Asian, African, or Middle Eastern
descent

• Pallor & fatigue


• Dark urine, jaundice & icterus
Manifestations
• Abdominal/back pain

• Hemolysis: ↓ hemoglobin, ↓ haptoglobin,


↑ bilirubin & LDH, ↑ reticulocytes
• Peripheral smear: bite cells & Heinz bodies
Laboratory
• Negative Coombs test
findings
• ↓ G6PD activity level (may be normal during
attack)

• Remove or treat responsible agent/condition


Management • Provide supportive care

G6PD = glucose-6-phosphate dehydrogenase; LDH = lactate dehydrogenase.

This patient developed hemolytic anemia after receiving trimethoprim-sulfamethoxazole, raising strong
suspicion for glucose-6-phosphate dehydrogenase (G6PD) deficiency. This X-linked, inherited disorder is
seen in ~10% of black men but also occurs in those of Mediterranean, Asian, and Middle Eastern
descent. Patients are generally asymptomatic but develop hemolysis when exposed to oxidative stressors such
as infection, medications (eg, trimethoprim-sulfamethoxazole, nitrofurantoin, primaquine), or certain foods
(eg, fava beans).

Hemolysis generally manifests with pallor and fatigue (due to anemia), jaundice (due to elevated bilirubin) and
dark urine (due to hemoglobinuria), and back/abdominal pain. Laboratory results will show low hemoglobin,
elevated indirect bilirubin, elevated lactate dehydrogenase, and decreased haptoglobin. Peripheral blood
smear typically reveals Heinz bodies (oxidized/precipitated hemoglobin) and bite cells (bite-shaped membrane
defects due to removal of Heinz bodies by splenic macrophages). Coombs testing will be negative, unlike
antibody-mediated forms of hemolysis.

G6PD activity testing can be normal (low sensitivity) in acute hemolytic episodes because older erythrocytes
(lowest G6PD activity) have been hemolyzed and young erythrocytes/reticulocytes (higher G6PD activity) are
disproportionally elevated. Therefore, in such settings, this test cannot be used to rule out G6PD deficiency;
repeat testing should be done at 3 months to confirm the diagnosis in those whose initial test was negative. No
intervention is generally required other than removing or treating the inciting oxidative stress.
*HIT type 2

This patient was initiated on low molecular weight heparin (enoxaparin) due to an acute deep
venous thrombosis. A week later, laboratory studies show a significant reduction in platelet
count, suggesting type 2 heparin-induced thrombocytopenia (HIT). A drop in platelets by
>50% or a new thrombus within 5-10 days of heparin initiation should raise suspicion for HIT
(HIT may manifest sooner in patients previously exposed to heparin).
Heparin induces a conformational change to a platelet surface protein (platelet factor 4 [PF4]),
which creates a neoantigen. In patients with HIT, the immune system responds by forming an
IgG autoantibody (HIT antibody). HIT antibodies then coat the surface of platelets, causing:

• Thrombocytopenia - the reticuloendothelial system (largely the spleen) removes HIT


antibody-coated platelets, causing mild to moderate thrombocytopenia (mean nadirs of
60,000/mm3). Severe thrombocytopenia (<20,000/mm3) is uncommon.
• Thrombus - HIT antibodies activate platelets, resulting in platelet aggregation and the
release of procoagulant factors. This puts patients at very high risk (as high as 50%) for
arterial and venous thrombus. Venous thrombosis in the leg veins, cardiac vessels, and
skin (at the site of heparin injection) is common. Arterial thrombosis may occur in the
heart, limbs, or central nervous system.

If HIT is suspected, all heparin products should be discontinued immediately and


anticoagulation should be initiated with a non-heparin medication (eg, argatroban,
fondaparinux). Diagnosis is confirmed by functional assay (eg, serotonin-release assay [gold
standard]) or high-titer immunoassay of the blood.
*HL prognosis

This patient with Hodgkin lymphoma (HL) was treated with chemotherapy and radiation therapy, which
cures most cases. However, patients who undergo treatment for HL are at increased risk of death compared to
an age-matched population due to the following:

• HL relapse (or recurrence): The leading cause of death for the first 8-10 years after treatment is HL
relapse. Relapses primarily occur in the first 2 years, but risk remains mildly elevated for life.
• Second malignancy: The leading cause of death >8-10 years after treatment is second malignancy, and
this risk increases with time. Breast and lung malignancies are most common due to their proximity
to the mediastinum, which is often irradiated as part of HL treatment. Hematologic malignancy (eg,
non-Hodgkin lymphoma) can also occur due to damage to hematologic stem cells from alkylating
chemotherapy; milder chemotherapy regimens (eg, doxorubicin, bleomycin, vinblastine, and
dacarbazine) are associated with less risk of second hematologic malignancy.
• Cardiovascular disease: The leading cause of death not due to malignancy is cardiovascular
disease. Most cardiovascular complications arise >10 years after treatment and are due to
chemotherapy- or radiation-related damage to the heart muscle, valves, and vasculature.

Patients who undergo treatment for HL typically receive intensive screening for HL relapse early after treatment
and for malignancy and cardiovascular disease as time progresses (eg, >5 years after treatment). Modifiable
risk factors for cancer (eg, quitting smoking) and heart disease (eg, control of hypertension and hyperlipidemia)
should be addressed to lessen the risk of early death from these conditions.

Educational objective:
Hodgkin lymphoma (HL) is usually curable with chemotherapy and radiation therapy. However, the risk of
death is significantly elevated in those who undergo treatment compared to an age-matched population. For the
first 8-10 years, most deaths are due to HL relapse; thereafter, most are due to a second malignancy and
cardiovascular disease.
*Physiologic anemia of infancy

Physiologic anemia of infancy


• ↑ Tissue oxygenation at birth → down-regulation of erythropoietin
Pathogenesis
• Asymptomatic term infant* age 2-3 months

Clinical features • Normocytic anemia (hemoglobin 9-11 g/dL)


• Low to normal reticulocyte count

• Reassurance
Treatment/prognosis • Anemia resolves with ↑ erythropoietin drive after age 3 months

*Preterm infants have a lower nadir at a younger age (ie, anemia of prematurity).

This patient had a fever warranting a complete blood count, which revealed anemia and
leukopenia. Repeat evaluation shows resolution of the leukopenia, which therefore was likely
due to transient bone marrow suppression in the setting of infection. However, this child's
anemia is persistent. In asymptomatic patients age 2-3 months with a normocytic anemia, the
most likely cause is physiologic anemia of infancy.
Physiologic anemia of infancy is a
transient decline in hemoglobin due to the down-regulation of erythropoietin triggered by a
sudden increase in tissue oxygenation at birth. Red blood cell production in the bone marrow
slows, leading to a physiologic hemoglobin nadir around 11 g/dL at age 2-3 months in term
infants (can be as low as 9 g/dL). Although the absolute value of the reticulocyte count may be
normal, a compensatory reticulocytosis in response to anemia is absent. After age 2-3 months,
the erythropoietin drive returns and hemoglobin levels rise.

Physiologic anemia of infancy is typically asymptomatic but may be identified on laboratory


evaluation for other purposes, as in this case. Findings suggestive of a pathologic cause of
anemia, such as hemoglobin <9 g/dL, signs of hemolysis (eg, elevated reticulocyte count), or
microcytosis, are not present.
*Drug-induced, immune-mediated hemolytic anemia
Drug-induced, immune-mediated hemolytic anemia
• Drug coats erythrocytes → IgG binding → splenic destruction of RBCs (extravascular
hemolysis)
Pathogenesis • Drug triggers immune complexes → complement-mediated destruction of RBCs
(intravascular hemolysis)

• Sudden onset (within hours of exposure)


• Anemia: fatigue, pallor, dyspnea
• Hemolysis: jaundice, dark urine, abdominal or back pain
o ↑ Reticulocytes, indirect bilirubin & LDH
Manifestations
o ↓ Haptoglobin
o Spherocytes on peripheral blood smear
o + Direct Coombs test (anti-IgG, anti-C3)

• Discontinue offending drug


• Transfusion (if severe)
Treatment
• ± Glucocorticoids, IVIG

IVIG = intravenous immunoglobulin; LDH = lactate dehydrogenase; RBCs = red blood cells.

This patient developed dark-colored urine, elevated bilirubin, and anemia shortly after receiving antibiotics,
raising strong suspicion for drug-induced hemolytic anemia. The most common triggers are nonsteroidal
anti-inflammatory drugs (eg, diclofenac), penicillins (eg, piperacillin), and cephalosporins (eg, ceftriaxone).

Although medications can trigger hemolysis through a variety of mechanisms (eg, oxidative injury,
complement-mediated injury), penicillins and cephalosporins typically cause hemolysis when they bind to the
erythrocyte surface, acting as a hapten that facilitates IgG attachment. Subsequently, the IgG-coated
erythrocytes are destroyed by splenic macrophages, leading to extravascular hemolysis.

Manifestations typically arise within hours or a few days of drug administration and include signs of anemia
(eg, tachycardia, dyspnea) and hemolysis (eg, elevated bilirubin, dark-colored urine). Severe hemolysis can
cause back or abdominal pain due to erythrocytosis in the vertebral bodies, liver, and spleen. Laboratory
evaluation usually shows increased bilirubin, lactate dehydrogenase, and reticulocytes; haptoglobin, which
binds free hemoglobin, will be very low.

The diagnosis of immune-mediated hemolysis is made with the direct antiglobulin (Coombs) test, which
detects IgG or C3 (a complement component) on the erythrocyte surface. Withdrawal of the offending drug
typically leads to resolution of symptoms within hours or days. However, blood transfusion or corticosteroids
are sometimes required in severe cases.
*Peutz-Jeghers syndrome

Peutz-Jeghers syndrome
• Autosomal dominant disorder
Etiology • Tumor suppressor gene mutation causes unregulated tissue growth

• Pigmented macules (eg, lips, buccal mucosa, palms/soles)


• ≥2 gastrointestinal hamartomatous polyps
o Abdominal pain due to obstruction or intussusception
o Anemia due to acute/chronic bleeding
Clinical features
o Rectal prolapse
• ↑ Cancer risk (eg, gastrointestinal, breast, genital tract)
• Positive family history

• Genetic testing
Diagnosis
• Annual anemia screening
Management • Cancer screening (eg, upper/lower endoscopy)

This patient with fatigue and pallor due to microcytic anemia has brown macules on the lips and
buccal mucosa that are characteristic of Peutz-Jeghers syndrome (PJS). PJS is an autosomal
dominant, familial polyposis syndrome caused by a mutation in a tumor suppressor gene,
leading to unregulated tissue growth, particularly in the gastrointestinal (GI) tract.

Classic findings include hyperpigmented mucocutaneous macules, which are most common
on the lips, buccal mucosa, and palms and soles. Characteristic hamartomatous polyps in the
GI tract may be asymptomatic or complicated by intestinal obstruction or iron deficiency
anemia due to occult bleeding, which is likely in this case. Patients are also at increased risk
for GI, breast, and genital tract cancers.

Management involves screening for various malignancies, as well as monitoring for polyp
formation and associated complications. In addition to an annual hemoglobin evaluation to
detect anemia, screening upper and lower endoscopies are indicated. Polyps are monitored
every few years, and polypectomy is indicated if lesions are large, symptomatic, or malignant.
*head and neck squamous cell carcinoma (SCC)

Malignancy is extremely likely in a patient with a persistent (>2 weeks), palpable (>1.5 cm), firm neck mass; a
smoking history; and no preceding infection. By far the most common malignancy in an upper cervical node is
mucosal head and neck squamous cell carcinoma (SCC). Indeed, the first (and only) apparent manifestation
may be a palpable cervical lymph node, representing regional nodal metastasis. Referred otalgia is another
common presenting symptom, facilitated by either the glossopharyngeal nerve (CN IX) (innervates both the
base of tongue and the external auditory canal [EAC]) or the vagus nerve (CN X; innervates parts of the
larynx/hypopharynx and the EAC).

Identification of the primary source of head and neck SCC is essential to direct treatment. Thorough
examination includes endoscopic visualization using laryngopharyngoscopy as well as neck imaging (CT with
contrast) to evaluate the primary site and characterize the cervical nodal disease. Fine-needle aspiration of the
lymph node is advised over open biopsy to avoid tumor seeding.
*Folate deficiency anemia
Folate deficiency anemia
• Chronic hemolysis (eg, sickle cell disease)
• Poor dietary intake
Etiology • Malabsorption (eg, gastric bypass)
• Medications (eg, methotrexate, phenytoin)

• Dyspnea, fatigue, pallor, weakness


Clinical features
• Macrocytic anemia
• Poor reticulocyte response (low to normal)
Laboratory
• Hypersegmented neutrophils
findings
• Low serum folate

• Folic acid supplementation


Treatment

This patient has progressive exertional dyspnea, fatigue, pallor, and generalized weakness due
to severe anemia. The most likely cause of macrocytic anemia (mean corpuscular volume
>100 μm3) in this patient with sickle cell disease (SCD) is folate deficiency.

SCD is a chronic hemolytic anemia in which red blood cells are typically normocytic and
normochromic with a compensatory elevated reticulocyte count. Without adequate folic acid
supplementation or intake, chronic hemolysis can lead to folate deficiency, as the bone marrow
uses folate in an attempt to produce red blood cells. Folate deficiency results in laboratory
findings of macrocytic anemia and hypersegmented neutrophils. Due to ineffective
erythropoiesis, the expected reticulocytosis is absent and an inappropriately low reticulocyte
count is seen (ie, low corrected reticulocyte count).

Although this patient's reticulocyte count appears normal (0.5%-1.5%), it is lower than expected
for SCD.

Adequate folic acid intake via diet or supplementation is recommended to correct the
underlying folate deficiency.
*acute promyelocytic leukemia (APL)

This patient's pancytopenia and atypical myeloid cells with Auer rods on peripheral blood
smear raises suspicion for acute promyelocytic leukemia (APL), a clinically unique form of
acute myelogenous leukemia (AML) that primarily affects young and middle-aged adults.

In APL, clonal promyelocytes fill the bone marrow and interfere with normal hematopoiesis,
leading to pancytopenia with anemia (eg, dyspnea on exertion, fatigue), thrombocytopenia (eg,
bleeding complications), and neutropenia (eg, infections). The diagnosis is often suspected
when a peripheral blood smear shows numerous atypical promyelocytes (large myeloid cells
with intracytoplasmic violet granules) that contain Auer rods (seen with other forms of
AML). However, confirmation of the diagnosis of APL requires cytogenic analysis for the
characteristic mutation, the fusion of the promyelocytic leukemia gene and the retinoic acid
receptor alpha gene (t[15;17]).

APL is considered a medical emergency due to the high risk of pulmonary/cerebrovascular


hemorrhage from tumor-induced consumptive coagulopathy. Although the exact mechanism
is unclear, APL appears to promote a bleeding diathesis via the activation of tissue factor
(disseminated intravascular coagulation) and the increased generation of plasmin (primary
hyperfibrinolysis). Urgent treatment with all-trans retinoic acid, which promotes
differentiation of the atypical promyelocytes, is required to prevent life-threatening bleeding
complications and induce long-term remission.
*intrinsic pediatric AKI

This patient's prodromal diarrheal illness, followed by anemia (likely hemolytic),


thrombocytopenia, and acute kidney injury (AKI), is consistent with hemolytic uremic
syndrome (HUS). HUS is typically seen in children and usually occurs due to Shiga toxin–
producing diarrheagenic pathogens such as Escherichia coli O157:H7 or, less commonly,
Shigella. The production and systemic circulation of the Shiga toxin cause injury to small
blood vessel endothelium, leading to a thrombotic microangiopathy and resultant:

• renal vascular occlusion (eg, afferent arteriole, glomerular capillaries) by microthrombi;


this blockage increases pressure within glomerular capillaries, leading to intrinsic acute
kidney injury (eg, BUN/creatinine ratio of <20:1, oliguria, hypertension, hematuria,
proteinuria).
• platelet consumption within microthrombi, resulting in thrombocytopenia.
• red blood cell (RBC) destruction due to physical RBC shearing from passage through
blood vessels occluded by systemic microthrombi, resulting in a hemolytic anemia (eg,
fatigue, pallor).

Treatment of HUS is supportive (eg, fluid management, transfusions) because most children
(60%-70%) recover completely.

(Choice A) This patient's murmur and tachycardia could be indications of heart failure,
possibly related to recent illness (eg, viral myocarditis, bacterial endocarditis), and decreased
cardiac function can cause tubular injury from prolonged renal ischemia. However, this
patient's acute anemia, thrombocytopenia, and AKI are most consistent with HUS, and his
systolic flow murmur is likely due to severe anemia.

(Choice B) Although IgA vasculitis (Henoch-Schonlein purpura), a disease of the renal


vasculature, can cause abdominal pain and AKI, patients typically have associated purpura
(>75%) and joint pain (>80%); thrombocytopenia is unusual.

(Choice C) Poststreptococcal glomerulonephritis is a common cause of intrinsic renal injury in


children but typically presents several weeks after group A streptococcal pharyngitis or
impetigo with edema, hypertension, and hematuria. Anemia and thrombocytopenia are
uncommon.

(Choice E) Acute renal vasoconstriction typically presents with prerenal AKI, which can occur
in children who take nonsteroidal anti-inflammatory drugs during periods of hypovolemia (eg,
gastroenteritis). However, patients typically have a normal complete blood count and a
BUN/creatinine ratio of >20:1, making this diagnosis less likely.

Educational objective:
Hemolytic uremic syndrome (HUS) is characterized by the triad of hemolytic anemia,
thrombocytopenia, and acute kidney injury (typically following a prodromal diarrheal illness
caused by Escherichia coli O157:H7). The mechanism of intrinsic renal injury in HUS is renal
vascular occlusion caused by capillary microthrombi
*splenectomy

This patient likely had a splenectomy during his operation for multiple gunshot wounds to the
abdomen. He now has high fever, hypotension, tachypnea, and tachycardia in the setting of
bacteremia with gram-positive cocci, suggesting an overwhelming Streptococcus pneumoniae
infection.

Encapsulated organisms such as S pneumoniae, Haemophilus influenzae, and Neisseria


meningitidis have a polysaccharide exterior that conceals antigenic epitopes and resists innate
phagocytosis. Therefore, these pathogens are largely eliminated via the humoral immune
response with antibody-mediated phagocytosis(opsonization) and antibody-mediated
complement activation. Much of this is dependent on splenic macrophages and the generation
of splenic opsonizing antibodies. As such, patients with asplenia are at high risk for fulminant
infection with encapsulated organisms. These patients should be immunized with
pneumococcal, meningococcal, and H influenzae type B vaccines and take oral antibiotics early
in the course of any febrile illness.
*Platelet dysfunction in CKD

Platelet dysfunction in chronic kidney disease


• Rising urea → upregulation of nitric oxide →
Pathogenesis ↓ platelet adhesion, activation & aggregation

• Easy bruising
Clinical features • Mucosal bleeding (eg, epistaxis, GI hemorrhage)

• Normal platelet count (may be mildly low)


Laboratory findings • Normal coagulation studies

• Desmopressin (↑ vWF secretion from endothelial cells)


o Only required if having bleeding symptoms
Treatment
or upcoming procedure

GI = gastrointestinal; vWF = von Willebrand factor.

Classic causes of prolonged bleeding (eg, persistent epistaxis) include platelet or coagulation disorders. In this
case, the patient has a normal platelet count and normal coagulation studies (eg, PT, activated PTT) in addition
to chronic kidney disease (CKD). Therefore, the most likely diagnosis is platelet dysfunction, a common
cause of easy bruising and mucosal bleeding in those with CKD.
Patients with CKD have disruption of primary hemostasis, including platelet–von Willebrand factor (vWF)
interactions. As levels of urea (the end product of ammonia breakdown) rise, its precursors are shunted to a
different pathway that leads to nitric oxide production. Inappropriately high nitric oxide results in decreased
platelet adhesion, activation, and aggregation.

In patients with CKD and bleeding tendencies, a diagnosis of platelet dysfunction can be established with a
normal platelet count and normal coagulation studies. Symptomatic patients are typically treated with
desmopressin (DDAVP), which stimulates the release of vWF and reduces bleeding time.

(Choice A) Antiplatelet antibodies are associated with (but not routinely measured in) immune
thrombocytopenia, which can cause persistent epistaxis. Other conditions associated with platelet antibodies
and epistaxis include drug-induced thrombocytopenia and, rarely, heparin-induced thrombocytopenia (which is
more likely to cause clotting). However, platelet count would be low (usually <10,000/mm3 in patients with
immune thrombocytopenia and mucosal bleeding).

(Choice B) Unexplained cytopenias, particularly involving multiple cell lines, may warrant bone marrow
biopsy to assess for an underlying cause (eg, aplastic crisis). This patient's normocytic anemia is characteristic
of CKD due to reduced erythropoietin production, making bone marrow evaluation unnecessary. In addition,
anemia would not cause persistent bleeding.

(Choice D) Factor VII deficiency is rare but may lead to mucosal bleeding. However, PT would be prolonged.

(Choice E) Thrombin time (TT) reflects the last step of the clotting cascade (fibrinogen to fibrin conversion),
which is mediated by thrombin; it is typically ordered if a prolonged PTT is suspected to be due to a fibrinogen
disorder or to anticoagulant exposure. For example, TT is elevated in disseminated intravascular coagulation,
which causes bleeding; unlike this case, patients typically have an acute risk factor (eg, sepsis, trauma),
thrombocytopenia, and prolonged PT and PTT.

Educational objective:
Platelet dysfunction is common in patients with chronic kidney disease due to decreased platelet adhesion,
activation, and aggregation. Patients often have easy bruising or mucosal bleeding, and platelet count and
coagulation studies are typically normal.
*Electrophoresis patterns in sickle cell syndromes

Electrophoresis patterns in sickle cell syndromes


HbA HbA2 HbF HbS HbC
Normal ++++ + + None None
Sickle cell trait +++ + + +++ None
Sickle cell anemia (SCA) None + + ++++ None
SCA on hydroxyurea None + ++ +++ None
Hemoglobin SC disease None + + +++ +++

All functioning hemoglobin contains a pair of alpha globulins and a pair of non-alpha globulins. The non-
alpha globulin differentiates the hemoglobin type and determines the physiologic properties of the molecule in
different conditions (eg, pH, temperature, 2,3-bisphosphoglycerate level). In healthy children and adults:

• The predominant hemoglobin is hemoglobin A (HbA) (alpha and beta globulins).


• Small quantities (1%-2%) of other hemoglobin are also present, including hemoglobin A2 (alpha and
delta globulins) and hemoglobin F (HbF) (alpha and gamma globulins).

Sickle cell disease (SCD) is marked by an absence of normal beta globulin due to mutations in both beta
globulin genes. The sickle mutation generates a new type of hemoglobin, hemoglobin S (HbS), which
polymerizes when deoxygenated. This leads to reduced erythrocyte deformability and increased erythrocyte
fragility, causing painful vasooclusion, organ ischemia, and chronic hemolysis.

Because patients with SCD are unable to generate normal beta globulin, they cannot produce HbA; therefore,
HbA will be undetectable on electrophoresis (Choices C, E, F). Other forms of hemoglobin that do not contain
beta globulin (eg, HbA2, HbF) are usually present at normal or mildly increased levels.

Because HbF prevents HbS from polymerizing, hydroxyurea, which increase HbF levels, is often
administered to reduce SCD complications. Patients who respond to hydroxyurea (as in this patient with
reduced symptoms) usually have 10%-30% HbF on electrophoresis. Side effects of therapy include
macrocytosis and myelosuppression.

(Choice A) Patients with sickle cell anemia (heterozygous sickle cell disease) who are not on hydroxyurea
predominantly have HbS with normal or mildly increased HbF. Hydroxyurea significantly increases HbF
production.

(Choice D) Hemoglobin SC disease is marked by the sickle mutation in one beta globulin and a different point
mutation in the other beta globulin (lysine for glutamine at position 6). This leads to 2 predominant hemoglobin
types: HbS and HbC. Treatment with hydroxyurea is also helpful in this condition and would cause elevated
(not normal) HbF levels.

Educational objective:
Sickle cell disease is marked by mutations in both beta globulins with at least one beta globulin having the
sickle mutation. This generates a new type of hemoglobin called HbS, which polymerizes when
deoxygenated. Treatment with hydroxyurea increases Hemoglobin F, which prevents HbS polymerization and
reduces symptoms.
*Evaluation of anemia
*immune mediated hemolysis

This patient's fatigue, exertional dyspnea, and low hemoglobin suggest acute symptomatic
anemia. Although anemia can be triggered by a wide range of conditions, concurrent clinical
data often narrows the differential diagnosis.

In this case, a high reticulocyte count suggests that there are adequate levels of iron, folate,
and vitamin B12, to generate new erythrocytes, which makes cytokine-mediated iron
dysregulation (eg, anemia of chronic disease) and folate deficiency (eg, leukemic folate
consumption) unlikely (Choices A and B). It also suggests that the bone marrow is responding
appropriately to the anemia by increasing the production of erythrocytes, which (along with the
normal platelet count) makes bone marrow infiltration with leukemic cells less likely (Choice
D).

Most cases of anemia with a high reticulocyte count are caused by acute bleeding or
hemolysis. In this case, hemolysis is most likely because patients with chronic lymphocytic
leukemia often have significant immune dysregulation, which triggers the formation of IgG
autoantibodies against the erythrocyte membrane (warm agglutinins). Red blood cells
(RBCs) coated with IgG are subsequently identified by the Fc receptor on splenic macrophages
and partially or wholly phagocytized, leading to extravascular, immune-mediated hemolysis
(ie, autoimmune hemolytic anemia). Patients typically present with signs of acute anemia and
RBC lysis (eg, jaundice, dark urine, elevated indirect bilirubin). The diagnosis is confirmed
with a direct antiglobulin (Coombs) test, which detects the presence of IgG or C3 (a
complement fragment) on erythrocytes.

(Choice E) Autoantibodies against erythroid precursors can lead to pure red cell aplasia, which
is marked by anemia with a paucity of reticulocytes (not reticulocytosis).

Educational objective:
Anemia with reticulocytosis suggests that the bone marrow is responding appropriately to the
anemia by generating new erythrocytes and that sufficient levels of folate, vitamin B12, and iron
are available for erythrocytosis. Anemia with reticulocytosis is commonly seen in acute
bleeding conditions and hemolysis. Chronic lymphocytic leukemia is associated with warm
autoimmune hemolytic anemia.
*Drug-induced neutropenia

Drug-induced neutropenia
• Methimazole, propylthiouracil
• Trimethoprim-sulfamethoxazole
Common triggers • Clozapine
• Sulfasalazine

• Immune mediated: drug or metabolite binds to


neutrophils & triggers a cell-mediated or humoral
response
Mechanism
• Bone marrow toxicity: drug or metabolite directly or
indirectly damages granulocyte precursors

• Usually within 3 months of drug exposure


• Oropharyngeal ulceration ± fever
Manifestations
• Other cell lines unaffected

• Discontinue inciting medication


• Antibiotics in setting of treatable infection
Management
• Consider granulocyte colony-stimulating factor

This patient with fever and pharyngitis has severe neutropenia (absolute neutrophil count
[ANC] <500/mm3) with normal platelet and erythrocyte counts. Given the recent initiation of
sulfasalazine, the most likely underlying etiology is drug-induced neutropenia.

Medications trigger neutropenia due to 1 of the following mechanisms:

• Direct toxicity: The medication directly destroys granulocyte precursors in the bone
marrow. Direct toxicity is usually dose dependent and is seen most often with cytotoxic
chemotherapy medications, clozapine, and dapsone.
• Immune-mediated destruction: The medication (or a metabolite) adheres to the
neutrophil surface and acts as a hapten, which triggers a cytotoxic T-cell or antibody
response that destroys the cell. Most cases are idiopathic (not dose dependent); common
triggers include sulfasalazine, trimethoprim-sulfamethoxazole, propylthiouracil, and
flecainide.

Drug-induced neutropenia usually presents with oropharyngeal mucositis (eg, sore throat,
pharyngeal erythema, ulcers) with or without fever. Platelet and leukocyte counts are generally
normal. When a patient is taking a medication associated with neutropenia (eg, sulfasalazine),
the most important first step is to immediately stop the offending agent. Most cases resolve
within 1-3 weeks.
Although oropharyngeal erythema, malaise, fever, and neutropenia can occur with viral
infections (eg, Epstein-Barr virus), the neutrophil nadir is usually mild (ANC 500-1,500/mm3)
and patients typically have other prominent symptoms (eg, lymphadenopathy, splenomegaly)
(Choice E). Neutropenia due to bacterial infection (eg, sepsis) is usually associated with a
decrease in the other cell lines (eg, anemia, thrombocytopenia).

(Choice A) Corticosteroid therapy is used to treat some conditions that cause immune-
mediated blood cell destruction (eg, thrombotic thrombocytopenic purpura, immune
thrombocytopenia). Although immune-mediated destruction of neutrophils may have a role in
the pathogenesis of drug-induced neutropenia, steroids have not been shown to improve
outcomes.

(Choice C) Rheumatologic conditions (eg, systemic lupus erythematosus) typically cause mild
neutropenia. Stopping a common offending medication (eg, sulfasalazine) would take
precedence over additional testing (eg, antinuclear antibody) at this time.

(Choice D) Bone marrow sampling is painful and invasive. Therefore, it is usually reserved for
cases of neutropenia that do not resolve with discontinuation of a possible inciting medication
or when there is a high degree of diagnostic uncertainty.

Educational objective:
Drug-induced neutropenia is typically severe (absolute neutrophil count <500/mm 3), isolated to
leukocytes (eg, no anemia or thrombocytopenia), and often manifests with oropharyngeal
mucositis and fever. The most important first step is discontinuation of the offending
medication. Commonly implicated agents include cytotoxic chemotherapy drugs, clozapine,
dapsone, sulfasalazine, trimethoprim-sulfamethoxazole, and propylthiouracil.
*Iron deficiency anemia in young children

Iron deficiency anemia in young children


• Prematurity
• Lead exposure
• Age <1
o Delayed introduction of solids (ie, exclusive
breastfeeding after 6 months)
Risk factors
o Cow's, soy, or goat's milk
• Age >1
o >24 oz (>700 mL) per day of cow's milk
o <3 servings/day of iron-rich foods

• Screening hemoglobin at age 1


Diagnosis • Hemoglobin <11 g/dL (<6.83 mmol/L), ↓ MCV, ↑ RDW

• Empiric trial of iron supplementation


Treatment
MCV = mean corpuscular volume; RDW = red blood cell distribution width.

This patient's microcytic anemia and excessive cow's milk intake are concerning for iron
deficiency anemia (IDA). Cow's milk has low iron content. In addition, the bioavailability of
that iron is poor because the calcium and caseinophosphopeptide found in cow's milk interfere
with intestinal absorption of dietary iron. Also, consuming large volumes of cow's milk
replaces the normal intake of iron-rich foods. For these reasons, children who consume >24
oz/day are especially at risk for IDA.

Although severe IDA can impair neurocognitive development, most children are asymptomatic
and have age-appropriate skills (as seen in this patient). Tachycardia and a flow murmur may
be present because cardiac output increases to compensate for the reduced oxygen-carrying
capacity of iron-deficient red blood cells (RBCs).

Inadequate iron stores in the bone marrow lead to a low erythrocyte count and the production
of hypochromic RBCs with a low mean corpuscular volume. However, RBC distribution
width (size variability) is elevated because the amount of iron available for RBC synthesis
varies throughout the day.
*sickle cell disease (SCD) & Pregnancy

Sickle cell disease Preeclampsia with


HELLP syndrome
acute pain episode severe features
• Persistent/severe
• Acute, severe • Epigastric/RUQ
headaches,
pain in any pain,
scotomata, ±
location nausea/vomiting,
Clinical features epigastric/RUQ
• Elevated BP ± headaches
pain
(due to pain) • Elevated BP
• Elevated BP

• Normal/slightly
Hemoglobin • Decreased • Decreased
levels increased

• ↑ Bilirubin • ↑ Bilirubin
Hemolysis • ↑ LDH • None • ↑ LDH
• ↓ Haptoglobin • ↓ Haptoglobin
Laboratory
evaluation Reticulocyte • Increased • Normal • Increased
count
Platelets • Normal • <100,000/mm3 • <100,000/mm3

• ≥2× upper limit • ≥2× upper limit


• Normal
AST/ALT of normal of normal

AST/ALT = aspartate transaminase/alanine transaminase; BP = blood pressure; HELLP =


Hemolysis, Elevated Liver enzymes, and Low Platelet count; Hgb = hemoglobin; LDH =
lactate dehydrogenase; RUQ = right upper quadrant.

This patient with sickle cell disease (SCD) and severe, diffuse pain is likely having an acute
pain episode (ie, vasoocclusive pain crisis). Pregnancy is associated with an increased number
of pain episodes, especially in the third trimester. This is because physiologic changes of
pregnancy (eg, increased metabolic demand) promote hemoglobin polymerization and sickle
formation; in addition, maintenance medications (eg, hydroxyurea) are often stopped by patients
due to fear of fetal toxicity.

This patient's severe nausea/vomiting (eg, ketonuria, elevated amylase and transaminase levels)
is the likely triggering factor for this pain episode. The low hemoglobin, elevated reticulocyte
count, and bilirubin provide evidence of chronic hemolysis characteristic of SCD. Fetal heart
rate decelerations are expected during a crisis (due to decreased fetal oxygenation and
perfusion) and resolve as the mother's condition stabilizes.
Treatment of a vasoocclusive pain episode in pregnant patients is similar to that in nonpregnant
patients: intravenous (IV) hydration and aggressive pain control (eg, morphine patient-
controlled analgesia). Prompt and sufficient pain control with opioids is critical for relieving
pain while its precipitating cause is treated.

(Choice A) After appropriate IV hydration and pain control, imaging can be considered if there
are no signs of improvement or obvious triggers of the painful episode.

(Choices B and D) Magnesium sulfate is used in preeclampsia with severe features and
HELLP (Hemolysis, Elevated Liver enzymes, Low Platelets) syndrome. Although hemolysis
is present, the transaminases are only mildly elevated and the platelet count is normal, making
HELLP syndrome unlikely. Preeclampsia is marked by hypertension (blood pressure ≥140/90
mm Hg), proteinuria, and signs of end-organ damage (eg, elevated creatinine and transaminase
levels). This patient does not meet blood pressure or laboratory criteria for
preeclampsia. Delivery is curative in HELLP syndrome and preeclampsia; however, the
associated blood loss can exacerbate an acute pain episode.

(Choice C) Nonsteroidal anti-inflammatory drugs (eg, ketorolac) are generally avoided in SCD
because of the risk for acute and chronic renal injury. In addition, they are avoided in
pregnancy after 32 weeks gestation due to the risk for premature closure of the fetal ductus
arteriosus.

Educational objective:
Pregnancy is associated with an increase in vasoocclusive pain episodes in patients with sickle
cell disease due to physiologic changes. Each episode is treated with aggressive pain control
and intravenous hydration during treatment of the precipitating causes.
*Ddx of bone pain in SCD

Differential diagnosis of bone pain in sickle cell disease


Cause Vaso-occlusive crisis Osteomyelitis Avascular necrosis
• Acute, severe pain
• Acute or subacute pain
• Pain >1 site (eg, • Chronic, worsening pain
• Focal pain at 1 site (eg,
dactylitis) (eg, femoral head)
long bone)
Clinical • +/- Low-grade fever • Absence of fever
• Prolonged fever
features • Erythema & warmth • Absence of warmth or
• Erythema & warmth
• May be preceded by erythema
• Positive blood culture
trigger (eg, dehydration)

Dactylitis (hand-foot syndrome) can be the earliest manifestation of vaso-occlusion in sickle cell disease
(SCD). Vaso-occlusion of the blood vessels supplying the metacarpals and metatarsals occurs secondary to
polymerization of sickled hemoglobin and increased red blood cell adhesion. This syndrome usually presents at
age 6 months to 4 years with an acute onset of pain and symmetric swelling of the hands and feet. Low-grade
fever is sometimes present.

Initial radiographs reveal only soft tissue swelling, although in recurrent episodes they can reveal a mottled
appearance of bone. Although most children with SCD in the United States are diagnosed by hemoglobin
electrophoresis on newborn screening, the presence of dactylitis in a previously healthy child should prompt
evaluation for SCD. Treatment involves hydration and pain control.

(Choice A) Systemic juvenile idiopathic arthritis can present in early childhood with joint pain but most
commonly presents with rash, high fever, and large joint involvement. Other autoimmune disorders (eg,
psoriatic arthritis, ankylosing spondylitis) that can cause dactylitis classically occur in older children and adults.

(Choice B) Hypertrophic osteoarthropathy is a clinical syndrome comprising digital clubbing and bony
swelling of the toes and fingers. Most commonly caused by pulmonary diseases (eg, cystic fibrosis) or
malignancy, these changes develop chronically and are typically seen in older children or adults.

(Choice C) Avascular necrosis, or osteonecrosis, of the femoral or humeral head is a complication in SCD
caused by infarction of the articular cartilage and bony surface after chronically decreased blood flow. It
presents with chronic, progressive hip pain with weight-bearing.

(Choices E and F) Due to splenic autoinfarction, patients with SCD have poor splenic function and are thus
prone to developing bacterial infections, including osteomyelitis. Osteomyelitis typically presents with
swelling, fever, and pain localized to a single bone (rather than bilateral hands and feet). Staphylococcus aureus
is the most common cause of bacterial osteomyelitis in all patients, and patients with SCD have an increased
risk of Salmonella osteomyelitis.

Educational objective:
Dactylitis can be the earliest manifestation of vaso-occlusive disease in sickle cell disease and presents with the
acute onset of bilateral hand and foot swelling and tenderness.
*HIT type 2
This patient with thrombocytopenia and thrombus progression 3 days into heparin therapy
likely has type 2 heparin-induced thrombocytopenia (HIT). Heparin induces a
conformational change to a platelet surface protein (platelet factor 4 [PF4]), which exposes a
neoantigen. In patients with type 2 HIT, the immune system responds by forming an IgG
autoantibody (HIT antibody) that then coats the surface of platelets and forms complexes
(heparin-PF4-HIT antibody), resulting in:

• Thrombocytopenia - the reticuloendothelial system (largely the spleen) removes


antibody-coated platelets, causing a mild to moderate thrombocytopenia (rarely
<20,000/mm3).
• Arterial and venous thrombus - HIT antibodies activate platelets, resulting in platelet
aggregation and the release of procoagulant factors. The risk of thrombus is as high as
50% in untreated HIT.

Although type 2 HIT usually manifests with a >50% drop in platelets 5-10 days after the
initiation of heparin, it can occur earlier (sometimes <1 day) in patients previously
exposed. Patients with end-stage renal disease typically receive unfractionated heparin with
hemodialysis to prevent blood clots in the dialysis tubing and dialyzer; therefore, this patient
who recently started hemodialysis has likely had heparin exposure. If HIT is suspected, all
heparin products should be discontinued immediately, and an alternate anticoagulant (eg,
argatroban, fondaparinux) should be initiated.

Heparin works by binding antithrombin, which inactivates factor Xa; this prolongs activated
partial thromboplastin time
*Treatment of acute DVT/PE
Treatment of acute deep vein thrombosis/pulmonary embolism
Oral factor Xa inhibitors Warfarin
Mechanism
Direct factor Xa inhibition Vitamin K antagonism
of action
Therapeutic onset 2-4 hr 5-7 days
Yes, overlap with UFH
Overlap needed? No
or LMWH for ~5 days
Laboratory monitoring No PT/INR
LMWH = low molecular weight heparin; UFH = unfractionated heparin.

This patient was recently diagnosed with deep vein thrombosis (DVT). He has been compliant with taking his
warfarin, but his schedule has prevented proper anticoagulation monitoring; his DVT has now progressed in the
setting of a subtherapeutic INR. The risk of pulmonary embolism (PE) is higher with proximal (eg, femoral
vein, iliac vein) than distal (eg, below the knee) DVT; therefore, he needs immediate anticoagulation.

Oral direct factor Xa inhibitors (eg, rivaroxaban, apixaban) are as effective as warfarin in the treatment of
acute DVT or PE and do not increase the risk of bleeding complications. These drugs have the advantage of
rapid onset of action, no requirement for laboratory (eg, INR) monitoring, and no requirement for overlap
therapy with heparin. Therefore, these agents are becoming preferred for the treatment of acute DVT and
PE. These drugs are an especially good option in patients who have difficulty with the dietary restrictions or
frequent INR monitoring required with warfarin.

(Choice A) In this patient with a worsening DVT and an INR of <2.0, it is not appropriate to increase the
warfarin dose without overlapping with heparin anticoagulation. In addition, if this patient remains on warfarin,
it is likely that his diet and schedule will continue to cause issues with maintaining a therapeutic INR.

(Choice B) Thrombolytic therapy is typically reserved for hemodynamically unstable patients with PE. Less
commonly, it is used for massive proximal DVT associated with significant symptomatic swelling and/or limb
ischemia. It is not indicated in this patient with moderate pretibial edema and absence of hypotension and
tachycardia.

(Choice C) Anticoagulation failure (eg, recurrent or extending thromboembolism while fully anticoagulated) or
anticoagulation contraindication (eg, active bleeding) are indications for inferior vena cava filter
placement. This patient is not considered to have anticoagulation failure as his INR is grossly subtherapeutic
(he is not fully anticoagulated).

(Choice E) Aspirin is inadequate for the treatment of acute DVT.

Educational objective:
Oral direct factor Xa inhibitors (eg, rivaroxaban, apixaban) have similar efficacy to warfarin in the treatment of
acute venous thromboembolism and do not increase the risk of bleeding complications. These drugs do not
require laboratory monitoring or overlap therapy with heparin and are becoming preferred agents for the
treatment of acute venous thromboembolism.
*Polycythemia vera
Polycythemia vera
• ↑ Blood viscosity
o Hypertension
o Erythromelalgia (burning cyanosis in hands/feet)
o Transient visual disturbances
Manifestations
• ↑ RBC turnover (gouty arthritis)
• Aquagenic pruritus
• Bleeding

• Facial plethora (ruddy cyanosis)


Examination • Splenomegaly

• Elevated hemoglobin
• Leukocytosis & thrombocytosis
Laboratory • Low erythropoietin level
• JAK2 mutation positive
findings
• Thrombosis
Complications • Myelofibrosis & acute leukemia

• Phlebotomy
Treatment • Hydroxyurea (if ↑ risk of thrombus)

RBC = red blood cell.

This patient with aquagenic pruritus (itching after bathing), facial plethora (ruddy cyanosis),
splenomegaly, and elevations in all 3 cell lines on complete blood count likely has
polycythemia vera (PV).

PV is a clonal myeloproliferative disorder characterized by erythrocytosis (increase in red


blood cell mass). It is a primary form of polycythemia, almost always caused by a JAK2
mutation. Normally, red blood cell production is dependent on erythropoietin (EPO), a
cytokine released by the kidneys (and liver) in response to tissue hypoxia; EPO activates the
JAK2 tyrosine kinase, which differentiates late myeloid cells into erythrocytes. In PV, red
blood cell production is driven by a constitutively active JAK2 gene rather than by tissue
hypoxia; therefore, EPO levels tend to be low.

Secondary forms of polycythemia (eg, due to chronic hypoxia or EPO-secreting tumors) are
caused by circulating factors (usually EPO) that stimulate the bone marrow to produce more red
blood cells. These forms of polycythemia are often characterized by high EPO levels.
*Febrile nonhemolytic transfusion reaction (FNHTR)

Febrile nonhemolytic transfusion reaction (FNHTR), the most common adverse transfusion
reaction, is a benign occurrence caused by cytokine release from leukocytes within the donor
blood product.

During preparation of blood products, whole blood is separated into red blood cells and
plasma. However, small amounts of leukocyte debris remain in the red cell
concentrate. During blood storage, these leukocytes can release cytokines, which when
transfused can cause transient fever, chills, and malaise within 1-6 hours of transfusion
initiation. Management includes transfusion cessation (to exclude other serious reactions) and
antipyretic administration.

Risk of recurrent FNHTR is high after a first episode, and prevention is best achieved by using
leukoreduced blood products for future transfusions. Leukoreduction involves filtering out
most of the leukocytes after blood collection but before blood storage, thereby minimizing
cytokine production within the packed red blood cell product. Leukoreduction also reduces the
risk of human leukocyte antigen alloimmunization and transmission of cytomegalovirus (which
typically resides in leukocytes). Many facilities empirically leukoreduce all blood products, but
this process is not universal.
(Choice A) Careful cross-matching of blood prevents acute hemolytic transfusion reaction
(AHTR), which is caused by mismatched donor and recipient blood. Symptoms include chills,
flank pain, and gross hematuria within an hour of transfusion. Unlike this patient, those with
AHTR have a positive direct antiglobulin (Coombs) test, increased plasma-free hemoglobin,
and hemoglobinuria.

(Choices B and E) Citrate, which binds ionized calcium, is the main anticoagulant used for
blood product storage and can cause hypocalcemia if a massive transfusion is required (eg,
cardiac surgery). Calcium gluconate infusion is used to prevent or treat severe
hypocalcemia. Similarly, warming the blood product is recommended during rapid, massive
transfusion to prevent hypothermia and arrhythmia.

(Choice C) Premedication with antipyretics or antihistamines (eg, diphenhydramine), which


was previously used in some centers, has been found not to reduce the incidence of FNHTR in
several studies.

Educational objective:
Febrile nonhemolytic transfusion reaction (FNHTR) is the most common adverse reaction and
is caused by release of cytokines from leukocytes in the stored blood product. Symptoms
develop within 1-6 hours and include transient fever, chills, and malaise. Leukoreduction of
donor blood helps prevent FNHTR
*Infantile vitamin K–deficient bleeding

Infantile vitamin K–deficient bleeding


• Low vitamin K stores (poor placental transfer, sterile gut,

Pathophysiology low content in breast milk)


• Inefficient vitamin K use by immature liver

• Classically presents on day 2-7 of life*


• Easy bruising
Clinical features • Umbilical, mucosal & gastrointestinal bleeding
• Intracranial hemorrhage

• ↑ PT
Laboratory findings • ↑ PTT (if severe)
• Normal platelet count

• Intramuscular vitamin K at birth


Prevention
*Can occur up to age 6 months.

This neonate who has not received prior medical care has bruising and prolonged PT and PTT,
findings concerning for vitamin K–deficient bleeding (VKDB).

Vitamin K is essential for enzymatic carboxylation and activation of coagulation factors II


(prothrombin), VII, IX, and X. Newborns are vitamin K deficient due to poor placental
transfer, lack of intestinal flora to synthesize vitamin K, and immature liver function (ie, poor
vitamin K utilization). Therefore, intramuscular vitamin K is given at birth to prevent
spontaneous bleeding. Neonates who do not receive this injection, such as this patient, are at
highest risk of VKDB, particularly if exclusively breastfed because breast milk contains
insufficient vitamin K.

Classic VKDB presents on day 2-7 of life with easy bruising, mucosal bleeding (eg,
epistaxis), gastrointestinal hemorrhage, and/or intracranial hemorrhage. Late-onset disease
can occur up to age 6 months. Because of reduced coagulation factor carboxylation, laboratory
evaluation shows prolonged PT and, in severe deficiency, prolonged PTT. The diagnosis is
confirmed by the reversal of laboratory abnormalities with vitamin K administration
*pseudothrombocytopenia

This patient has mild thrombocytopenia with no associated symptoms (eg, bruising,
mucocutaneous bleeding), no personal/family history of bleeding diathesis, and no abnormal
physical examination findings. Peripheral blood smear reveals large clumps of platelets. This
presentation raises suspicion for pseudothrombocytopenia.

Pseudothrombocytopenia is a laboratory error caused by platelet aggregation in vitro. Most


cases are due to incompletely mixed blood samples or the presence of serum antibodies to
ethylenediaminetetraacetic acid (EDTA), an anticoagulant used in hematologic testing. The
error is generally identified when a patient with mild thrombocytopenia has peripheral blood
smear evidence of large clumps of platelets. Drawing blood samples in tubes with a non-
EDTA anticoagulant (eg, heparin, sodium citrate) normalizes the automated platelet count and
confirms the diagnosis. Because patients with pseudothrombocytopenia do not have true
thrombocytopenia, they do not require intervention or monitoring.
(Choice A) Gestational thrombocytopenia is the most common cause of thrombocytopenia
during pregnancy. It is generally mild and does not cause maternal or fetal
morbidity. However, gestational thrombocytopenia usually arises in the second half of the
pregnancy (not at 10 weeks), and peripheral blood smear would reveal a paucity of platelets
(not platelet clumping).

(Choice B) Glanzmann thrombasthenia is an autosomal recessive disorder associated with


impaired platelet aggregation. Patients generally have a history of mucocutaneous bleeding,
normal platelet counts, and no platelet clumping on peripheral blood smear.

(Choice C) Idiopathic thrombocytopenic purpura, which is approximately tenfold more


common in pregnant women than in the general population, usually presents with mild
thrombocytopenia. Patients are often asymptomatic and have no history of bleeding
disorder. However, peripheral blood smear would show a paucity of platelets, not platelet
clumping.

(Choice E) Vitamin B12 deficiency can occasionally cause mild thrombocytopenia and
leukopenia. However, peripheral blood smear usually shows hypersegmented neutrophils and
macrocytic anemia; platelet clumping would not be seen.

(Choice F) Von Willebrand disease is the most common inherited bleeding disorder. It is
marked by a defect in platelet aggregation due to a deficiency of von Willebrand factor. Most
cases are asymptomatic, and patients have normal platelet counts and peripheral smears.

Educational objective:
Pseudothrombocytopenia is a laboratory error characterized by platelet aggregation in vitro. It
is generally confirmed when peripheral smear reveals large clumps of platelets. Patients do not
require additional evaluation or follow-up and are not at risk for bleeding
*Hairy cell leukemia

Hairy cell leukemia


• Clonal B-cell neoplasm
• Middle-age/older adults
Features
• BRAF mutation

• Pancytopenia due to bone marrow fibrosis


o Granulocytopenia (infections)
Manifestations
o Anemia (fatigue, weakness)
o Thrombocytopenia (bleeding, bruising)
• Splenomegaly (early satiety)
• Hepatomegaly/lymphadenopathy rare

• Peripheral smear – "hairy" leukocyte cells


Diagnosis • Bone marrow biopsy with flow cytometry

• Chemotherapy (for moderate/severe)


Treatment
• Life expectancy is often near-normal

This patient with 2 recent episodes of pneumonia has splenomegaly, anemia, thrombocytopenia,
mild leukocytosis, and peripheral blood smear evidence of a leukocyte with "hairy" cytoplasmic
projections suggesting the diagnosis of hairy cell leukemia.

Hairy cell leukemia is a chronic B-cell malignancy that occurs primarily in people age
>50. Manifestations arise due to the neoplastic infiltration of the bone marrow, spleen, and
peripheral blood as follows:

• Bone marrow infiltration – causes bone marrow fibrosis with cytopenias. This may
lead to recurrent infections (granulocytopenia), bleeding/bruising (thrombocytopenia), or
fatigue/weakness (anemia).
• Splenic infiltration – causes splenomegaly. Mass effect may lead to symptoms of
abdominal fullness or early satiety (gastric compression).
• Peripheral blood infiltration – peripheral blood smear shows abnormal circulating
lymphocytes with "hairy" cytoplasmic projections. Although most patients are
leukopenic due to bone marrow infiltration, a minority (such as this patient) have mild
leukocytosis due to increased circulating lymphocytes.

Hairy cell leukemia does not typically cause hepatomegaly, lymphadenopathy, or B


symptoms. Diagnosis requires bone marrow biopsy with flow cytometry.
(Choice A) Acute lymphoblastic leukemia is seen primarily in children (not adults) and usually
manifests with symptoms related to neutropenia, anemia, or
thrombocytopenia. Lymphadenopathy is common, and peripheral blood smear typically shows
lymphoblasts (small cells, scant cytoplasm), not lymphocytes with "hairy" projections.
(Choice B) Chronic lymphocytic leukemia is usually characterized by significant leukocytosis
(often >100,000/mm3), lymphadenopathy and B symptoms (unlike this patient). Peripheral
smear typically shows "smudge cells".

(Choice C) Gaucher disease is a lysosomal storage disease that may manifest with massive
splenomegaly and cytopenias. However, hepatomegaly is universal and a "hairy" leukocyte
would not be seen on peripheral smear.
(Choice E) Hodgkin lymphoma is a B-cell neoplasm with characteristic Reed-Sternberg cells
that usually manifests with a mediastinal mass or a painless, enlarged lymph node (often
cervical).

(Choice F) Infectious mononucleosis may be associated with splenomegaly and atypical


lymphocytosis. However, patients tend to be young and have the classic triad of fever, tonsillar
pharyngitis, and lymphadenopathy (unlike this patient).

(Choice G) Myelodysplastic syndrome is a malignant stem cell disorder that may cause
anemia, leukopenia, and/or thrombocytopenia. Splenomegaly and leukocytosis are not typical
features of myelodysplastic syndrome.

Educational objective:
Hairy cell leukemia is a chronic B-cell neoplasm that infiltrates the bone marrow, spleen, and
peripheral blood, resulting in cytopenias, splenomegaly, and peripheral smear evidence of
atypical lymphocytes with "hairy" cytoplasmic projections. Hepatomegaly, lymphadenopathy,
and B symptoms are uncommon. Diagnosis is made with bone marrow biopsy.
*Chronic myeloid leukemia (CML)

Chronic myeloid leukemia (CML) is a clonal myeloproliferative disorder driven by the


abnormal fusion gene BCR-ABL (due to a translocation between chromosomes 9 and 22). This
gene causes leukemogenesis due to a constitutively active tyrosine kinase. First-line treatment
for most CML patients involves tyrosine kinase inhibitors such as imatinib. Although these
drugs are not curative, they often can induce long-term remission.
Patients with CML may present asymptomatically, although fatigue, night sweats, weight loss,
and abdominal fullness (due to splenomegaly) are not uncommon. Patients may experience
weight loss and early satiety from splenomegaly. Peripheral blood smear in CML reveals
dramatic leukocytosis (often >100,000/mm3) with absolute basophilia and a shift toward very
early neutrophil precursors (promyelocytes, myelocytes).
*Hemophilia A & B
Hemophilia A & B
• X-linked recessive
Inheritance
• Delayed/prolonged bleeding after mild trauma
o Hemarthrosis, intramuscular hematomas
Clinical o Gastrointestinal or genitourinary tract bleeding
features o Intracranial hemorrhage
• Complications: hemophilic arthropathy

• ↑ Activated PTT
• Normal platelet count & PT
Laboratory
findings • Absent or ↓ factor VIII (hemophilia A) or factor IX
(hemophilia B) activity

• Factor replacement
Treatment • Desmopressin for mild hemophilia A

This patient's thigh hematoma after minor trauma and family history of a bleeding disorder in a
maternal uncle are concerning for hemophilia. Hemophilia is an X-linked recessive bleeding
disorder due to deficiency of factor VIII (hemophilia A) or factor IX (hemophilia B). The most
common sites of bleeding are the joints (80%), especially the knee. Hemarthrosis presents
with joint pain and swelling after minimal or no trauma, and episodes typically begin during
toddlerhood when the child is ambulatory. Hemorrhage into the skeletal muscle (ie, hematoma)
after minor trauma is also common.

Characteristic laboratory findings include a prolonged PTT and decreased or absent factor
VIII or IX activity. Treatment involves replacing the missing factor with a recombinant or
purified product.
*warfarin & inherited protein S

Warfarin is a coumarin anticoagulant most commonly used in the setting of thrombophilia and
cardiac conditions that predispose to intracardiac thrombus formation. Warfarin functions by
inhibiting synthesis of the vitamin K-dependent clotting factors II, VII, IX, and X, as well as the
vitamin K-dependent anticoagulant proteins C and S. As a result, it can cause a decrease in
measured levels of protein S. This may lead to an incorrect diagnosis of inherited protein S
deficiency. If possible, vitamin K antagonists such as warfarin should be discontinued for 2
weeks prior to evaluating protein S levels.

(Choice A) Aspirin is an antiplatelet agent that inhibits cyclooxygenase-1, thereby inhibiting


thromboxane A2 synthesis.
(Choice B) Clopidogrel is an antiplatelet agent that prevents platelet activation by blocking
adenosine diphosphate receptors on the surface of platelets.

(Choice C) Heparin activates antithrombin III, which in turn inactivates factor IIa (thrombin),
factor IXa, and factor Xa.

(Choice D) Simvastatin is a lipid-lowering agent that functions by inhibiting 3-hydroxy-3-


methylglutaryl-coenzyme reductase.

Educational objective:
Warfarin is an anticoagulant agent that functions by inhibiting the synthesis of vitamin K-
dependent factors II, VII, IX, and X, protein C, and protein
Neuromyelitis optica

Neuromyelitis optica
• Immune-mediated demyelination & destruction of axons in
Pathophysiology the optic nerves, spinal cord, & brainstem

• Acute attacks of neurologic deficits


• 10:1 female/male ratio
• Optic neuritis
Clinical features • Transverse myelitis
• Area postrema syndrome (intractable hiccups, nausea &
vomiting)

• MRI findings
o Brain: optic nerve enhancement
o Spinal cord: longitudinally extensive central necrotic

Diagnosis lesions
• Aquaporin-4 autoantibody
• CSF analysis: absence of oligoclonal bands virtually
excludes MS

MS = multiple sclerosis.

This patient has severe progressive neurologic deficits, including transverse myelitis (rapidly
progressive weakness and sensation deficits with a localizing spinal sensory level) and bilateral
optic neuritis, symptoms concerning for multiple sclerosis (MS). However, several features
suggest neuromyelitis optica (NMO).

NMO is a severe autoimmune disease that attacks both myelin and axons in the CNS. The
clinical presentation is often dramatic, with severe neurologic deficits manifesting over a few
days in otherwise young, healthy patients (median age ~40). MRI reveals longitudinally
extensive (>3 vertebral segments) spinal cord inflammation (usually <2 in MS), and analysis
of cerebrospinal fluid (CSF) is negative for oligoclonal bands (present in ~90% of patients
with MS). The presence of aquaporin-4 (AQP4) autoantibodies channels confirms the
diagnosis. These are preferentially expressed in the optic nerves, spinal cord, and area
postrema of the brainstem, resulting in the following:

• Optic neuritis
• Transverse myelitis
• Area postrema syndrome (vomiting, hiccups)

NMO is associated with high rates of longterm disability and mortality. Because of differences
in prognosis and treatment, NMO must be distinguished from MS. Additional features more
commonly seen in NMO include gray matter involvement (vs. white matter predominant in MS)
and bilateral optic neuritis. In addition, AQP4 autoantibodies are specific for NMO (Choice
D).

(Choice A) Positive antinuclear antibodies with negative CSF oligoclonal bands occur in
systemic lupus erythematosus (SLE), a multisystem autoimmune disorder that can have
manifestations in the CNS, including transverse myelitis. However, neurologic manifestations
of SLE more commonly include stroke, seizures, and psychiatric syndromes (eg, psychosis). In
addition, this patient's MRI findings and AQP4 autoantibodies are consistent with NMO.

(Choice B) Guillain-Barré syndrome (GBS) presents with rapidly progressive ascending


paralysis, but significant sensory loss with a distinct spinal level is more consistent with
transverse myelitis. Optic neuritis, MRI findings, and positive AQP4 autoantibodies are not
consistent with GBS.

(Choice E) Certain acute viral infections (eg, West Nile), can impact the spinal cord and brain
(ie, acute encephalomyelitis); rarely, they could cause optic neuritis and/or transverse
myelitis. However, viral encephalomyelitis usually presents with fever and signs of
encephalitis (eg, mental status changes) or meningitis (eg, neck stiffness). Extensive transverse
myelitis and the presence of AQP4 autoantibodies are not consistent with viral
encephalomyelitis.

Educational objective:
Neuromyelitis optica is a severe progressive CNS demyelinating disorder that typically leads to
transverse myelitis, optic neuritis, and area postrema syndrome. MRI reveals extensive spinal
cord inflammation of white and gray matter, cerebrospinal fluid does not contain oligoclonal
bands, and serum aquaporin-4 autoantibodies are specific to the diagnosis.
*Acute lymphoblastic leukemia

Acute lymphoblastic leukemia


• Most common childhood cancer
Epidemiology • Peak age: 2-5 years

• Nonspecific systemic symptoms (eg,


fever, weight loss)
• Leukemic cells overcrowd bone
marrow
o Pallor/fatigue (anemia)
o Easy bruising/petechiae
(thrombocytopenia)
o Bone pain
• Extramedullary spread
Clinical
o Lymphadenopathy
features
o Hepatosplenomegaly
o Testicular enlargement
• Mediastinal mass (T cell lineage):
airway compression &/or superior vena
cava syndrome
• Leptomeningeal spread: neurologic
symptoms (eg, cranial nerve deficits,
meningismus)

• Complete blood count*


• Bone marrow biopsy (>20% blasts is
Evaluation diagnostic) with flow cytometry
& diagnosis • Lumbar puncture to evaluate for CNS
involvement

*≥2 cytopenias (leukocytes may be ↓ or ↑), ± blasts on


peripheral smear.

This adolescent has an anterior mediastinal mass with associated superior vena cava (SVC)
syndrome, a condition in which SVC obstruction leads to venous swelling of the head and neck
(eg, jugular venous distension), as well as laryngeal edema (eg, cough, difficulty breathing). In
the setting of pancytopenia and testicular enlargement, these findings are worrisome for acute
lymphoblastic leukemia (ALL) .

Although most common in young children, ALL in adolescents and young adults is often
associated with a mass of thymic origin (eg, anterior mediastinal mass) that causes compressive
symptoms. Extramedullary leukemic spread commonly results in hepatosplenomegaly and
may cause nontender, testicular enlargement, as seen here. In addition, classic findings of
bone marrow failure may be present and include fatigue/pallor from anemia, bruising caused by
thrombocytopenia, or recurrent infections due to neutropenia; however, white blood cell count
is often variable because leukocytosis can occur once leukemic cells overcrowd the bone
marrow and spill into the periphery.

Diagnosis of ALL is confirmed by abnormally increased lymphoblasts (>20%) on bone marrow


aspirate. In the case of a mediastinal mass, flow cytometry of the bone marrow aspirate usually
reveals T lymphoblasts as the cells of origin due to their proliferation within the thymus during
lymphocyte maturation.

(Choice B) Acute myeloid leukemia (AML) can present with hepatosplenomegaly and
pancytopenia. However, compared to AML, ALL is a much more likely diagnosis in
children. In addition, mediastinal involvement with subsequent SVC syndrome would be
unusual in AML because the thymus is the site of T lymphocyte (rather than myeloid cell)
proliferation; and testicular enlargement is not typical of AML.

(Choice C) Burkitt lymphoma is a neoplasm of mature B cells associated with Epstein-Barr


virus infection and typically affects children. Although pancytopenia and hepatosplenomegaly
can occur, patients typically have a mass involving the mandible or abdominal viscera;
mediastinal involvement is rare.

(Choice D) Chronic lymphocytic leukemia can present with hepatosplenomegaly, anemia, and
thrombocytopenia, but patients are usually older adults (median age 70) and have
lymphocytosis. This patient's age, mediastinal mass, and leukocyte count are inconsistent with
this diagnosis.

(Choice E) Thymoma is a common cause of a mediastinal mass that can cause obstructive
symptoms but typically occurs in patients age >40, making this diagnosis unlikely. Moreover,
testicular examination would be normal, and pancytopenia would not be expected.

Educational objective:
Acute lymphoblastic leukemia should be considered in an adolescent patient with an anterior
mediastinal mass in addition to pancytopenia, hepatosplenomegaly, and testicular
enlargement. Presentation often relates to compression of mediastinal structures, such as
superior vena cava syndrome causing jugular venous distension and difficulty breathing/cough
*Hodgkin lymphoma

Hodgkin lymphoma
• Bimodal peak incidence: age 15-35 & >60
Epidemiology • Association with EBV in immunosuppression

• Painless lymphadenopathy
• Mediastinal mass
Manifestations • B symptoms (ie, fever, sweats, weight loss)
• Pruritus

• Lymph node biopsy


Diagnosis • Reed-Sternberg cells on histology

EBV = Epstein-Barr virus.

This patient with a large anterior mediastinal mass, pruritus, alcohol-associated pain, elevated
lactate dehydrogenase (LDH), and eosinophilia likely has Hodgkin lymphoma (HL).

HL is a lymphoid neoplasm that most often affects those age 15-35 or >60. It generally
develops in a single lymph node and spreads to contiguous nodes via the lymphatic and thoracic
ducts. Although most patients have painless peripheral lymphadenopathy in the
cervical/supraclavicular chains, approximately 50% have significant mediastinal
lymphadenopathy. This can compress adjacent structures and cause a nonproductive cough,
shortness of breath, and/or retrosternal pain.

Patients with HL occasionally have severe pain in areas of lymphadenopathy (eg, chest)
following exposure to small quantities of alcohol (eg, drinking a beer). The etiology is unclear,
but the pain may be due to alcohol-induced vasodilation within the lymph nodes, causing
capsular distension. Patients also frequently have long-standing fatigue, B symptoms, and
pruritus. Laboratory evaluation often reveals normocytic anemia, eosinophilia (tumor cells
secrete cytokines that generate eosinophils), lymphopenia, and elevated LDH (associated with
high tumor burden). Lymph node biopsy showing Reed-Sternberg cells is diagnostic.

(Choice A) Germ cell tumors can also cause an anterior mediastinal mass, compressive
symptoms (eg, nonproductive cough), and elevated LDH levels. However, the tumors most
often arise in individuals age 20-40 (vs 65), and eosinophilia is uncommon. In addition, pain
does not typically worsen with exposure to alcohol, and pruritus is unexpected.

(Choice B) Histoplasmosis occasionally causes a mediastinal mass (eg, due to granuloma


formation). However, most patients have pulmonary infiltrates, fever, and other systemic
symptoms (eg, headache, myalgia). In addition, eosinophilia is uncommon, and there is often
an antecedent history of exposure to bird (eg, chicken coop) or bat (eg, cave) droppings.
(Choice D) Sarcoidosis can present with chest pain, cough, anemia, and
eosinophilia. However, it is usually characterized by hilar (vs mediastinal) lymphadenopathy,
and most cases arise in young adults. ACE (vs LDH) is usually elevated.

(Choice E) Thymomas often cause an anterior mediastinal mass, and their growth can cause
compressive symptoms (eg, nonproductive cough). However, eosinophilia is uncommon, and
alcohol-associated pain and pruritus suggest HL. In addition, patients with thymoma often
develop paraneoplastic myasthenia gravis.

Educational objective:
Hodgkin lymphoma is a common cause of a mediastinal mass that often presents with
compressive symptoms (eg, cough, shortness of breath, retrosternal pain). Peripheral
lymphadenopathy, B symptoms, pruritus, and alcohol-associated pain may also be
present. Laboratory evaluation often reveals elevated serum lactate dehydrogenase and
eosinophilia.
*Inhibitor vs Factor Deficiency
*Transfusion reactions associated with hypotension

Transfusion reactions associated with hypotension


Reaction Onset* Cause Clinical features
• Shock,
Seconds to Recipient anti-IgA angioedema/urticaria &
Anaphylaxis
minutes antibodies respiratory distress

• Respiratory distress &


Transfusion- Donor noncardiogenic
Minutes to pulmonary edema
related acute lung antileukocyte
hours • Bilateral pulmonary
injury antibodies
infiltrates

• Fever, flank pain,


Minutes to ABO
Acute hemolysis hemoglobinuria & DIC
hours incompatibility
Bacterial • Fever, chills, septic
Minutes to
Bacterial sepsis contamination of shock & DIC
hours
donor product
*Time after transfusion initiation.

DIC = disseminated intravascular coagulation.

This patient's rapid deterioration immediately after blood transfusion initiation is consistent
with an anaphylactic reaction. Anaphylaxis occurs seconds to minutes after initiation of
transfusion due to massive histamine release. Symptoms include respiratory distress (eg,
dyspnea, hypoxia), angioedema, and hypotension. Wheezing is often appreciated, although
decreased breath sounds can occur with severe bronchospasm preventing air entry. Vomiting
and urticaria may also be present. Without treatment, symptoms can rapidly progress to
respiratory failure and shock.

The first step in management is to immediately stop the transfusion and administer
intramuscular epinephrine. Bronchodilators and antihistamines are typically also
administered. In some cases, hemodynamic and respiratory support (eg, vasopressors,
mechanical ventilation) may be required.

Patients with IgA deficiency are at higher risk of anaphylactic transfusion reaction because
they may have anti-IgA antibodies that react with donor blood IgA and contribute to histamine
release. Although IgA deficiency can present with recurrent sinopulmonary infections, it is
often asymptomatic and may be diagnosed only after an anaphylactic transfusion
reaction. Future transfusions should include IgA-deficient plasma and washed red blood cell
products.
(Choice B) Aspiration pneumonitis is caused by gastric acid aspiration, most commonly in the
setting of altered mental status or dysphagia. It can lead to hypoxia but typically causes
crackles and would be unexpected in a patient with no history of aspiration.

(Choice C) Fat embolism can be a complication of femoral or pelvic fracture and typically
presents with respiratory distress and hypoxia. However, patients typically develop symptoms
>24 hours after injury and often also have altered consciousness and petechial rash.

(Choice D) Acute hemolytic transfusion reaction from ABO incompatibility is rare but can
occur due to clerical errors (eg, misidentification of patient). Patients are hypotensive but also
have fever, flank pain, and hemoglobinuria.

(Choice E) Febrile nonhemolytic transfusion reaction, the most common adverse reaction to
transfusion, occurs within 1-6 hours of transfusion. During blood storage, leukocytes release
cytokines that, when transfused, cause transient fevers, chills, and malaise. Febrile
nonhemolytic transfusion reaction is unlikely given this patient's respiratory distress, shock, and
lack of fever.

(Choice F) Transfusion-associated circulatory overload is characterized by pulmonary edema


due to volume overload. Patients have respiratory distress and hypoxia, but hypotension would
not be expected (many patients are hypertensive).

Educational objective:
Anaphylactic transfusion reaction is characterized by a rapid onset (seconds to minutes) of
respiratory distress and hypotension, which quickly progresses to respiratory failure and
shock. Acute management includes immediate transfusion cessation and administration of
epinephrine. IgA-deficient patients are at increased risk.
*Budd-Chiari syndrome

Budd-Chiari syndrome
• Hepatic venous outflow obstruction
• Usually due to:
o Myeloproliferative disorder (eg, PV)
Etiology
o Malignancy (eg, hepatocellular carcinoma)
o Oral contraception use/pregnancy

• Acute
o Jaundice, hepatic encephalopathy, variceal bleeding
o Prolonged INR/PTT; elevated transaminases
• Subacute/chronic
Manifestations
o Vague, progressive abdominal pain
o Hepatomegaly, splenomegaly, ascites
o Mild/moderate elevation in bilirubin, transaminases

• Abdominal Doppler ultrasonography: ↓ hepatic vein flow


Diagnosis • Investigation for underlying disorders (eg, JAK2 testing for PV)

PV = polycythemia vera.

This patient's aquagenic pruritus, facial plethora, and dramatically elevated hematocrit raise strong suspicion for
polycythemia vera (PV), a chronic myeloproliferative disorder marked by elevated red blood cell
mass. Although PV is often discovered incidentally on routine laboratory testing, patients may have aquagenic
pruritus (itching worse with water), erythromelalgia (burning in feet/hands), transient vision disturbances
(from hyperviscosity), or complications related to thrombosis/hemorrhage. The etiology of some of these
findings is unclear, but improvement in pruritus and erythromelalgia with aspirin may reflect a causative role
for prostaglandins (which are pruritogenic).

Risk of venous and arterial thrombosis is significantly increased in PV due to increased blood viscosity and
abnormalities in platelet and leukocyte function. One classic thrombotic complication is Budd-Chiari
syndrome, a disorder marked by hepatic venous outflow obstruction (usually due to hepatic vein
thrombosis). Manifestations may be acute, subacute, or chronic; subacute cases often present with progressive
abdominal discomfort, ascites (eg, abdominal distension, flank dullness), and hepatosplenomegaly (this may
also be caused by PV directly). Mild/moderate elevations in liver transaminases, bilirubin, and alkaline
phosphatase also commonly occur.

The diagnosis of Budd-Chiari syndrome is established with noninvasive imaging such as abdominal Doppler
ultrasonography, which typically shows abnormal hepatic blood flow, hepatomegaly, splenomegaly, and
ascites. Patients with suspected PV (as in this case) should also undergo blood testing for the JAK2 V617F
mutation, which is almost always positive.
*Autoimmune hemolytic anemia (AIHA)
Autoimmune hemolytic anemia (AIHA)
Warm agglutinin AIHA Cold agglutinin AIHA
• Drugs (eg, penicillin)
• Viral infections • Infections (eg, Mycoplasma
• Autoimmune (eg, SLE) pneumoniae infection &
Etiology • Immunodeficiency states infectious mononucleosis)
o Lymphoproliferative (eg, • Lymphoproliferative diseases
CLL)

• Symptoms of anemia
• Livedo reticularis & acral
• Asymptomatic to life-
cyanosis with cold exposure
Clinical threatening anemia
that disappear with warming
• Direct Coombs positive with
presentation • Direct Coombs positive with
anti-IgG, anti-C3, or both
anti-C3 or anti-IgM, but usually
not IgG

• Corticosteroids
• Avoidance of cold temperatures
• Splenectomy for refractory
Treatment • Rituximab ± fludarabine
disease

• Venous thromboembolism • Ischemia & peripheral gangrene


Complications • Lymphoproliferative disorders • Lymphoproliferative disorders

CLL = chronic lymphocytic leukemia; SLE = systemic lupus erythematosus.


*Causes of hemolysis

Causes of hemolysis
• Microangiopathic hemolytic anemia (eg, disseminated intravascular coagulation)
• Transfusion reactions
Intravascular • Infections (eg, clostridial sepsis)
hemolysis • Paroxysmal nocturnal hemoglobinuria
• Intravenous Rho(D) immunoglobulin infusion

• Intrinsic RBC enzyme deficiencies (eg, G6PD)


• Hemoglobinopathies (eg, sickle cell, thalassemia)
• Membrane defects (eg, hereditary spherocytosis)
Extravascular
• Hypersplenism, intravenous immunoglobulin infusion
hemolysis
• Warm- or cold-agglutinin autoimmune hemolytic anemia (most cases)
• Infections (eg, Bartonella, malaria)

G6PD = glucose-6-phosphate dehydrogenase; RBC = red blood cell.

This patient's pallor, jaundice, splenomegaly, and laboratory findings are consistent with hemolytic
anemia. The most likely diagnosis is autoimmune hemolytic anemia (AIHA), which is due to autoantibodies
to red blood cells (RBCs).

Hemolysis can occur intra- or extravascularly. Intravascular hemolysis is due to significant RBC structural
damage resulting in RBC destruction within the intravascular space (eg, paroxysmal nocturnal hemoglobinuria,
disseminated intravascular coagulation). The hemoglobin released from hemolyzed RBCs binds to haptoglobin,
and the hemoglobin-haptoglobin complex is cleared by the liver. This leads to markedly reduced serum
haptoglobin (to undetectable levels). RBC hemolysis also results in elevated indirect bilirubin levels (from
heme breakdown) and raised serum lactate dehydrogenase (LDH) levels (released from RBCs).

In extravascular hemolysis, the RBCs are predominantly destroyed by phagocytes in the reticuloendothelial
system (eg, lymph nodes, spleen). As a result, there is less hemoglobin release than in intravascular hemolysis,
so laboratory results usually show normal to slightly low haptoglobin, slightly elevated LDH, and elevated
indirect bilirubin. Extravascular hemolysis can be due to causes such as antibody-mediated RBC destruction
(eg, most cases of AIHA) or intrinsic RBC enzyme (eg, glucose-6-phosphate dehydrogenase deficiency) or
membrane (eg, hereditary spherocytosis) defects.

This patient's laboratory findings (anemia, indirect hyperbilirubinemia, low-normal haptoglobin, and slightly
elevated LDH) all suggest extravascular hemolysis; her peripheral smear shows spherocytes without central
pallor. These findings can be seen in AIHA as well as in hereditary spherocytosis. AIHA usually has a
negative family history and positive Coombs test. Hereditary spherocytosis usually has a strong family history
and a negative Coombs test. If the Coombs test is negative and there is still a high index of suspicion for AIHA,
a micro-Coombs test can be performed to confirm the diagnosis. This patient most likely has warm-agglutinin
(IgG autoantibody mediated-) AIHA from an autoimmune cause. AIHA usually causes extravascular
hemolysis, although some degree of intravascular hemolysis can occur, especially with IgM-mediated AIHA
*Anemia of prematurity (AOP)

Anemia of prematurity
• ↑ Oxygenation at birth → ↓ EPO production
o Impaired transition from hepatic to renal EPO*
• Exacerbating factors in premature infants:
Pathogenesis o ↓ RBC life span
o Frequent blood draws
o Iron depletion

• Often asymptomatic
Clinical features • Tachycardia, poor weight gain, apnea, hypoxia

• Normocytic, normochromic anemia


Laboratory
• Inadequate reticulocyte response
findings
• RBC transfusion if severe or symptomatic
• Minimize exacerbating factors (eg, iron supplementation, limit
Management
blood draws)

*Typically occurs in the 3rd trimester.

EPO = erythropoietin; RBC = red blood cell.


This patient has normocytic, normochromic anemia with a minimal reticulocyte response,
which indicates that the bone marrow is not producing adequate red blood cells (RBCs) to
compensate for the anemia. The most likely cause in this 7-week-old, preterm infant is anemia
of prematurity (AOP).

After birth, increased oxygenation (ie, breathing, ductus closure) triggers reduced
erythropoietin (EPO) production by the liver and kidney. Low levels of circulating EPO
impair erythropoiesis in the bone marrow, which normally causes a mild, transient anemia that
reaches a nadir of around 9-11 g/dL at age 2-3 months in term infants (ie, physiologic anemia of
infancy). Compared to term infants, preterm infants have a lower initial hematocrit at birth, a
shorter erythrocyte life span, and less capacity to produce renal EPO. Therefore, their
erythrocyte nadir tends to be more severe (eg, ~7 g/dL) and occur earlier (eg, 1-2 months),
resulting in AOP. Frequent phlebotomy in the neonatal intensive care unit can further
exacerbate the condition.

Many cases of AOP are asymptomatic. However, mild tachycardia is common and a flow
murmur, apnea, and poor weight gain also can occur. Laboratory evaluation shows
normocytic, normochromic anemia with normal platelets and leukocytes. Although the
absolute reticulocyte count may be normal for age, reticulocytosis in response to anemia is
absent, as seen in this patient.
*SSRI- & SNRI-associated sexual dysfunction

SSRI- & SNRI-associated sexual dysfunction

• Decreased libido
• Anorgasmia
Features • Delayed ejaculation
• Common cause of nonadherence

• Rule out sexual dysfunction due to depression, medical conditions, primary


sexual disorder, stress/relationship issues, substance abuse
Assessment & • Switch to non-SSRI or non-SNRI antidepressant: Bupropion or mirtazapine
management • Adjunctive therapy with sildenafil or bupropion
• Dose reduction for patients on high-dose SSRI (monitor for loss of efficacy)

SNRI = serotonin-norepinephrine reuptake inhibitor; SSRI = selective serotonin reuptake inhibitor.

The patient is most likely experiencing SSRI-induced sexual dysfunction secondary to treatment with
escitalopram. Both selective serotonin reuptake inhibitors (SSRIs) and serotonin-norepinephrine reuptake
inhibitors (SNRIs) can cause sexual side effects. In women, these effects may include decreased sexual
interest and arousal if mild or moderate, or anorgasmia if severe. In men, decreased libido, and in more
severe cases erectile dysfunction and inability to ejaculate, may occur.

Psychological factors, substance use, and medical conditions such as vascular, hormonal, and neurological
disorders may also cause sexual dysfunction. However, despite the presence of baseline obesity, diabetes, and
hypertension in this patient, these conditions are stable and less likely to be responsible for the recent onset of
her worsening sexual dysfunction (Choices C, F, and G).

(Choice A) Bupropion does not cause sexual dysfunction and is often tried as adjunctive therapy to manage
SSRI-related sexual dysfunction.

(Choice B) Depression-related sexual dysfunction would be expected to improve along with the patient's other
depressive symptoms and therefore would not explain her worsening symptoms.

(Choice E) Female sexual interest/arousal disorder is characterized by significant impairment in sexual arousal
and interest lasting 6 months or longer; it is unrelated to substances, medications, or other medical etiologies.

Educational objective:
Serotonergic antidepressants such as selective serotonin reuptake inhibitors (SSRIs) and serotonin-
norepinephrine reuptake inhibitors (SNRIs) can cause sexual dysfunction in both women and men. Psychiatric
and other medical conditions can also cause sexual dysfunction. However, the temporal association of the
symptoms (or their worsening) to SSRI/SNRI antidepressant use often indicates a medication-related cause.
*Thrombotic thrombocytopenic purpura

Thrombotic thrombocytopenic purpura


• ↓ ADAMTS13 level → uncleaved vWF multimers → platelet

Pathophysiology trapping & activation


• Acquired (autoantibody) or hereditary

• Hemolytic anemia (↑ LDH, ↓ haptoglobin) with schistocytes


• Thrombocytopenia (↑ bleeding time, normal PT/PTT)

Sometimes with:
Clinical features
• Renal failure
• Neurologic manifestations
• Fever

• Plasma exchange
• Glucocorticoids
Management • Rituximab
• Caplacizumab

LDH = lactate dehydrogenase; vWF = von Willebrand factor.

This patient likely has thrombotic thrombocytopenic purpura (TTP), a life-threatening


thrombotic microangiopathy that classically causes a pentad of manifestations:

• Thrombocytopenia
• Microangiopathic hemolytic anemia (MAHA)
• Renal insufficiency
• Neurologic changes (eg, headache, confusion, coma, stroke)
• Fever

However, most patients do not have all 5 manifestations at the time of diagnosis. Cases are
most common in previously healthy individuals in middle age who are female, black, pregnant,
or obese.

Immune TTP is caused by autoantibodies against the plasma protease ADAMTS13. Low
levels of ADAMTS13 lead to the accumulation of long chains of von Willebrand factor on the
endothelial wall, which traps platelets and generates thrombi in areas of high shearing force (eg,
arterioles, capillaries). Patients with TTP will have MAHA (eg, schistocytes) on peripheral
blood smear; coagulation studies are generally normal (in contrast to those with disseminated
intravascular coagulation). Although purpura often occurs in the setting of severe
thrombocytopenia, it is not always present in patients with TTP and is not required for the
diagnosis.

TTP is life threatening and must be treated with emergency plasma exchange (PEX), which
removes the patient's plasma and replaces it with donor plasma. This replenishes ADAMTS13
and removes the autoantibodies. Without emergency PEX, mortality is approximately 90%.

TTP is closely related to hemolytic uremic syndrome (HUS). Both cause MAHA and
thrombocytopenia. Unlike TTP, HUS is typically associated with Escherichia coli O157:H7
infection and is primarily a disorder of the renal system; neurologic findings and fever are
uncommon
*cancer pain management

This patient has bone metastases with progressive pain despite specific cancer-directed
treatment (ie, androgen deprivation therapy, radiotherapy).

Mild cancer-related pain can usually be managed with nonopioid analgesics (eg,
acetaminophen, nonsteroidal anti-inflammatory drugs [NSAIDs]). However, if nonopioid
interventions are ineffective, as in this patient with moderate to severe (eg, >4/10) pain
limiting activity despite NSAIDs, short-acting opioids (eg, oxycodone, morphine) should be
offered. Short-acting opioids are safe and well tolerated when taken as directed, and the dose
and schedule can be easily adjusted to the patient's pain pattern.

Once short-acting opioids have been started, if the patient's pain requires frequent dosing or if
bedtime doses do not provide adequate relief through the night, conversion to long-acting
opioids can improve convenience (eg, less frequent dosing) and help stabilize pain control (eg,
steady drug levels). Options include extended-release morphine and transdermal
fentanyl. However, due to the risk of respiratory depression, patients not receiving opioids
should generally not be started on long-acting opioids initially (Choice E).

Rather, they should have a trial of short-acting opioids that can be readily titrated and then
converted to a long-acting opioid dose that can be safely tolerated. This approach provides
adequate analgesia while avoiding the risk of "overshoot" from the accumulation of long-acting
opioids prior to reaching their steady state. Even after transition to a long-acting formulation,
short-acting opioids should still be available for rescue treatment of breakthrough pain.

(Choice B) Naproxen has a longer duration of action than ibuprofen and can be used in patients
who require frequent dosing. Switching from one NSAID to another can be considered for
patients with mild pain, but this patient's activity-limiting pain warrants a trial of opioids.

(Choice C) Systemic glucocorticoids (eg, dexamethasone) are used urgently for the treatment
of malignant neurologic compromise (eg, spinal cord compression). They are also sometimes
used as adjunctive therapy in patients who have bone pain without neurologic deficits if there is
incomplete improvement on opioid therapy alone.

(Choice D) Topical capsaicin is useful for mild to moderate joint and neuropathic pain (eg,
postherpetic neuralgia); it is unlikely to relieve metastatic bone pain.

Educational objective:
Mild cancer-related pain can usually be managed with nonopioid analgesics. However, if initial
interventions are not effective, intermittent doses of short-acting opioids should be offered. If
the pain requires frequent dosing or if bedtime dosing does not provide relief through the night,
then conversion to a long-acting opioid is helpful.
*(WM) vs (MM) vs (MGUS)

Waldenström macroglobulinemia vs multiple myeloma


Waldenström macroglobulinemia Multiple myeloma
• Hyperviscosity syndrome
• Osteolytic lesions/fractures
• Neuropathy
• Anemia
• Bleeding
Major manifestations • Hypercalcemia
• Hepatosplenomegaly
• Renal insufficiency
• Lymphadenopathy

• IgM • IgG, IgA, light chains


Monoclonal antibody
• Rouleaux • Rouleaux
Peripheral smear
• >10% clonal B cells • >10% clonal plasma cells
Bone marrow biopsy

Waldenstrom macroglobulinemia (WM) is a lymphoplasmocytic malignancy characterized


by the excessive production of monoclonal IgM antibody. Clinical manifestations of WM
stem from elevated serum IgM (hyperviscosity syndrome, neuropathy, cryoglobulinemia) and
neoplastic infiltration of tissue (hepatosplenomegaly, lymphadenopathy,
cytopenias). Peripheral blood smear may show rouleaux formation (or erythrocyte
agglutination) due to elevated serum protein. Serum protein electrophoresis (SPEP) is an
important screening study; patients with WM have a monoclonal spike (M-spike) of
IgM. Diagnosis is then confirmed by bone marrow biopsy showing >10% clonal B cells with
specific cytogenetic features.
This patient presents with manifestations of hyperviscosity syndrome (diplopia, tinnitus, headache,
dilated/segmented funduscopic findings), neuropathy (electric sensation), and evidence of infiltrative disease
(hepatosplenomegaly, anemia, thrombocytopenia), suggesting WM.

(Choice B) Patients with monoclonal gammopathy of undetermined significance (MGUS) also have M spikes,
but they are most commonly due to IgA, IgG, or IgD (IgM is seen in 15% of cases). MGUS is marked by
smaller M spikes (<3 g/dL), bone marrow biopsy with <10% monoclonal plasma cells, and, unlike this patient,
no end-organ effects (no anemia, hepatosplenomegaly, lymphadenopathy, or systemic symptoms).

(Choice C) Multiple myeloma (MM) is also a plasma cell neoplasm that produces a monoclonal antibody spike
by SPEP; however, patients with MM rarely (<0.5%) produce monoclonal IgM. MM is typically characterized
by monoclonal IgG, IgA, or light chains. In addition, patients with MM tend to present with bone pain rather
than hyperviscosity, neuropathy, and tissue infiltration.
*Cognitive behavioral therapy for insomnia (CBT-I)
This patient with chronic insomnia has developed behavioral maladaptions (eg, anxiety about sleep, sleeping
later on weekends, lying awake in bed for long periods of time). Cognitive behavioral therapy for insomnia
(CBT-I) is first-line treatment for patients with chronic insomnia to improve sleep behaviors; when
implemented correctly, it can successfully resolve chronic insomnia without medications.

In CBT-I, patients are first instructed to keep a sleep diary (eg, bedtime, approximate time to fall asleep, timing
and duration of nighttime awakenings, final wake time), enabling calculation of current sleep efficiency (ie,
number of hours spent asleep / number of hours in bed). Then, targeted strategies are implemented to increase
sleep efficiency to near 100%, increase sleep drive, and strengthen the association between being in bed and
sleeping. Strategies may include:

• Establishing an optimal and consistent wake and sleep time across weekdays and weekends (ie,
avoiding sleeping late)
• Limiting time spent in bed awake. Patients should use the bed only for sleep or sex (eg, instead of
watching television) and physically leave the bed if they are awake in bed >20 minutes.
• Improving sleep hygiene (eg, moderating caffeine intake, limiting alcohol use before bed, addressing
sleep anxiety)

(Choice A) Going to bed earlier typically worsens chronic insomnia because it reduces sleep efficiency (eg,
increasing time spent awake in bed waiting to sleep). Typically, CBT-I initially involves paradoxically
delaying bedtime (but keeping a set wake time) until sleep efficiency is 100%; gradually, bedtime is moved
earlier until the patient achieves a sufficient quantity of sleep.

(Choice B) Sleep difficulties can result from major depressive disorder, which is often treated with selective
serotonin reuptake inhibitors (eg, sertraline). However, this patient enjoys her job and has no evidence of low
self-esteem or feeling hopeless. Her reduction in social activities is more likely connected to her underlying
preoccupation about sleep, and CBT-I should be tried first.

(Choices C and D) Benzodiazepines and the nonbenzodiazepine hypnotic medication zolpidem can be used for
acute treatment of situational insomnia. However, long-term use is associated with adverse effects, including
somnolence, dependence, and cognitive impairment. Therefore, CBT-I should be tried first.

Educational objective:
In patients with chronic insomnia, first-line treatment involves cognitive behavioral therapy for insomnia (CBT-
I) to improve sleep hygiene and sleep efficiency. Patients should first keep a sleep diary, which helps to
identify opportunities to improve sleep efficiency (eg, limiting time in bed) and establish an optimal, consistent
sleep and wake time

You might also like